សំណួរត្រៀម្រឡងតេស្ត Level 3 2

You might also like

Download as pdf or txt
Download as pdf or txt
You are on page 1of 170

វិញ្ញ

ា សារត្រៀម្រឡងតរស្តភាសាអង់តលេស្ ក្មិរ ៣
តរៀរចំតោយសាស្រ្សាតចារយរត្ងៀនភាសាអង់តលេស្
តៅសាកលវិទ្យាល័យ អនតរជារិ

Grammar Test
Part 1

I. Choose one answer only to match with the appropriate tenses.

1. Every Monday, Sreypov ___________her kids to football practice.


a. drives
b. is driving
c. has been driving
d. was driving
2. Usually, I work as a secretary at ABT, but this summer I ___________French at a language
school in Paris. That is why I am in Paris.
a. study
b. am studying
c. have been studying
d. was studying
3. Shhhhh! Be quiet! John (sleep) ___________.
a. sleeps
b. is sleeping
c. has been sleeping
d. was sleeping
4. Don't forget to take your umbrella. It ___________.
a. rains
b. is raining
c. has been raining
d. was raining
5. I hate living in Seattle because it always ___________.
a. rains
b. is raining
c. has been raining
d. was raining
6. I'm sorry I can't hear what you say because everybody ___________so loudly.
a. talks
b. is talking
c. has been talking
d. was talking
7. Justin currently ___________a book about his adventures in Tibet. I hope he can find a good
publisher when he is finished.
a. writes
b. is writing
c. has been writing
d. was writing
8. Jim: Do you want to come over for dinner tonight?
Denise: Oh, I'm sorry, I can't. I ___________to a movie tonight with some friends.
a. go
b. am going
c. have been going
d. was going
9. The business cards are normally printed by a company in New York. Their prices
___________inexpensive, yet the quality of their work is quite good.
a. are
b. are being
c. have been being
d. was being
10. This delicious chocolate ___________made by a small chocolate in Zurich, Switzerland.
a. is
b. are
c. was
d. were
11. She ____________ card. -She is typing a computer in her bedroom.
a. doesn’t play
b. don’t play
c. isn’t playing
d. wasn’t playing
12. What _______ you _________ every weekend? -I always read books and play tennis.
a. do/do
b. are/doing
c. does/do
d. were/doing
13. The vegetarian _________________ meat.
a. isn’t eating
b. don’t eat
c. wasn’t eat
d. doesn’t eat
14. Vandy and Dany are Chinese. They _______________ from Hong Kong.
a. are coming
b. came
c. come
d. comes
15. They will be here very soon. They ____________ by bus. a
a. are coming
b. came
c. come
d. comes
16. My youngest sister cannot answer the phone. She ________________a shower.
a. has
b. have
c. had
d. is having
17. She _____________ two new pairs of jeans.
a. have
b. has
c. had
d. is having
18. My daddy is not at home. He _______________ a car to work.
a. is driving
b. drives
c. drive
d. drove
19. Doctor said, “I ____________ the problem but I cannot help you.”
a. am seeing
b. see
c. sees
d. saw
20. This room is usually used for studying. But today it _______________ for meeting.
a. is use
b. used
c. was used
d. is being used
21. When I phoned my friends, they _____________ monopoly.
a. were playing
b. are playing
c. played
d. play
22. Yesterday at six I __________dinner.
a. was preparing
b. are preparing
c. prepared
d. prepare
23. The kids __________in the garden when it suddenly began to rain.
a. were playing
b. are playing
c. played
d. play
24. I ________ - I didn't hear you come in.
a. was sleeping
b. slept
c. am sleeping
d. sleep
25. I ________ to see her twice, but she wasn't home.
a. was coming
b. came
c. am coming
d. come
26. What ________? I was watching TV.
a. did you do
b. were you doing
c. are you doing
d. do you do
27. A: Why are you holding a piece of paper? B: I ____________a letter to my friends back home in
Texas.
a. will write
b. am going to write
c. wrote
d. write
28. A: I'm about to fall asleep. I need to wake up! B: I ____________you a cup of coffee. That will
wake you up.
a. will get
b. am going to get
c. got
d. get
29. A: I can't hear the television! B: I ____________it up so you can hear it.
a. will turn
b. am going to turn
c. turned
d. turn
30. We are so excited about our trip next month to France. We (visit) ____________Paris, Nice and
Grenoble.
a. will visit
b. are going to visit
c. visited
d. visit
31. Ted: It is so hot in here! Sarah: I (turn) ____________the air-conditioning on.
a. will turn
b. am going to turn
c. turned
d. turn
32. I think he (be) ____________the next President of the United States.
a. will be
b. am going to be
c. was
d. is
II. Identify the tenses with the following sentences.
1. I am writing a letter.
a. Present Simple
b. Present continue
c. Past Continue in Passive
2. We have been studying English at IU for three years.
a. Present Perfect Simple
b. Past Perfect Simple
c. Present Perfect Continue
3. This house has been built for ten years.
a. Past Simple in Passive
b. Present Simple
c. Present Perfect in Passive
4. Where were you this morning?
a. Past Simple in Passive
b. Past Continue in Passive
c. Past Simple
5. His car was stolen last week.
a. Past Simple in Passive
b. Past Simple
c. Past Perfect in Passive
6. The next lesson will be being taught by this time tomorrow.
a. Future Continue
b. Future Continue in Passive
c. Future Perfect Simple
7. I will be waiting for you.
a. Future Simple
b. Future Continue
c. Future Continue in Passive
8. I will have retired by the year 2020.
a. Future Continue
b. Future Perfect Simple
c. Future Perfect in Passive
9. I will have been studying here for 2 years.
a. Future Perfect Simple
b. Future Perfect in Passive
c. Future Perfect Continue
10. This letter is be being written by Smith.
a. Present Simple
b. Present Continue
c. Present Continue in Passive
11. She has been living in Kampot for ten years.
a. Present Continue
b. Present Simple
c. Present Perfect Continue
12. Some bread was bought from 3A Bakery's.
a. Past Simple
b. Past Simple in Passive Voice
c. Past Continue
13. This virus has been deleted for many days.
a. Present Perfect Simple in PV
b. Present Simple in PV
c. Present Perfect Simple
14. Someone has repaired this computer for many days.
a. Present Simple in PV
b. Present Simple
c. Present Perfect Simple
15. Is the newspaper being read by your grandfather?
a. Past simple in PV
b. Present Perfect Simple in PV
c. Present Continue in PV
16. Will the clothes be being washed by her at this time tomorrow?
a. Future Perfect Continue
b. Future Continue in PV
c. Future Simple in PV
17. How long had she been learning English at IU?
a. Past Perfect Continue in PV
b. Past Perfect Continue
c. Future Perfect Continue in PV
18. What time did his aunt come back from market this morning?
a. Past Simple
b. Past Perfect Simple
c. Past Simple in PV
19. How long will your daughter have been living in Kampong Soum?
a. Future Perfect Simple
b. Future Continue in PV
c. Future Perfect Continue
20. Is the homework done by his daughter every night?
a. Present Simple in PV
b. Past Simple in PV
c. Present Perfect in PV

III. Identify the following clauses if they are the dependent clauses or independent clauses.
a. Dependent Clause b. Independent Clause
1. I am staying in Phnom Penh hotel.
2. When she speaks English.
3. Because I want to marry her.
4. While I was doing my homework last night.
5. According to Smith (2009), English plays more important role in communication.
6. Where did you go yesterday?
7. How long have you been studying English at IU?
8. If I have free time.
9. Since we were living in Kampot town.
10. Whatever you want to eat.
11. Because she is only sixteen.
12. The file is on the desk.
13. When she teaches basic marketing.
14. Where the tourists outnumber the natives.
15. Can he fix the car by Friday?
16. That he would not want to change.
17. Although the clutch is broken.
18. Kunthea will call long distance.
19. The rate after midnight is cheaper.
20. If Sonita can splice the wires.
21. Because she is ill, I must work overtime.
22. Since duplicator is not working properly.
23. While you were out, three important clients called.
24. When the time is up, the class will turn in the papers.
25. The clerk was upset when the customer complained.
26. Miss. Bopha designed the newest office building.
27. The appointment was cancelled by the secretary.
28. Before you post this month’s checks.
29. It rained hard during the night.
30. Before you actually send your e-mail messages.
31. As you know, all memos should have a clear subject line.
32. The new trainees reviewed their notes while they waited for the instructor.
33. I lost the instruction manual that came with this answering machine.
34. Barney’s Bagels, which is located in Cranston, also sells sandwiches and pastries.
35. My supervisor reminded me that our company offers tuition reimbursement.
36. Any policyholder who wants extra coverage should contact his or her agent.
37. The customer asked whether the delivery could be made after 5p.m.
38. The figures that you requested are not available at this time.

IV. Identify the underline word in the following sentences whether it is a direct object, an
indirect object or an object of preposition.

a. Direct Object b. Indirect Object c. Object of Preposition

1. Mary wrote a letter to her aunt.


2. The teacher handed out the exams to the students.
3. Bill sent an invitation to the party to his friends.
4. The cat ran down the street.
5. Mother baked the cake for her daughter's party.
6. Give the papers to the lawyer today.
7. The soup was salty.
8. Jose wrote the postcard and sent it to his parents.
9. Nimol asked me some questions.
10. We wrote our parents a letter last week.
11. Kunthea is typing a computer.
12. Please show me your dictionary.
13. Nita sent us some mangoes.
14. Look there! Our children are swimming in the sea.
15. Don’t look at me!
16. Could I borrow you some money?
17. Could I borrow some money from you?
18. Shall we ask you some questions?
19. How long have you known her?
20. My parents bought me a new computer.
21. Can you please give me some money?

22. Chanthoeun always teaches the Parts of Speech to his students every Tuesday, Wednesday,
and Friday.

23. Please show us your book PARTS OF SPEECH!

24. Bye the way, how much does your phone cost?

25. Nisa sent her daughter a modern lab top.

26. Sonita told her own problems to her boyfriend.

27. Can we please borrow you your car?

28. Who does she love?

29. Don’t ask me about my own problems!

30. We are not sure if you gave us an invitation letter.

31. May you please tell me that story again?

V. Identify the underline word in the following sentences if it is a main verb, a modal
auxiliary verb, or principle auxiliary verb.
a. Main Verb b. Modal Auxiliary Verb c. Principle Auxiliary Verb
1. Does she speak English?
2. How many dictionaries do you have?
3. Where were you this morning?
4. What were you doing at this time yesterday?
5. How long have you been studying English at the International University?
6. We have two cats and three dogs.
7. You might stay home because it is raining.
8. Will you come here?
9. Could you swim when you were ten years old?
10. This lesson is being taught next day.
11. If you come late, the train leaves you.
12. Kanika does not have a course for this semester.
13. When will they come back here?
14. I am not reading newspaper.
15. Two of his children were playing tennis last Sunday evening.
16. What did you do last night?
17. We have never met our lecturer for one week.
18. The topic sentence might stand for four positions.
19. What you want me to do I can.
20. May you pass me a phone?
21. We do not live in Siem Reap.
22. What could you do when you were five years old?
23. Either Khama or Rithy must do housework every day.
24. How long has Chanthoeun been teaching English?
25. The newspaper was being read by his grandfather.
26. If you do not water the flowers, they die.
27. If I were Amara, I would be happy.
28. I will help you if I have enough time.
29. If I had not seen it with my own eyes, I would not have thought it possible.
30. What suites you; it does not suite me!
31. Neither our parents nor teacher can answer our questions.
32. Where were you eating breakfast at 7:30a.m.?

VI. Compete these sentences with the coordinating conjunctions.


a. for b. and c. nor d. but e. or f. yet g. so

1. My mum put on a coat, __________ she was cold.


2. Kunthea wants to get a scholarship, __________ she studies hard.
3. Dara needs some money to buy house __________ marry to his girlfriend.
4. Vibol does not have wife, __________ does he have girlfriend.
5. Some of my students had a small party, __________ they got the impressive result for the
final exam.
6. Does your aunt speak Chinese __________English?
7. We went to Praek Leab, __________ the class was cancelled.
8. Vanary is not beautiful enough, __________ she is friendly and lovely.
9. The food is delicious, __________ it is too expensive.
10. We did not understand the lessons, __________ we got the low marks for the final exam.
11. Maria tried to read a novel in French, ___________it was too difficult.
12. To get from Vancouver to Victoria, you can fly, ___________you can ride the ferry.
13. I bought a bottle of wine, ___________we drank it together.
14. The waiter was not very nice, ___________the food was delicious.
15. I went to buy a Rolling Stones CD, ___________the shop didn't have it.
16. Anna needed some money, ___________she took a part-time job.
17. Julie has a guitar, ___________she plays it really well.
18. The concert was cancelled, ___________we went to a nightclub instead.
19. I burned my tongue, ____________ the soup was still too hot to eat.
20. The physics instructor explained the theory, ___________ I did not understand it.
VII. Passive & Active Voice. Choose the correct tenses to match the following sentences.
1. They were interviewing her for the job. She ________________ for the job.
a. was being interviewed b. was interviewed c. has been interviewed
2. Tom is writing the letter.
The letter ________________ by Tom.
a. was written b. is being written c. has been written
3. Everyone understands English.
English ________________ by everyone.
a. is understood b. has been understood c. was understood
4. The employees brought up this issue during the meeting. This issue ________________ by the
employees during the meeting.
a. has been brought up b. is brought up c. was brought up
5. The professor told him not to talk in class. He ________________ by the professor not to talk in class.
a. has been told b. was told c. was being told
6. They say that women are smarter than men. Women ________________ to be smarter than men.
a. were being said b. were said c. are said
7. The fire has destroyed the house. The house ________________ by the fire.
a. has been destroyed b. was being destroyed c. is destroyed
8. She would have told you. You ________________ by her.
a. would have been told b. would be told c. were being told
9. She would reject the offer. The offer ________________ by her.
a. will have been rejected b. would be rejected c. will be rejected
10. This surprises me. I ________________ by this.
a. would have been surprised b. will be surprised c. am surprised
11. IFL bought Nessafe.
a) Nessafe had been bought by IFL. b) Nessafe was bought by IFL.
c) Nessafe is bought by IFL.
12. Fred asked Simon.
a) Simon was asked by Fred. b) Simon had been asked by Fred c) Simon is asked by
Fred.
13. Caroline paid for the meal.
a) The meal is paid for by Caroline. b) The meal has been paid for by
Caroline.
c) The meal was paid for by Caroline
14. Harry will meet you at the airport.
a) You will have been met by Harry at the airport. b) You will be met by Harry at the airport.
c) You would be met by Harry at the airport.
15. Tom respects Lindsay's opinion.
a) Lindsay's opinion would be respected by Tom. b) Lindsay's opinion was respected by Tom.
c) Lindsay's opinion is respected by Tom.
16. Jane upset me by what she said.
a) I would be upset by what Jane said. b) I was upset by what Jane said.
c) I would have been upset by what Jane said.
17. I'm going to inform John later today.
a) John would be informed by me later today. b) John was informed by me later today.
c) John is going to be informed by me later today.
18. The police arrested 12 people.
a) 12 people were arrested by the police. b) 12 people have been arrested by the
police.
c) 12 people should be arrested by the police.
19. Shiela might come to see you tomorrow.
a) You might be visited by Shiela tomorrow. b) You will might be visited by Shiela
tomorrow.
c) You would be visited by Shiela tomorrow.
20. Frank Sinatra sang My Way.
a) My Way was sung by Frank Sinatra. b) My Way had been sung by Frank Sinatra.
c) My Way would be sung by Frank Sinatra.
21. Julia rescued three cats.
a. Three cats are rescued by Julia. b. Three cats were rescued by Julia.
C. Three cats had been rescued by Julia.
22. Steven has forgotten the book.
a. The book has been forgotten by Steven. B. The book had been forgotten by Steven.
c. The book is forgotten by Steven.
23. The mechanic has not repaired the DVD recorder.
a. The DVD recorder has not been repaired by mechanic. B. The DVD recorder had not been
repaired by mechanic. C. The DVD recorder is not repaired by mechanic.
24. Sue puts the rucksack on the floor.
a. The rucksack has been put on the floor by Sue. B. The rucksack had been put on the floor by
Sue.
c. The rucksack is put on the floor by Sue.
25. The girls had lost the match.
a. The match has been lost by the girls. B. The match had been lost by the girls.
c. The match is lost by the girls.
26. They make shoes in that factory.
Shoes _________________in that factory.
a. are made b. has been made c. had been made
27. People must not leave bicycles in the driveway.
Bicycles _________________in the driveway.
a. might not be left b. must not be left c. is not left
28. They built that skyscraper in 1934.
That skyscraper _________________in 1934.
a. is built b. was built c. has been built
29. The students will finish the course by July.
The course _________________by July.
a. is finished b. was finished c. will be finished
30. They are repairing the streets this month.
The streets _________________this month.
a. are being repaired b. are repaired c. have been repaired
31. They make these tools of plastic.
These tools _________________of plastic.
a. are made b. are being made c. has been made
32. They have finished the new product design.
The new product design _________________.
a. is designed b. is being designed c. has been designed
33. They were cooking dinner when I arrived.
Dinner _________________when I arrived.
a. is being cooked b. was cooked c. was being cooked
34. Smithers painted 'Red Sunset' in 1986.
'Red Sunset' _________________in 1986 by Smithers.
a. is painted b. is being painted c. was painted
35. Did the plan interest you?
_________________in the plan?
a. are you interested b. were you interested c. have you been interested
36. They had finished the preparations by the time the guests arrived.
The preparations _________________by the time the guests arrived.
a. are finished b. had been finished c. were finished
37. You should take care when working on electrical equipment.
Care _________________when working on electrical equipment.
a. would be taken b. will be taken c. should be taken
38. Someone will speak Japanese at the meeting.
Japanese _________________at the meeting.
a. will be spoken b. would be spoken c. should be spoken
39. Karen is going to prepare the refreshments.
The refreshments _________________by Karen.
a. are going to be prepared b. are being prepared c. are prepared

VIII. Identify the underline word in the following sentences if it is a simple subject, a
complete subject or a compound subject.

a. Simple Subject b. Complete Subject c. Compound Subject

1. What is that man doing?


2. Yesterday morning, Nita and Rotha did not go to school.
3. By Sunday evening, his beautiful girlfriend will have studied master degree for two years.
4. This morning, who was your lecturer chatting with?
5. At eight O’clock tomorrow, Piseth, Chanthoeun, Virak, Rasy, and Sreypov are having a
meeting.
6. Some of my money is in the bank.
7. Neither everyone nor I like drinking beer.
8. As a result, either Chanthoeun or his students like listening to the English songs.
9. According to the explanation, some students don’t understand the prepositions clearly.
10. Smoking cigarette can damage your health.
11. By the end of July, Darika will have stayed in Japan for two weeks.
12. Yesterday, Nary, Khunthea, Sopheap, and Sothea were sitting outside the school at
5:00p.m.
13. While typing a computer, a handsome man with brown hair talked to me.
14. During Khmer New Year, neither my students nor I went to school.
15. What is everybody doing outside the class?
16. Excuse me. How old is your lovely daughter?
17. In the class, nobody is listening to his/her lecturer.
18. Two years ago, either our lecturer or parents were abroad.
19. Channa and Thearith are reading a newspaper in the living room.
20. While doing the major test, I sat next to one of my classmates.
11. When I got home, my mother and sister were cooking.
12. Either my classmates or I had a party after we finished the final examination.
13. Chariya, Sokleang, Amara, Virak, and Chanthoeun were talking while it was raining.
14. How long have you been studying English at the International University?
15. Yesterday evening, what did you and your brothers do?
IX. Identify the underline word if it is a simple predicate, a complete predicate or a
compound predicate in the following sentences. (10 marks)
a. Simple Predicate b. Complete Predicate c. Compound Predicate
1. The new baby has my old room.
2. We decorated the room with wallpaper.
3. A toy hangs over the crib.
4. The baby plays most of the day.
5. My mother sings to him at night.
6. My friend sewed clothes for the baby.
7. We are sitting and watching TV in the living room.
8. Jeremy wears the bib during meals.
9. We have been learning English for ten years.
10. My house was built in 1993.
11. Amara, Samoeun and I come from Kampot province.
12. Chanthoeun has 4 sisters and 3 brothers.
13. Either Rasy or Sokleang can speak English well.
14. My parents are the gods in my heart.
15. Everyone in my English class has a modern computer.
16. Where did you go last Sunday?
17. Yesterday evening, Dara, Borin and Virak were watching football match.
18. We have been waiting for you for ten minutes.
19. Who was sitting next to the window?
20. That man with the brown hair is sitting next to the window.
21. Who loves that beautiful girl?
22. Socheata, Chariya, Daneth, and I are sitting and watching T.V.
23. Neither of my students drinks wine and beer.
24. No one remembers to bring his/her dictionary.
25. Someone in my English class come and study music every weekend.
26. Living abroad, I had some problems.
27. Rotha, Sokhorn, Srey Tum, and Ramma either can speak or write Japanese fluently.
28. Either of those children can answer my questions correctly.
29. We really want to know what IU stands for.
30. Who neither smokes cigarette nor drinks beer?
X. Identify the underline word if it is a linking verb or an action verb. (10 marks)
a. Linking Verb b. Action Verb
1. The ghost appeared in the doorway.
2. The child appeared tired.
3. Bill became the president of the student council.
4. The cloth on the table felt soft and fuzzy.
5. Jeannie grows taller every day.
6. My father grows the tree every year.
7. John felt sick after lunch.
8. The leftover food from the picnic smelled rotten.
9. Steve smelled the flowers.
10. My mother tastes the soup.
11. One of my children likes learning computer.
12. Neither of her family stays in London.
13. Cheata, Navy, Sothea and David are swimming in the sea.
14. All people love fame and jobs.
15. They usually come to school late.
16. Everyone, please keep silent!
17. Love either makes somebody sad or happy.
18. Neither Chanthoeun nor Piseth lives in Cambodia.
19. Soleang, Rasy, Bunthoeun, and Savanroth work at the InterED Institute.
20. Preapsovath became a famous singer in 1995.
21. Chanthoeun wrote three books: Parts of Speech, Grammar Tenses, and Writing Skills.
22. His life turned better step by step.
23. I have been waiting for my mother for fifteen minutes.
24. In my life, I like sitting in the coffee shop the most.
25. For many years ago, either Kakada or his brothers were great businessmen in Phnom Penh.
26. Don’t call me at night!
27. During Khmer New Year, everybody was happy in their hometown.
28. Her bedroom smells very nice.
29. He himself killed his dog.
30. They looked very beautiful when they came from the beauty shop.
31. Several students are talkative and intelligent.
32. Where were his sons when we were watching TV?
33. I am wonder whether she came here yesterday?
34. Rika and Bopha, could we please go back home now?

Part 2

1. Present and past


1. At first I didn’t like my job, but______ to enjoy it now.
A I’m starting B I start
2. I don’t understand this sentence. What______?
A does mean this word B does this word mean C means this word
3. Robert______ away two or three times a year.
A is going usually B is usually going C usually goes D goes usually
4. How______ now? Better than before?
A you are feeling B do you feel C are you feeling
5. It was a boring weekend. ______anything.
A I didn’t B I don’t do C I didn’t do
6. Matt______ while we were having dinner.
A phoned B was phoning C has phoned

2. Present perfect and past


7. James is on holiday. He______ to Italy.
A is gone B has gone C has been
8 Everything is going well. There______ any problems so far.
A weren’t B have been C haven’t been
9. Sarah has lost her passport again. This is the second time this______.
A has happened B happens C happened D is happening
10. Why are you out of breath? ______?
A Are you running B Have you run C Have you been running
11. Where’s the book I gave you? What______ with it?
A have you done B have you been doing C are you doing
12. ‘How long______ Jane?’ ‘A long time. Since we were at school.’
A do you know B have you known C have you been knowing
13. Sally has been working here______ .
A for six months B since six months C six months ago D six months
14. It’s two years______ Joe.
A that I don’t see B that I haven’t seen C since I didn’t see D since I last saw
15. It______ raining for a while, but now it’s raining again.
A stopped B has stopped C was stopped
16.10 My mother______ in Italy.
A grew up B has grown up C had grown up
17. ______a lot of sweets when you were a child?
A Have you eaten B Had you eaten C Did you eat
18. Jack______ in New York for ten years. Now he lives in Los Angeles.
A lived B has lived C has been living
19. The people sitting next to me on the plane were nervous. ______ before.
A They haven’t flown B They didn’t fly C They hadn’t flown
D They’d never flown E They weren’t flying
20. Katherine was lying on the sofa. She was tired because______very hard.
A she was working B she’s been working C she’d been working
21. ______a car when you were living in Paris?
A Had you B Were you having C Have you had D Did you have
22. I______ tennis a lot, but I don’t play very much now.
A was playing B was used to play C used to play

3. Future
23. I’m tired. ______to bed now. Goodnight.
A I go B I’m going
24. ______tomorrow, so we can go out somewhere.
A I’m not working B I don’t work C I won’t work
25. That bag looks heavy. ______you with it.
A I’m helping B I help C I’ll help
26. I think the weather______ nice later.
A will be B is C is going to be D shall be
27. ‘Anna is in hospital.’ ‘Yes, I know. ______ her this evening.’
A I visit B I’m going to visit C I’ll visit
28. We’re late. The film______ by the time we get to the cinema.
A will already start B will be already started C will already have started
29. Don’t worry______ late tonight.
A if I’m B when I’m C when I’ll be D if I’ll be

4. Modals
30. The fire spread quickly, but everybody______ from the building.
A was able to escape B managed to escape C could escape
31. I’m so tired I______ for a week.
A can sleep B could sleep C could have slept
32. The story______ be true, but I don’t think it is.
A might B can C could D may
33. Why did you stay at a hotel? You______ with me.
A can stay B could stay C could have stayed
34. I lost one of my gloves. I______ it somewhere.
A must drop B must have dropped C must be dropping D must have been dropping
35. ‘Why wasn’t Amy at the meeting yesterday?’ ‘She______ about it.’
A might not know B may not know C might not have known D may not have known
36. What______ to get a new driving license?
A have I to do B do I have to do C I must do D I have to
37. We have plenty of time. We______ hurry.
A don’t need to B mustn’t C needn’t
38. You missed a great party last night. You______. Why didn’t you?
A must have come B should have come C ought to have come D had to come
39. Jane won the lottery. I suggested______ a car with the money she won.
A that she buy B that she should buy C her to buy D that she bought
40. You’re always at home. You______ out more often.
A should go B had better go C had better to go
41. It’s late. It’s time______ home.
A we go B we must go C we should go D we went E to go
42. ______ a little longer, but I really have to go now.
A I’d stay B I’ll stay C I can stay D I’d have stayed

5. If and wish
43. I’m not tired enough to go to bed. If I______ to bed now, I wouldn’t sleep.
A go B went C had gone D would go
44. If I were rich, ______ a lot.
A I’ll travel B I can travel C I would travel D I travelled
45. I wish I______ have to work tomorrow, but unfortunately I do.
A don’t B didn’t C wouldn’t D won’t
46. The view was wonderful. I would have taken some pictures if ______a camera with me.
A I had B I would have C I would have had D I’d had
47. The weather is horrible. I wish it______ raining.
A would stop B stopped C stops D will stop

6. Passive
48. We______ by a loud noise during the night.
A woke up B are woken up C were woken up D were waking up
49. A new supermarket is going to______ next year.
A build B be built C be building D building
50. There’s somebody walking behind us. I think______ .
A we are following B we are being following C we are followed D we are being followed
51. ‘Where______?’ ‘In Chicago.’
A were you born B are you born C have you been born D did you born
52. There was a fight, but nobody______ .
A was hurt B got hurt C hurt
53. Jane______ to phone me last night, but she didn’t.
A supposed B is supposed C was supposed
54. Where______? Which hairdresser did you go to?
A did you cut your hair B have you cut your hair
C did you have cut your hair D did you have your hair cut
7. Reported speech
55. Paul left the room suddenly. He______ said he to go.
A had B has C have
56. (You meet Joe in the street.) Joe, this is a surprise. Rachel said you______ in hospital.
A are B were C was
57. Anna______ and left.
A said goodbye to me B said me goodbye C told me goodbye

8. Questions and auxiliary verbs


58. ‘What time______?’ ‘At 8.30.’
A starts the film B does start the film C does the film start
59. ‘Do you know where______?’ ‘No, he didn’t say.’
A Tom has gone B has Tom gone C has gone Tom
60. The police officer stopped us and asked us where______.
A were we going B are we going C we are going D we were going
61. ‘Do you think it will rain?’ ‘______ .’
A I hope not B I don’t hope C I don’t hope so
62. ‘You don’t know where Karen is, ______?’ ‘Sorry, I have no idea.’
A don’t you B do you C is she D are you

9. -ing and to ...


63. You can’t stop people______ what they want.
A doing B do C to do D from doing
64. I’d better go now. I promised______ late.
A not being B not to be C to not be D I wouldn’t be
65. Do you want______ with you or do you want to go alone?
A me coming B me to come C that I come D that I will come
66. I know I locked the door. I clearly remember______ it.
A locking B to lock C to have locked
67. She tried to be serious, but she couldn’t help______ .
A laughing B to laugh C that she laughed D laugh
68. Paul lives in Berlin now. He likes______ there.
A living B to live
69. It’s not my favourite job, but I like______ the kitchen as often as possible.
A cleaning B clean C to clean D that I clean
70. I’m tired. I’d rather______ out this evening, if you don’t mind.
A not going B not to go C don’t go D not go
71. I’d rather______ anyone what I said.
A you don’t tell B not you tell C you didn’t tell D you wouldn’t tell
72. Are you looking forward______ on holiday?
A going B to go C to going D that you go
73. When Lisa first came to Britain, she wasn’t used______ on the left.
A driving B to driving C to drive D drive
74. I’m thinking______ a house. Do you think that’s a good idea?
A to buy B of to buy C of buying D about buying
75. I had no trouble______ a place to stay. In fact it was surprisingly easy.
A find B found C to find D finding
76. I called the restaurant______ a table.
A for reserve B to reserve C for reserving D for to reserve
77. James doesn’t speak clearly. .
A It is hard to understand him B He is hard to understand C He is hard to understand him
78. The path was icy, so we walked very carefully. We were afraid______.
A of falling B from falling C to fall D to falling
79. I didn’t hear you______ in. You must have been very quiet.
A come B to come C came
80. ______a hotel, we looked for somewhere to eat.
A Finding B After finding C Having found D We found
10. Articles and nouns
81. It wasn’t your fault. It was______ .
A accident B an accident C some accident
82. Where are you going to put all your______ ?
A furniture B furnitures
83. ‘Where are you going?’ ‘I’m going to buy______ .’
A a bread B some bread C a loaf of bread
84. Sandra is______ . She works at a large hospital.
A nurse B a nurse C the nurse
85. Helen works six days______ week.
A in B for Ca D the
86. There are millions of stars in______ .
A space B a space C the space
87. Every day______ starts at 9 and finishes at 3.
A school B a school C the school
88. ______changed a lot in the last thirty years.
A Life has B The life has C The lives have
89. When______ invented?
A was camera B were cameras C were the cameras D was the camera
90. Have you been to______ ?
A Canada or United States B the Canada or the United States
C Canada or the United States D the Canada or United States
91. On our first day in Moscow, we visited______ .
A Kremlin B a Kremlin C the Kremlin
92. I have some news for you. ______.
A It’s good news B They are good news C It’s a good news
93. It took us quite a long time to get here. It was______ journey.
A three hour B a three-hours C a three-hour
94. This isn’t my book. It’s______ .
A my sister B my sister’s C from my sister D of my sister E of my sister’s

11. Pronouns and determiners


95. What time shall we______ tomorrow?
A meet B meet us C meet ourselves
96. I’m going to a wedding on Saturday. ______is getting married.
A A friend of me B A friend of mine C One my friends
97. They live on a busy road. ______a lot of noise from the traffic.
A It must be B It must have C There must have D There must be
98. He’s lazy. He never does______ work.
A some B any C no
99. ‘What would you like to eat?’ ‘I don’t mind. ______ – whatever you have.’
A Something B Anything C Nothing
100. The course didn’t go well. ______of the students were happy.
A All B No-one C None D Nobody
101. We went shopping and spent______ money.
A a lot of B much C lots of D many
102. I was ill yesterday. I spent______ in bed.
A the most of day B most of day C the most of the day D most of the day
103. I asked two people how to get to the station, but______ of them knew.
A none B either C both D neither
104. Our holiday was a disaster. ______went wrong.
A Everything B All C All things D All of things
105. The bus service is excellent. There’s a bus______ ten minutes.
A each B every C all
106. There were four books on the table. ______a different colour.
A Each of books was B Each of the books was C Each book was

12. Relative clauses


107. I don’t like stories______ have unhappy endings.
A that B they C which D who
108. I didn’t believe them at first, but in fact everything______ was true.
A they said B that they said C what they said
109. We helped some people______ .
A their car had broken down B which car had broken down
C whose car had broken down D that their car had broken down
110. Anna told me about her new job, ______ a lot.
A that she’s enjoying B which she’s enjoying C she’s enjoying D she’s enjoying it
111. Sarah couldn’t meet us, ______ was a shame.
A that B it C what D which
112. George showed me some pictures______ by his father.
A painting B painted C that were painted D they were painted

13. Adjectives and adverbs


113. Jane doesn’t enjoy her job any more. She’s ______ because every day she does exactly the same thing.
A boring B bored
114. Lisa was carrying a ______ bag.
A black small plastic B small and black plastic C small black plastic D plastic small black
115. Maria’s English is excellent. She speaks______.
A perfectly English B English perfectly C perfect English D English perfect
116. He______ to find a job, but he had no luck.
A tried hard B tried hardly C hardly tried
117. I haven’t seen her for______, I’ve forgotten what she looks like.
A so long B so long time C a such long time D such a long time
118. Don’t stand on that chair. It isn’t______ .
A enough strong to stand on B strong enough to stand on it
C strong enough to stand on D strong enough for stand on
119. Sarah is doing OK at the moment. She has______ .
A a quite good job B quite a good job C a pretty good job
120. The exam was quite easy –______ I expected.
A more easy that B more easy than C easier than D easier as
121. The more expensive the hotel, ______.
A the service will be better B will be better the service
C the better the service D better the service will be
122. Patrick is a fast runner. I can’t run as fast as______.
A he B him C he can
123. What’s______ you’ve ever made?
A most important decision B the more important decision
C the decision more important D the most important decision
124. Ben likes walking. ______ .
A Every morning he walks to work B He walks to work every morning
C He walks every morning to work D He every morning walks to work
125. Joe never phones me. ______ .
A Always I have to phone him B I always have to phone him
C I have always to phone him D I have to phone always him
126. Lucy______ . She left last month.
A still doesn’t work here B doesn’t still work here C no more works here
D doesn’t work here anymore E no longer works here
127. ______she can’t drive, she has a car.
A Even B Even when C Even if D Even though

14. Conjunctions and prepositions


128. I couldn’t sleep______ very tired.
A although I was B despite I was C despite of being D in spite of being
129. You should insure your bike______ stolen.
A in case it will be B if it will be C in case it is D if it is
130. The club is for members only. You______ you’re a member.
A can’t go in if B can go in only if C can’t go in unless D can go in unless
131. Yesterday we watched TV all evening______ we didn’t have anything better to do.
A when B as C while D since
132. ‘What’s that noise?’ ‘It sounds______ a baby crying.’
A as B like C as if D as though
133. They are very kind to me. They treat me______ their own son.
A like I’m B as if I’m C as if I was D as if I were
134. I’m going to be in Moscow next week. I hope the weather will be good______ .
A while I’ll be there B while I’m there C during my visit D during I’m there
135. Joe is away at the moment. I don’t know exactly when he’s coming back, but I’m sure he’ll be
back______ Monday.
A by B until

15. Prepositions
136. Bye! I’ll see you______.
A at Friday morning B on Friday morning C in Friday morning D Friday morning
137. I’m going away______ the end of January.
A at B on C in
138. When we were in Italy, we spent a few days______ Venice.
A at B to C in
139. Our apartment is______ the second floor of the building.
A at B on C in D to
140. I saw Steve______ a conference on Saturday.
A at B on C in D to
141. What time did you______ the hotel?
A arrive to B arrive at C arrive in D get to E get in
142. I’m going______ holiday next week. I’ll be away for two weeks.
A at B on C in D for
143. We travelled______ 6.45 train, which arrived at 8.30.
A in the B on the C by the D by
144. ‘Who is this painting______? Picasso?’ ‘I have no idea.’
A of B from C by
145. The accident was my fault, so I had to pay for the damage______ the other car.
A of B for C to D on E at
146. I like them very much. They have always been very nice______ me.
A of B for C to D with
147. I’m not very good______ repairing things.
A at B for C in D about
148. I don’t understand this sentence. Can you______ ?
A explain to me this word B explain me this word C explain this word to me
149. If you’re worried about the problem, you should do something______ it.
A for B about C against D with
150. ‘Who is Tom Hart?’ ‘I have no idea. I’ve never heard______ him.’
A about B from C after D of
151. I don’t know what time we’ll arrive. It depends______ the traffic.
A of B for C from D on
152. I prefer tea______ coffee.
A to B than C against D over

16. Phrasal verbs


153. These shoes are uncomfortable. I’m going to______.
A take off B take them off C take off them
154. They were playing cards, so I______.
A joined in B came in C got in D broke in
155. Nobody believed Paul at first, but he______ to be right.
A worked out B came out C found out D turned out
156. We can’t______ making a decision. We have to decide now.
A put away B put over C put off D put out
157. ‘Have you finished painting the kitchen?’ ‘Nearly, I’ll______tomorrow.’
A finish it up B finish it over C finish it off
158. You can always rely on Paul. He’ll never______.
A put you up B let you down C take you over D see you off
159. Children under 16______ half the population of the city.
A make up B put up C take up D bring up
160. I’m surprised to hear that Kate and Paul have______. They seemed very happy together.
A broken up B ended up C finished up D split up
161. I parked in a no-parking zone, but I______ it.
A came up with B got away with C made off with D got on with

17. Mixed
162. ‘______anything tomorrow evening?’ ‘No, why?’
A Do you do B Are you doing C Will you do
163. ‘I can’t open this bottle.’ ‘Give it to me.______ it.’
A I open B I’ll open C I’m going to open
164. ‘Is Emily here yet?’ ‘Not yet. I’ll let you know as soon as______.’
A she arrives B she’s arriving C she’ll arrive
165. ‘Are you free tomorrow afternoon?’ ‘No,______.’
A I work B I’m working C I’ll work
166. ‘What time is the film tonight?’ ‘______at 8.40.’
A It starts B It’s going to start C It will start
167. ‘Are you going to the beach tomorrow?’ ‘Yes, if the weather______ good.’
A is going to be B will be C is
168. ‘What time______ tomorrow?’ ‘How about 8.30?’
A do we meet B are we meeting C shall we meet
169. ‘When______?’ ‘Tomorrow.’
A does the festival finish B is the festival finished C is the festival finishing
170. ‘What time will you be home tonight?’ ‘I’m not sure. I______ late.’
A may be B might be C can be
171. I can’t find the theatre tickets. They______ out of my pocket.
A must have fallen B should have fallen C had to fall
172. Somebody ran in front of the car as I was driving. Luckily, I______ just in time.
A could stop B could have stopped C managed to stop
173. We have plenty of time. We______ yet.
A mustn’t go B don’t have to go C don’t need to go
174. I didn’t go out yesterday. I______ with my friends, but I didn’t feel like it.
A could go B could have gone C must have gone
175. I looked everywhere for Helen, but I______ her.
A couldn’t find B couldn’t have found C wasn’t able to find
176. ‘What do you think of my theory?’ ‘You______ right, but I’m not sure.’
A could be B must be C might be
177. Our flight was delayed. We______ for two hours.
A must wait B must have waited C had to wait
178. I’m not sure whether I’ll be free on Saturday. I______.
A must have to work B may have to work C might have to work

179. At first they didn’t believe me when I told them what had happened, but in the end I______ them that I
was telling the truth.
A was able to convince B managed to convince C could convince
180. I promised I’d call Amy this evening. I______
A mustn’t forget B needn’t forget C don’t have to forget
181. Why did you leave without me? You ______ for me.
A must have waited B had to wait C should have waited
182. Lisa called me this morning. She suggested______ lunch together.
A we have B we should have C to have
183. That jacket looks good on you. ______ it more often.
A You’d better wear B You should wear C You ought to wear
184. Do you think I should buy a car? What ______ in my position?
A will you do B would you do C should you do
185. I don’t remember ______ about the accident.
A anything B something C nothing
186. Chris and I have known ______ for quite a long time.
A us B each other C ourselves
187. ‘How oft en do the buses run?’ ‘______ twenty minutes.’
A All B Each C Every
188. I shouted for help, but ______ came.
A nobody B no-one C anybody
189. Last night we went out with some friends of ______.
A us B our C ours
190. It didn’t take us a long time to get here. ______traffic.
A It wasn’t much B There wasn’t much C It wasn’t a lot
191. Can I have ______ milk in my coffee, please?
A a little B any C some
192. Sometimes I find it difficult to ______.
A concentrate B concentrate me C concentrate myself
193. There’s ______ on at the cinema that I want to see, so there’s no point in going.
A something B anything C nothing
194. I drink ______ water every day.
A much B a lot of C lots of
195. ______in the city centre are open on Sunday.
A Most of shops B Most of the shops C The most of the shops
196. There were about twenty people in the photo. I didn’t recognise ______of them.
A any B none C either
197. I’ve been waiting ______ for Sarah to phone.
A all morning B the whole morning C all the morning
198. I can’t afford to buy anything in this shop. ______ so expensive.
A All is B Everything is C All are
199. Nobody believed Paul at first but he ______ to be right.
A came out B turned out C worked out D carried out
200. Here’s some good news. It will ______.
A turn you up B put you up C blow you up D cheer you up
201. The children were behaving badly, so I______.
A told them up B told them off C told them out D told them over
202. The club committee is______ of the president, the secretary and seven other members.
A set up B made up C set out D made out
203. Why did you decide not to apply for the job? What______?
A put you off B put you out C turned you off D turned you away
204. I had no idea that he was lying to me. I was completely______.
A taken in B taken down C taken off D taken over
205. Helen started a course at college, but she______ after six months.
A went out B fell out C turned out D dropped out
206. You can’t predict everything. Often things don’t______ as you expect.
A make out B break out C turn out D get out
207. What’s all this noise? What’s______?
A going off B getting off C going on D getting on
208. It’s a very busy airport. There are planes ______or landing every few minutes.
A going up B taking off C getting up D driving off
209. The road was blocked by a bus that had______.
A broken down B dropped out C driven off D held up
210. How are you______ in your new job? Are you enjoying it?
A keeping on B going on C carrying on D getting on
211. I______ tennis every Sunday morning.
A playing B play C am playing D am play
212. Don't make so much noise. Noriko______ to study for her ESL test!
A try B tries C tried D is trying
213. Jun-Sik______ his teeth before breakfast every morning.
A will cleaned B is cleaning C cleans D clean
214. Sorry, she can't come to the phone. She______ a bath!
A is having B having C have D has

215. ______many times every winter in Frankfurt.


A It snows B It snowed C It is snowing D It is snow
216. How many students in your class ______from Korea?
A comes B come C came D are coming
217. Weather report: "It's seven o'clock in Frankfurt and ______"
A there is snow B it`s snowing C it snows D it snowed
218. Babies ______when they are hungry.
A cry B cries C cried D are crying
219.Jane: "What ______in the evenings?" Mary: "Usually I watch TV or read a book."
A you doing B you do C do you do D are you doing
220. Jane: "What______?" Mary: "I'm trying to fix my calculator."
A you doing B you do C do you do D are you doing
221. Jane ______her blue jeans today, but usually she wears a skirt or a dress.
A wears B wearing C wear D is wearing
222. I think I ______a new calculator. This one does not work properly any more.
A needs B needed C need D am needing
223. Sorry, you can't borrow my pencil. I ______ it myself.
A was using B using C use D am using
224. At a school dance: Jane: "______yourself?" Mary: "Yes, I'm having a great time!"
A You enjoying B Enjoy you C Do you enjoy D Are you enjoying
225. I've just finished reading a story called Dangerous Game. It's about a man who ______his wife
because he doesn't want to lose her.
A kills B killed C kill D is killing
226. What time ______
A the train leaves? B leaves the train? C is the train leaving? D does the train leave?
227. Jane: "Are you going to the dance on Friday?"
Mary: "No, I'm not. I ______school dances; they're loud, hot and crowded!"
A not enjoy B don`t enjoy C doesn`t enjoy D am not enjoying
228. I ______ for my pen. Have you seen it?
A will look B looking C look D am looking
229. You can keep my iPod if you like. I ______ it any more.
A don`t use B doesn`t use C didn`t use D am not using
230. The phone ______Can you answer it, please?
A rings B ring C rang D is ringing
231. You'd better go to bed early tonight. The plane ______at 6 o'clock tomorrow morning so we'll have
to be up by 4.30!
A will leave B leaves C leave D is leaving
232. My father ______too much!
A smoking B smokes C smoke D is smoking
233. You should see the new Brad Pitt film. He plays the part of a cowboy who ______up a lawless town.
A is cleaning B cleans C cleaned D clean
234. Woo-Jin and I ______ basketball after school today. Do you want to play too?
A played B play C is playing D are playing
235. Oil ______on water.
A is floating B floats C floating D float
236. Why ______? You should be listening to me!
A you talk B you are talking C do you talk D are you talking
237. What ______next weekend, Yoshi?
A you doing B you do C do you do D are you doing
238. Jane: Do you play the piano? Mary: No, I ______any musical instrument!
A isn`t playing B don`t play C doesn`t play D am not playing
239. Money ______on trees!
A isn`t growing B don`t grow C doesn`t grow D didn`t grow
240. You ______into things and breaking them. How can anyone be so clumsy?
A are always bumping B always bumps C always bumping D always bump
241. "What ______ about the plan to shorten the school holidays?"
A you think B will you think C do you think D are you thinking
242. Jane: "What ______about?" Mary: "My summer holidays! I wish I was still lying on the beach."
A you thinking B does he think C do you think D are you thinking
243. My father ______me to school this week because we had an accident and our car is being repaired.
A isn`t driving B hadn`t driven C don`t drive D doesn`t drive
244. A new hotel ______in the centre of town.
A is built B is building C is being built D built
245. I ______to bed very late last night.
A went B was going C have gone D go
246. I ______tennis yesterday.
A will play B played C play D have played
247. My brother is an author. He ______many books.
A wrote B write C is writing D has written
248. My brother, who died last year, was an author. He ______many books.
A wrote B writes C was writing D has written
249. I ______to New York last Christmas.
A have flown B fly C flew D am flowing

250. My grandparents live in the USA so I ______to New York many times.
A have flown B flies C flew D am flying
251. I ______tennis three times already this week and it's still only Wednesday.
A was playing B played C play D have played
252. Student to the teacher: " ______! What shall I do now?"
A I`ve finished B I finished C I am finishing D I am finish
253. I ______to bed very late last night.
A went B was going C have gone D go
254. This tree ______really fast. Last year it was only half this size.
A will grow B has grown C grows D grew
255. I ______to the theater many times when I lived in London.
A went B was going C have went D have gone
256. Where's my calculator? I put it on the desk a minute ago and now ______!
A it`s disappearing B it`s disappeared C it disappears D it disappeared
257. My father ______in a bank for 10 years, then he became a computer programmer.
A works B worked C was working D has worked
258. My father ______at his bank for 10 years. He really likes his job.
A worked B was working C is working D has worked
259. My grandfather died 10 years before I was born, so I ______him.
A was not meeting B meet not C have not met D did not meet
260. I'm very hungry. It's 11.30 and I ______anything yet today.
A haven`t eaten B don`t eat C didn`t eat D am not eating
261. ______the football match on TV last night?
A Were you seeing B Have you seen C Do you see D Did you see
262. Last week was very wet, but it ______once yet this week.
A isn`t raining B haven`t rained C hasn`t rained D didn`t rain
263. ______a new computer. Do you want to see it?
A I`ve bought B I was buying C I buy D I bought
264. ______my dictionary. Have you seen it?
A I`ve lost B I`m lost C I lost D I lose
265. I bought a new computer last week, but it ______so I took it back to the shop.
A worked not B hasn`t worked C don`t work D didn`t work
266. I tried to find you before school today. ______
A Where were you? B Where was you? C Where have you been? D Where did you be?
267. My math teacher wasn't happy yesterday. I ______to bring my calculator to class.
A was forgetting B have forgotten C forgot D forget

268. Have you seen my pencil? It was here a minute ago and now ______.
A it`s gone B it went C it was going D it have gone
269. I ______to the cinema since I came to Germany.
A went not B haven`t gone C hasn`t gone D didn`t go
270. I like your watch. How long ______it?
A you have B have you had C did you have D are you having
271. Have you ever ridden a horse? - No, but I ______a camel when I visited Egypt last year.
A was riding B rode C ride D have ridden
272. My big sister ______a baby. She's going to call it Yoshi.
A was having B has had C has D had
273. I went to the school dance last Friday, but I ______it very much
A haven`t enjoyed B don`t enjoy C didn`t enjoyed D didn`t enjoy
274. I ______very well last night. There was loud music coming from a house down the road.
A wasn`t sleeping B haven`t slept C didn`t slept D didn`t sleep
275. I ______very well since I bought a new bed. I don't find it very comfortable.
A was sleeping B haven`t slept C haven`t sleep D didn`t sleep
276. I ______my foot playing tennis last week, and now I can hardly walk.
A was injuring B injured C injure D have injured
277. ______my hand on a piece of glass. Do you have a Band-Aid?
A I`ve cut B I was cutting C I cutted D I cut
278. ______my calculator? I can't find it anywhere?
A You seen B Have you seen C Did you seen D Did you see
279. I was in a big hurry so I ______no time to phone you.
A was having B have had C have D had
280. My grandmother ______for 62 years before dying at the age of 89!
A was smoking B smoked C have smoked D has smoked
281. My grandmother ______for 62 years and she never even coughs. What a lucky woman!
A smokes B smoked C is smoking D has smoked
282. I ______late for school again this morning. The teacher was not happy!
A have arrived B arrived C arrive D am arriving
283. "Where's Emiko?" - "______to the library!"
A She`s gone B She went C She is going D She goes
284. I ______anything since breakfast so I feel very thirsty now.
A wasn`t drinking B haven`t drunk C don`t drink D didn`t drink
285. The plane ______at New York three hours late.
A has arrived B was arriving C did arrive D arrived

286. I can't go home until I ______this job.


A have finished B was finishing C had finished D finished
287. I ______ in London since I was a little child.
A lived B was living C have lived D did live
288. As soon as I saw the man, I realised that we ______ before, in Caracas.
A met B were meeting C have met D had met
289. After leaving London, we ______ on to Birmingham without stopping.
A drove B were driving C have driven D had driven
290. She ran away with her lover, while her husband ______in Australia.
A worked B was working C has worked D had worked
291. Where can he be? I can only imagine that he ______ an accident somewhere.
A had B was having C has had D did have
292. He had worked in the company for 15 years before he ______ promoted
A got B was getting C has got D had got
293. I ______the office after everyone else.
A left B was leaving C have left D had left
294. Hi, I'm really pleased to see you again, but I'm afraid I ______ your name
A forgot B was forgetting C have forgotten D had forgotten
295. The president expects ______ for a second term in office.
A be re-elected B to be re-elected C being re-elected
296. We've both decided ______smoking.
A give up B to give up C giving up
297. The job I had involved ______ tombola tickets in the street.
A sell B to sell C selling D to selling
298. I really think that you should stop ______ about everything.
A to complain B complain C complaining
299. I want ______ to bed early this evening.
A go B to go C going
300. He always remembered ______ a big bottle of water if he went walking in the mountains.
A to take B take C taking
301. I clearly remember ______ a red car outside his house.
A to see B see C seeing D to seeing
302. They never mentioned ______him yesterday.
A see B to see C seeing D to seeing
303. The Beatles got ______for the Queen of England.
A to play B playing C play

304. When we were half-way to Scotland, we stopped ______ a meal in a motorway service station
A have B to have C having
305. They thought they could ______ the painting for a lot of money
A sell B to sell C selling
306. When we cross the river, don't let ______ of the dog.
A go B to go C going D to going
307. The engine finally got ______ at the sixth attempt.
A go B to go C going
308. My brother really enjoys ______ in Tahiti.
A to work B work C working
309. No-one was allowed ______the room.
A leave B to leave C leaving D to leaving
310. If you break his camera, it means ______ him a new one.
A to buy B buy C buying
311. He broke my camera and means ______ me a new one.
A to buy B buy C buying
312. You'll have to make ______ with what's left.
A to do B do C to doing D doing
313. If I study hard, I'll learn ______ English perfectly.
A speak B to speak C speaking D to speaking
314. She really regretted ______told him to go home.
A have B to have C having

សំ ណួរត្រៀម្រឡងតរសតភាសាអង់តលេក្មរ
ិ ៣
Writing Test

Why Are Social Networking Sites So Popular?


1- Social networking sites become more and more popular every day, and they are popular all around the world.
In Japan, the top site is Mixi. In Europe, it is Bebo. The most popular site in Latin America is Orkut. In the
United States, the top site is Facebook. In fact, Facebook is one of the most popular social networking sites in
the world. A Harvard University student started Facebook in 2004, and it spread to more than 400 million users
in just a few years.
2- Why is the social networking trend spreading so rapidly? One reason that these websites are popular is
because people are social. We like to communicate with other people. We make friends with people in school,
at work, and online. Most people like to stay closely connected to their friends and family. We use cell phones,
email, instant messaging, and websites to learn what our friends are doing. The Internet is a good way to
socialize and communicate, and social networking sites allow people to do this in many ways.
3- Social networking sites are interactive and personal. People can share photographs of themselves and of
others. They can tell people what they are doing at any moment and keep in touch. They can post a link to a site
with their favorite song or band. They can join groups with others who share their interests. Many people post
videos of themselves on sites like YouTube. Other users can comment on these photos and videos. This
interaction makes these websites become more popular.
Choose the best response.
1. Which of the following is the main idea of paragraph 2?
A. People stay connected to their friends and family in many ways.
B. Social networking sites are popular because people are social.
C. The Internet is a good way to socialize and communicate.
2. Which of the following is the main idea of paragraph 3?
A. On social networking sites, people can join groups with others who share their interests.
B. Many people post videos of themselves on sites like YouTube, and others can comment on these
videos.
C. Social networking sites are very popular because they are interactive and personal.
3. Which of the sentences is NOT a main idea in this reading passage?
A. A Harvard University student started Facebook in 2004, and it spread to more than 400 million
users in just a few years.
B. Every day more and more people join social networking sites all around the world.
C. Because social networking sites are interactive and personal, their popularity has increased.
Should I Kiss, Bow, or Shake Hands?
1- Most people want to be polite and behave well around others. Being polite means knowing how to greet and
talk to people. It means using good manners when eating. It means knowing how to give and receive gifts
appropriately. Polite behavior in one country, however, may be impolite in another part of the world. Travelers
need to understand the cultural differences in politeness so that they don’t cause embarrassment.
2- For instance, when people meet, they often shake hands. How long should a handshake be? Should you hold
the other person’s hand gently or firmly? In the United States, people prefer to shake hands firmly for a few
seconds. In some Middle Eastern countries, people hold the person’s hand gently for a longer time.
Handshaking varies around the world.
3- What about eye contact? In some countries you show respect when you look someone directly in the eye. In
other parts of the world, to look at someone directly is rude. To be respectful, a person looks down at the
ground.
4- There are also cultural differences in the way people use personal space. When two people are talking, should
they stand close together or far apart? Exactly how close should they stand? In North America, for instance,
people usually stand about an arm’s length apart during a conversation. However, in some countries in the
Middle East and Latin America, people stand closer. It can be awkward if one person likes to stand close and
the other person likes to stand farther apart.
5- Another cultural difference is time. If someone invites you to dinner at their house at 6 p.m., what time
should you get there? Should you arrive early, late, or exactly on time? In Germany, it is important to arrive on
time. In Argentina, polite dinner guests usually come 30 to 60 minutes after the time of the invitation. When
traveling, remember that each country has a different definition of time.
Choose the best response.
4. Which of the following is NOT an example of a supporting detail in paragraph 1?
A. Most people want to be polite and behave well around others.
B. Being polite means knowing how to greet and talk to people.
C. It means knowing how to give and receive gifts appropriately.
5. Which of the following details about the Middle East is NOT found in the text?
A. People in the Middle East stand closer than Americans during a conversation.
B. When shaking hands, people in the Middle East hold a person’s hand gently for a longer time.
C. In the Middle East, it is important to bring a gift to someone’s house.
6. Which of the following is the main idea of paragraph 2?
A. When people meet, they often shake hands.
B. How long should a handshake be?
C. Handshaking varies around the world.
7. Which of the following is NOT a supporting detail in paragraph 2?
A. In some Middle Eastern countries, people hold the person’s hand gently for a longer time.
B. Should you hold the other person’s hand gently or firmly?
C. In the United States, people prefer to shake hands firmly for a few seconds.

The Richest and the Best


1- Manchester United Football Club is the richest sports team in the world. The English club is worth more than
$1.8 billion. The Spanish soccer team Real Madrid is worth $1.3 billion. These clubs have enough money to pay
the high salaries of the best players in the world. For example, in 2008, Manchester United paid Cristiano
Ronaldo more than $18 million. They paid Wayne Rooney more than $16 million. Other teams also spend a lot
of money on their players. These wealthy clubs include Barcelona in Spain, AC Milan in Italy, and Chelsea in
England. As a result, these teams have been very successful. For example, Manchester United won
championships in 2006, 2008, and 2009. Money is a clear advantage for these professional teams.
2- The wealthiest teams make money in several ways. First, they can sell tickets at high prices because fans
want to see the top players in action. Second, television stations pay the teams to broadcast their games. Third,
large companies give financial support to the best teams. These corporate sponsors pay money so that their
company name can be on the clubs’ shirts, shoes, and stadiums.

3- The recent success of the English team Chelsea shows an example of the effect of money on a team. For
many years, Chelsea was a club with little money and few wins. Everything changed in 2003, when Russian
billionaire Roman Abramovich bought the team for $233 million. In his first year, Abramovich spent more than
$350 million on players. By 2006, Chelsea had won two championships. Now Chelsea is one of the richest
clubs in the world.

4- Many sports fans feel that money in sports creates an unfair advantage. Some teams have so much money
that it seems impossible for poorer clubs to beat them. Wealthy teams are usually more successful, so they sell
more tickets and make more money. Teams that do not have as much money usually are not as successful. It is
difficult for them to get money from tickets, television, and corporate sponsors. Sports fans know that money
gives some clubs a great advantage.
Choose the best response.
8. Which of the following details would NOT be listed in your notes about the text?
A. In 2008, Manchester United paid Wayne Rooney more than $16 million.
B. The Spanish soccer team Real Madrid is worth $1.3 billion.
C. Cristiano Ronaldo is the richest player on the Real Madrid team.
9. When taking notes about the text, what main idea should be listed for paragraph 2?
A. The best teams can sell their tickets at high prices.
B. There are several ways the richest teams can earn money.
C. Companies pay teams to have their name on the teams’ shirts.
10. What supporting idea would be in your notes about paragraph 3?
A. TV stations paid money to broadcast the games.
B. A Russian billionaire bought the team for $233 million.
C. Wayne Rooney was paid more than $16 million.
11. What is an important way to use notes that you have taken?
A. to study for a test
B. to think of ideas
C. to write to friends

A Disposable Society
1- In our modern world, when something wears out, we throw it away and buy a replacement. If a shirt is torn
or a coffee machine breaks, you throw it away. The problem is that countries around the world have growing
mountains of trash because people are throwing out more trash than ever before. For example, in the United
States, the amount of trash per person nearly doubled from 1960 to 2000.

2- How did we become a throwaway society? First of all, it is now easier to replace an item than to spend time
and money to repair it. Thanks to modern manufacturing and technology, companies are able to produce items
quickly and inexpensively. Products are plentiful and prices are low, so we would rather buy something new
than repair it. Even if we did want to repair something, many items—from toasters to TVs—are almost
impossible to repair. These products contain many tiny, complicated parts. Some even contain small computer
chips. It’s easier to throw these items away and buy new ones than to fix them.

3- Another contributing factor is our love of disposable products. As busy people, we are always looking for
ways to save time and make our lives easier. Why should we use cloth kitchen towels? It is easier to use a paper
towel once and toss it out. Companies manufacture thousands of different kinds of disposable items: paper
plates, plastic cups, cameras, and razors for shaving, to name a few. Because these products aren’t designed to
last, companies know that consumers will have to replace them, buying them over and over again. “What’s
wrong with that?” you ask. The problem is that disposable products are contributing to our trash problem.
Choose the best response.
12. Which of the following statements from the reading is a fact?
A. We would rather buy something new than repair it.
B. It is easier to use a paper towel once and toss it out.
C. Companies manufacture thousands of different disposable items.
13. Which of the following statements from the text is an opinion?
A. In the United States, the amount of trash per person nearly doubled from 1960 to 2000.
B. It’s easier to throw items away and buy new ones than to fix them.
C. Some toasters and TVs contain small computer chips.
14. Which of the following statements is a fact?
A. He’s the best president we’ve ever had in this country.
B. It’s impressive that he’s one of the youngest presidents of the United States.
C. He was sworn in as president on January 20, 2009.
15. Which of the following statements is an opinion?
A. It’s cold in here.
B. I know when the library opens.
C. She is a great dancer.
16. Which of the following statements is a fact?
A. The last day we had any significant rainfall was on July 1, 2010 when there was an afternoon
shower.
B. Today is the most miserable day of the summer.
C. There are billions of people in the world, and most of them love music.

The Ingredients of a Good Story


1- American playwright Elmer Rice said that a play or story in its simplest form has three parts. He said that the
writer needs to:
1. Put a man up a tree.
2. Throw stones at him.
3. Get him down.
This may seem surprisingly simple, but his clever idea works well for many kinds of stories. Let’s look at how
you can use this recipe for writing your own short story.

2- In part 1, Put a man up a tree, we have two important ingredients of a story: the main character and the
setting. Your main character is the person at the center of your story. Who is this person? What does your
character look like? What is his or her personality? Your setting is where your story takes place. This is the
physical location of the story. In some stories, the setting is very important, while in other stories, it isn’t. In the
example, the setting is a tree, and this is essential to the story. The setting can also include the time of the story,
or whether it takes place in the past, present, or future. The time span is also important. For a short story, the
time span is usually short.

3- Part 2 of the story, Throw stones at him, is the conflict. The conflict is the problem facing the main character.
In the example, the character’s problem is that he is in a tree, and he wants to come down. There is a conflict
between the character and the person who is throwing stones at him. The conflict in a story is the basic problem
that your character must resolve. What is the problem facing your character? Why does it matter to the main
character?

4- In part 3, Get him down, the main character solves the problem or overcomes the challenge at the end of the
story. He comes down from the tree. This is the resolution of the story. In a good story, the resolution isn’t
always easy, and the main character may resolve the problem in an unexpected way.
Choose the best response.
17. Which of the following is NOT a tip for summarizing?
A. Take notes about the main ideas and important details in a text.
B. Don’t go back to reread the text a second time.
C. Put your ideas together into sentences or paragraphs.
18. Look at paragraph 2. Which is the best summary?
A. Paragraph 2 talks about the introduction of a story. It introduces two of the most important parts
of a story, the main character and the setting, and explains what each of them is.
B. The main character and the setting are two of the most important parts of a story. The main
character is the person at the center of the story. The setting is where the story takes place. It is
important to think carefully about both of these parts of the story and ask yourself a lot of
questions.
C. It’s really important to write as much as possible about the main character of the story. Also the
setting is important. Make sure your story has this information in it, and give lots of details.
19. Look at paragraph 3. Which of the following is NOT an important idea for a summary of this paragraph?
A. It is important to make the person stop throwing stones at the main character.
B. The conflict in a story is the basic problem that your character must resolve.
C. As a writer, you must show why the problem is important to the main character.
20. Look at paragraph 4. Which is the best summary?
A. The third part of a story is the end, when the character solves the problem.
B. In the third part of a story, the main character resolves the problem that he faced in the middle of
the story, and the story ends. In a good story, the resolution is difficult and different from what
you would expect.
C. In the third part of a story, the character comes down from the tree because the person has finally
stopped throwing stones at him.

Read the paragraph. Then choose the best definition for each word using the context.
United Parcel Service (UPS) is a big delivery company. Its company color is brown. When UPS started in the
1920s, brown was a good color for a safe, reliable company. From the beginning, UPS used brown trucks and
brown uniforms. In today’s world, brown may seem like a boring color choice for a company. But UPS decided
to make it a positive symbol of its business. Their ads ask, “What can brown do for you?” When people see the
big brown UPS trucks, the company hopes they will think of excellent, dependable service.
21. reliable
A. dependable
B. mature
C. secure
22. boring
A. stable
B. uninteresting
C. bad
23. symbol
A. sign
B. gesture
C. idea

Read the paragraph. Choose the sentence that shows the main idea.
24. Social networking sites first became popular with college students. At one college, students said that
they spent almost two hours every day just on Facebook. Teenagers also use these sites to stay
connected with their friends. These days, even older people are using social networks. The Internet
keeps changing, but one trend is clear: People enjoy using websites that let them connect with others.
A. These days, even older people are using social networks.
B. People enjoy using websites that let them connect with others.
C. Social networking sites first became popular with college students.

Read the main idea. Then choose the best supporting sentence for the main idea.
25. In many cultures, it is rude to touch people you do not know very well.
A. For example, in Thailand, it is rude to touch a person’s head with the palm of your hand.
B. In many cultures, it is also rude to stand too close to somebody when talking.
C. Touching people you know is usually OK.
26. Each culture has specific rules for gift-giving.
A. In every culture, gift-giving is an important way to show appreciation and love for others.
B. In China, you should open a gift after the person is gone.
C. Rules for gift-giving vary from culture to culture.

Choose the best response.


27. Which of the sentences below shows a very strong positive statement of opinion?
A. Officials must allow girls to play the same sports as boys.
B. Officials should allow girls to play the same sports as boys.
C. Officials ought to allow girls to play the same sports as boys.
28. Which of the sentences below shows a weak negative statement of opinion?
A. Professional athletes must not be allowed to take drugs.
B. Owners must not put a limit on the salaries of professional soccer players.
C. Soccer coaches should not earn more money than soccer players.
29. Which phrase is NOT used to introduce an opinion in an opinion paragraph?
A. I don’t think
B. I believe
C. I know
30. Which sentence would be a strong concluding sentence in a paragraph about college sports players?
A. In my opinion, college sports players should have good grades to play sports.
B. For these reasons, I feel college sports players should focus more on their grades than on their
sports.
C. Second, they are representing the college or university when they play.

Complete the sentence. Choose A, B, or C.


31. I don’t have a good Internet __________. Sometimes I can’t get online.
A. connect
B. conects
C. connection
32. Hong __________ movies very much. Every weekend, he goes to the theater.
A. enjoys
B. enjoyment
C. enjoyed
33. This __________ about traveling to Spain is very useful.
A. information
B. inform
C. informing
34. We have a __________ of drinks to choose from. Which would you like?
A. mass
B. culture
C. variety
35. They plan to __________ a new library system next year.
A. argue
B. establish
C. encourage
36. I don’t buy coffee at that shop because of the bad __________. The coffee is always cold by the time I
get it.
A. service
B. environment
C. smell
37. The mayor doesn’t support the construction of a new mall because it will hurt the __________.
A. building
B. emotion
C. environment
38. Her parents __________ her to study more to improve her grades.
A. encourage
B. represent
C. request
39. You should __________ talking to your boss if you’re not satisfied at work.
A. affect
B. consider
C. prefer

Read the sentence. Is the underlined word a noun or a verb? Choose A, B, or C.


40. I am not good at math, so I have to study very hard.
A. noun
B. adjective
C. verb
41. I like watching previews of new movies when I go to the movie theater.
A. noun
B. verb
C. adjective
42. Jasmine gave her friend a gift for his birthday.
A. noun
B. verb
C. adjective
43. She uses recycled paper in her artwork.
A. adjective
B. verb
C. noun
44. The threatened animals left the forest.
A. adjective
B. verb
C. noun
45. The city lowered the amount of electricity it used.
A. adjective
B. verb
C. noun
46. The old factory released a lot of smoke.
A. adjective
B. noun
C. verb
47. Reducing waste was an expected benefit of the recycling program.
A. adjective
B. verb
C. noun
48. Indoor farmers often use potted plants to grow vegetables.
A. adjective
B. verb
A. noun

Read the paragraph. Choose the best topic sentence for the paragraph.
49. More and more things are being done by computer these days. Job applications, banking, and travel
reservations can all be done by computer. Most jobs require a person to know how to use a computer.
Also, people are using computers more and more to socialize and connect with other people. Email,
instant messaging, and social networking sites like Facebook are all examples of how people are using
computers to connect.
A. It is very important to learn how to use a computer.
B. You can download and play music and videos on the computer.
C. Most people know how to use a computer today.

Read the paragraph. Then choose the best response.


FREEWRITING TOPIC: What is your favorite color? Why?
Purple is my favorite color. I think it is an interesting color. You don’t see it every day. When I was a child I
had my favorite purple shirt. I wore it almost every day! I also had a dog when I was a child. When I see purple,
it makes me think of mysterious and exciting things.
50. Which sentence does NOT focus on the topic?
A. You don't see it every day.
B. I also had a dog when I was a child.
C. It makes me think of mysterious and exciting things.
51. Which of the following sentences can be added to the paragraph?
A. My favorite flower, the iris, is purple, too.
B. I think green is a very pretty color, too.
C. One mysterious thing I think of is dreams.

Choose the best response.


52. Which is the best topic sentence?
A. My first teacher made a very good impression on me.
B. My first teacher always came to class well prepared.
C. My first teacher greeted the students kindly.
53. Which is the best topic sentence?
A. A good doctor tries to understand how patients feel.
B. A good doctor has two important qualities.
C. A good doctor is caring toward patients.
54. Which is the best supporting sentence?
A. Read the job descriptions very carefully.
B. Here are five tips for searching job websites.
C. Finding the right job online is not easy.
55. Which is the best concluding sentence?
A. First of all, you need to listen to them so they know you care.
B. Do not interrupt them when they are speaking and keep steady eye contact.
C. When you follow these steps, it is easier to deal with difficult customers.
56. Which of the following makes a better topic sentence for a paragraph about how we choose our music?
A. In a study, researchers gave a list of 48 unknown songs to 14,000 teenagers.
B. Researchers say that hit songs become popular because listeners care about what other people think.
C. Some people look at the list of favorite songs posted by someone they know.

Read the paragraph. Then answer the question with the sentence that synthesizes information from the
paragraph and the passage.
H1N1 Flu
Laboratory studies have shown that no children and very few adults younger than 60 already have immunity to
the 2009 H1N1 flu virus. However, about one-third of adults over 60 may have immunity to the virus.
57. Which of the following is NOT a reasonable conclusion about the flu virus?
A. Both the seasonal flu and the H1N1 flu are dangerous for the same groups of people.
B. Children of any age can get both the H1N1 flu and the seasonal flu quite easily.
C. Adults younger than 60 are more likely to get the H1N1 flu than adults over 60.
58. Which of the following can NOT be concluded about the H1N1 virus?
A. The H1N1 flu is quite different from the normal seasonal flu.
B. Adults over 60 are more likely to get the seasonal flu than the H1N1 flu.
C. The H1N1 flu has existed almost as long as the seasonal flu.

The sentences below form a paragraph. Match the sentences with their order in the paragraph.
_______ 59. I don’t like the idea of an electronic reading device.
A. second
_______ 60. Also, when reading an e-book, I always worry about the battery running
sentence
out.
B. first sentence
_______ 61. Finally, I think a house doesn’t feel like a home without a collection of
C. last sentence
real books.
D. third sentence
_______ 62. In particular, I don’t like the way an e-book feels in my hands.

The sentences below form a paragraph with supporting examples. Match the sentences with their order
in the paragraph.
_____ 63. One kiss is nothing for the French; when two French
people meet, they usually give four kisses, two on each
cheek.
_____ 64. For example, in Mexico, when two people meet, they
A. first sentence
typically give each other one kiss on the left cheek.
B. second sentence
_____ 65. Unlike in France and Mexico, Americans don’t kiss each
C. third sentence
other on the cheek when they meet; they prefer to shake
D. fourth sentence
hands or hug if they know each other well.
_____ 66. In many cultures, people kiss each other on the cheek
when they meet as a form of greeting.

Read the sentences for a narrative paragraph. Then number the sentences to show the correct order.
_______ 67. The princess didn’t believe him, but she kissed him anyway. As soon as she did, he turned into a
handsome prince.
_______ 68. One day, she found a frog in her bedroom. The frog asked her to kiss him. He said he would
become a handsome prince.
_______ 69. Finally, the prince and the princess got married, and they lived happily ever after.
_______ 69. Many, many years ago, there was a princess who wanted to meet a handsome prince.

Read the steps for writing a defining paragraph. Then number the steps in the correct order.
_______ 70. Make sure the definition is clear.
_______ 71. Explain how the term or concept is different from similar terms.
_______ 72. Write a topic sentence that states the term or concept and defines it.
_______ 73. Write about the term or concept using explanations and examples.
_______ 74. Explain what the concept is not.

Read the sentences for an opinion paragraph. Then number the sentences to show the correct order.
_______ 75. Cheap tennis rackets only cost around thirty dollars.
_______ 76. Tennis is a good sport to play.
_______ 77. First, it does not cost much money.
_______ 78. Tennis players strengthen their legs when running.
_______ 79. Second, it is good for your body.
_______ 80. Running also helps exercise the lungs.

Read the sentence. Is it a fact or an opinion? Choose A or B.


81. It is difficult to get away from exposure to advertisements.
A. fact
B. opinion
82. Many European countries have tried to regulate advertising.
A. fact
B. opinion
83. The Swedish government banned advertising directed at children under twelve.
A. fact
B. opinion
84. Advertisers in Canada are not allowed to exaggerate the prices of items.
A. fact
B. opinion
85. Canada’s self-regulating advertising standards are simple but beneficial.
A. fact
B. opinion

Choose the correct way to say each number or mathematical term.


86. 703-506-4993
A. seven zero three, five zero six, forty-nine nine three
B. seventy three, fifty six, forty nine ninety three
C. seven oh three, five oh six, four nine nine three
87. 12 + 5 = 17
A. twelve plus five equals seventeen
B. twelve times five is seventeen
C. twelve divided by five is seventeen
88. 230,000
A. twenty three thousand
B. two hundred thirty thousand
C. twenty three million

Complete the sentence with the correct conjunction.


but so and or

89. He is sick, ____________________he is staying home today.

90. Bhim has one brother ____________________ one sister.


91. Do you like dogs ____________________ cats more?

92. I called her, ____________________ she did not answer the phone.

Read the compound sentence. Choose the correct reason for using the underlined coordinating
conjunction.
93. The movie was terrible, so we decided to leave early.
A. related ideas
B. contrasting ideas
C. a result
D. a choice
94. I like reading the stories in the magazine, but it has too many ads.
A. related ideas
B. contrasting ideas
C. a result
D. a choice
95. Advertisers can make TV ads, and they can advertise on the Internet.
A. related ideas
B. contrasting ideas
C. a result
D. a choice

Read the statement. Write S (complete sentence) or F (sentence fragment).


_______96. Because there are so many people using social networking sites.
_______97. Although I didn’t like text messaging at first, now I love it.
_______98. When I send a text message to my teenage son, I expect a quick response.
_______99. I use abbreviations like IOU because they are easier and faster.
_______100. When dictionaries began including words like cos and wot.
_______101. Since the Internet has allowed people to come into contact with more and more people.
_______102. After the scientists studied the effects of texting on literacy.

Complete the letter to the editor with the sentences below. Write the letter next to the correct sentence.
Dear Editor,
My eight-year-old daughter loves to watch her favorite shows on the weekend. I don’t mind her watching a little
TV to relax, and I often sit with her so we can enjoy the shows together. However, I am getting tired of having
to wait through all the ads to see what happens next. Recently, I watched a show with my daughter, and there
were more than ten ads! (a) _____.
(b) _____. For example, they do not know that ads make them want something they don’t need. Just yesterday,
my daughter asked to go to the store to get her a game that was advertised during her favorite show. She got
upset when I tried to explain that she didn’t really need the game.
(c) _____. The ads are usually much louder than the shows children are watching. (d) _____. Imagine the
damage a lot of ads could do to children’s hearing!
My daughter is a smart girl, but she is not able to understand the real power behind advertising. And I worry
about loud ads damaging her hearing. (e) _____. That is why I think there should be no more ads played during
children shows.
Sincerely,
A Concerned Parent
_______ 103. Every time an ad comes on, I have to turn down the volume so it doesn’t hurt my own ears.
_______ 104. I do not think television stations should play ads during kids’ shows.
_______ 105. My other concern is that today’s ads may be dangerous for children’s ears.
_______ 106. Advertising can confuse and harm children.
_______ 107. First of all, children aren’t old enough to understand how ads work.

Complete the problem-solution essay with the sentences below. Write the letter next to the correct
sentence.
The 3 Rs
There’s simply too much waste on our planet, and almost all of it is generated by human beings. We need to
decrease the amount of trash we produce. (a) _____.
(b) _____. People throw away millions of plastic water bottles each year, for example. But many of us could
buy just one metal or hard plastic bottle and use it again and again. It is an easy way to reduce the number of
plastic bottles that end up in the trash.
(c) _____. Instead of throwing away old plastic shopping bags, we can find alternative uses for them. For
example, we can reuse them as garbage bags in small wastebaskets or while in our cars. Not only are they
effective, but they are also free.
(d) _____. Many cities have recycling programs for glass, plastic, and paper items. This old “trash” can be
turned into new products with little effort. People can also recycle electronic devices, such as cell phones and
computers. They can be fixed and resold, and they benefit other people and the planet.
(e) _____. Reducing, reusing and recycling waste are simple ideas that can have enormous effects. If everyone
worked together to follow these rules, the world would be a much better place to live.
_______ 108. People can also decrease waste by reusing items they already have.
_______ 109. One of the easiest ways to help our planet is by reducing the amount of trash we produce.
_______ 110. In all, when people stick to the three Rs, they produce less waste and help keep our planet a
cleaner and healthier place to live.
_______ 111. We can cut down the amount of waste we make if we stick to three basic rules: reduce, reuse,
and recycle.
_______ 112. In addition, recycling can greatly cut down on waste.

Complete the cause/effect essay with the sentences below. Write the letter next to the correct sentence.
Random Acts of Kindness
We are usually kind to the people we love. Many people are reluctant to help strangers, however. We should all
try to be more kind to people in need. People should try to do one random act of kindness for a stranger a day.
(a) _____.
(b) _____. It can be something as simple as holding the door open for a stranger. It can be as thoughtful as
telling someone they dropped money on the floor. When I go grocery shopping, I try to find ways to help other
shoppers. (c) _____. When I leave the store, I offer to put back another person’s cart. These are simple things to
do, but they help others. People are usually thankful, because they never expected the help.
(d) _____. Tal Ben-Shahar, a psychology teacher at Harvard University, found that giving help makes people
feel good about themselves. He asked his students to do five small acts of kindness a day, such as giving change
to a homeless person or being nice to waiters at a restaurant. When they did these acts, he found that his
students were happier as a result.
(e) _____. It helps the giver and the receiver both. I have already promised myself to do one act of kindness
each day, and I hope others will, too. It doesn’t take much effort. Every time you go out, just look for little
things you can do for other people, and you will very happy with the results.
_______ 113. Then they can help both other people and themselves.
_______ 114. When someone can’t reach an item, I offer to get it for them.
_______ 115. A random act of kindness can also benefit the person giving help.
_______ 116. A random act of kindness is a small task that helps others.
_______ 117. In all, everyone benefits from a random act of kindness.

Complete the opinion essay with the sentences below. Write the letter next to the correct sentence.
Don’t Make Me Text
It seems like everyone today loves to text. In fact, many teenagers prefer it to a conventional phone call. You
can hear and see people texting all around you—in stores, at work, and even in their cars. (a) _____. I disagree
with them. I much prefer calling my family and friends, because it safer, cheaper, and more personal.
(b) _____. When I need to talk to someone while I’m driving, I always talk over the phone. That way, I can see
what is happening around me. When people text, they have to look down at their phones for a long time, which
takes their eyes off the road. This has caused many accidents. With a traditional phone call, you can avoid these
kinds of accidents.
(c) _____. If people don’t text, they don’t really need a cell phone. They can just use a home phone and only
have one phone bill. People who use their cell phones to text message often have to pay more money for the
service. If they go over their limit, they can be charged a lot of money for the extra texts.
(d) _____. I love getting phone calls from my family because I like hearing their voices. I always recognize my
mother or best friend just by hearing them say “hi” when I answer the phone. That personal touch gets lost
when people text. Everyone’s “hi” looks the same on a cell phone screen. Friends might respond with LOL if I
text something funny, but it is nothing compared to hearing their real laugh.
(e) _____. When people just use a phone, they can avoid car accidents, save money, and have more personal
communication with their family and friends. I do not think texting will ever go away, but maybe soon more
people will long for the good old days of a simple phone call.
_______118. Although texting is popular, these reasons have shown calling is just as good, and often
better.
_______119. In addition, phone calls are cheaper.
_______120. First of all, phone calls are much less dangerous than texting.
_______121. People who text say that it is an easier way to communicate.
_______122. Another reason I prefer calling to texting is because it is more personal.

ស្ំណួរត្រៀម្រឡងតរស្តភាសាអង់តលេស្ក្មិរ ៣
READING SKILLS
Unit 1
Read the article. Then choose the best response.

Conversation Skills
1 The greatest way to make a good first impression is to demonstrate that the other person, not you, is the center
of attention. When you are only interested in talking about yourself, other people don’t feel that you appreciate
them. Show that you are interested in others. Then new acquaintances will want to see you again. Recently I
went to a conference. At lunch, my wife and I sat with several people we didn’t know. While most of the people
made good impressions, one man did not. We decided to avoid him all weekend. He talked about himself the
entire time. No one else got a chance to speak. Unfortunately, he probably thought we were interested in his life
story. I like this definition of a bore: “Somebody who talks about himself so much that you don’t get to talk
about yourself.”
2 You’ll impress other people when you practice good listening skills. Give interested responses: “Hmmm ...
interesting!” “Tell me more, please.” “What did you do next?” Your partner will welcome your help in keeping
the conversation going. You show you’re a good listener when you maintain steady eye contact. Think about
how you feel when someone you are talking to looks around the room. You think the person would rather be
talking to someone else. Try to use the name of the person you’ve just met frequently. “Judy, I like that
suggestion.” “Your vacation must have been exciting, Fred.” You show that you have paid attention from the
start, catching the name during the introduction. Equally as important, you’ll make conversations more personal
by including the listener’s name several times.
3 Avoid making other people uncomfortable. Be careful with jokes. Something you think is funny may be
offensive to others. You don’t want to hurt someone’s feelings. Also, don’t disagree with someone you have
just met. If you disagree too much in your first conversation, the other person may think you are just too
different. When you know the person better, you can express another opinion.

Read the statements. Write T (true) or F (false).


_______ 1. You should make yourself the center of attention.
_______ 2. A bore is someone who lets you talk about yourself.
_______ 3. Keeping eye contact shows you are interested in someone.
_______ 4. Jokes can hurt other people’s feelings.
_______ 5. You should disagree with people you have just met.
Read the sentence. In Paragraph 2, is it a supporting detail or main idea? Choose A or B.
6. Practice good listening skills.
A. supporting detail
B. main idea
7. Help keep the conversation going.
A. supporting detail
B. main idea
8. Use your partner’s name.
A. supporting detail
B. main idea

Read the sentence. In Paragraph 3, is it a supporting detail or main idea? Choose A or B.


9. Don’t make offensive jokes.
A. supporting detail
B. main idea
10. Don’t disagree with someone you’ve just met.
A. supporting detail
B. main idea
11. Don’t make people feel uncomfortable.
A. supporting detail
B. main idea

Match each sentence with the word that completes it.


12. Smiling can _____ to others that you are a friendly person. A. confidence
13. When you are well prepared for a speech, you usually have more _____. B. demonstrate
14. It is important to ask a lot questions when you want to _____ a conversation. C. maintain
15. The guests _____ the kindness of their host. D. strangers
16. The student _____ his words carefully when he talked to the teacher. E. selected
17. I am very comfortable talking with my friends, but I am shy around _____. F. appreciated
Complete the sentence with the correct word from the word bank:
research weakness professional punctual exaggerates considers
18. The man looked ____________________ in his black suit and tie.

19. She ____________________ it rude to interrupt people when they are talking.

20. The child always ____________________ his stories, but his mother knows the truth.

21. Finding the right job can require a lot of time and ____________________.

22. Talking in public is her ____________________, so she doesn’t do it often.

23. He tries to be ____________________, but he is still always late to class.

Complete the sentence. Choose A, B, or C.


24. She is an _____ singer who has written many famous songs.
A. accomplish
B. accomplished
C. accomplishment
25. I felt _____ less nervous after I finished my speech.
A. consider
B. considerable
C. considerably
26. He spoke so _____ that I had no doubt he was telling the truth.
A. confident
B. confidently
C. confidence
27. The teacher’s _____ was very clear and easy to follow.
A. demonstrate
B. demonstrative
C. demonstration
28. He wanted to _____ the interviewer, so he dressed professionally.
A. impress
B. impressive
C. impression
29. Taking care of a child is a big _____.
A. responsible
B. responsibly
C. responsibility
Match each sentence beginning with the correct ending.
_____ 30. People will know you’re interested A. when you speak at a job interview.
_____ 31. You will make a bad impression B. people will understand you better.
_____ 32. If you are late, C. if you do not dress appropriately.
_____ 33. You should not use slang D. employers will want to hire you.
_____ 34. If you smile, E. people will think you are friendly.
_____ 35. If you have a strong job history, F. you will miss your appointment.
_____ 36. When you speak slowly, G. if you maintain eye contact.

Choose the best response.


37. Which is the best topic sentence?
A. My first teacher made a very good impression on me.
B. My first teacher always came to class well prepared.
C. My first teacher greeted the students kindly.
38. Which is the best topic sentence?
A. A good doctor tries to understand how patients feel.
B. A good doctor has two important qualities.
C. A good doctor is caring toward patients.
39. Which is the best supporting sentence?
A. Read the job descriptions very carefully.
B. Here are five tips for searching job websites.
C. Finding the right job online is not easy.
40. Which is the best concluding sentence?
A. First of all, you need to listen to them so they know you care.
B. Do not interrupt them when they are speaking and keep steady eye contact.
C. When you follow these steps, it is easier to deal with difficult customers.
Complete each sentence with the correct form of the word in parentheses.
32. The word blackberry ____________________ (have) a nice sound.

33. An uncommon name ____________________ (get) your attention.

34. Good names ____________________ (be) easy to say.

Read each pair of nouns in parentheses. Choose the proper noun and write it with a capital letter.
35. _______________________________________________________________

(girl / anna)

36. _______________________________________________________________

(july / month)

37. _______________________________________________________________

(city / tokyo)

Correct the error in capitalization in each sentence.


38. My first name is kelly.

_______________________________________________________________

39. I was named for my irish grandfather.

_______________________________________________________________

40. He came to the united states when he was 17 years old.

_______________________________________________________________

UNIT 2
Read the article. Then choose the best response.

The Tongue and Taste


How the Tongue Works
1 The human tongue is made up of a group of muscles and taste buds that work together to recognize taste. The
average adult tongue has 10,000 taste buds, which are tiny bumps located on the tongue. Tiny hairs on the end
of the taste buds tell us whether food is sweet, sour, bitter, or salty. The taste buds send messages to the brain as
chemicals from the food enter the nose. Together, the taste buds and nose tell the brain exactly what the tongue
is tasting. This complex system helps humans survive by recognizing which foods are safe and which might be
dangerous.
Nontasters, Medium Tasters, Supertasters
2 Although all humans have taste buds, we do not all have the same number of them. Medium tasters typically
have 10,000 taste buds. These “average tasters” make up about 50 percent of the world population. Nontasters,
25 percent of the population, have half the number of taste buds as medium tasters. The remaining 25 percent
are supertasters. Supertasters have four to six times as many taste buds as nontasters and twice as many as
medium tasters. Research shows that supertasters are more likely to be women and from Asia, Africa, and South
America.
Different Worlds for Different Tasters
3 Supertasters live in a very colorful world of tastes, nontasters live in a gray world, and medium tasters are
somewhere between the two. Supertasters think that a lot of foods are too strong. In addition to having more
taste buds, supertasters are born with a gene that makes them sensitive to bitter foods. Consequently, they
dislike broccoli, cauliflower, grapefruit, and even coffee. With more taste buds, they can more easily feel fatty
foods in their mouths. As a result, they stay away from high-fat food items like french fries and sweets. They
are also very sensitive to pain on the tongue, so they avoid spicy food. Nontasters, on the other hand, experience
fewer tastes in general, so they can enjoy hot foods like chili and pepper with much less pain.
Supertasters, Nontasters, and Diet
4 As a rule, humans avoid foods that taste bad and eat foods that give them pleasure. Since supertasters avoid
bitter fruits and vegetables, their diets are sometimes not balanced, which could put them more at risk for
certain types of cancers. However, they also dislike fatty and sweet foods, so they tend to be thinner and at
lower risk for heart disease and diabetes. In contrast, nontasters like foods high in fat because their tongues do
not react negatively to them. All people should pay attention to what they eat, but nontasters and supertasters
must be more aware of the foods they are consuming or avoiding and find other ways to make up the difference.
Match each subtitle with the correct information from the text.
A. why supertasters live in a colorful world and nontasters live
1. Supertasters, Nontasters, and Diet
in a gray world
2. Different Worlds for Different
B. how taste buds send signals to the brain about what the
Tasters
tongue is tasting
3. Nontasters, Medium Tasters,
C. why supertasters and nontasters need to watch what they eat
Supertasters
D. how many taste buds nontasters, medium tasters, and
4. How the Tongue Works
supertasters have

Read the statements. Write T (true) or F (false).


_______ 5. The average adult has 10,000 taste buds.
_______ 6. Taste buds send chemical messages to the nose.
_______ 7. Supertasters have twice as many taste buds as nontasters.
_______ 8. Nontasters dislike broccoli and cauliflower.
_______ 9. Supertasters tend to be at a low risk for heart disease.

Match each sentence with the word(s) that complete(s) it.


10. A _____ diet can help people stay healthy. A. likely
11. People who smoke are more _____ to develop heart disease than nonsmokers are— B. identify
two to four times as likely, in fact. C. sensitive
12. Busy people _____ eat their meals very quickly. to
13. You are _____ to gain weight if you eat a lot of fatty foods and don’t exercise. D. at risk
14. Food labels help people _____ how many calories foods have. E. balanced
15. She does not eat bread because she is _____ wheat. F. typically

Complete the sentence with the correct word from the word bank:
method concept portion region cuisine property

16. It is hard for young children to understand the ____________________ of nutrition.

17. Foods and recipes are different from one ____________________ of France to another.

18. A healthy diet includes a large ____________________ of fruits and vegetables.

19. Yang is the ____________________ of Chinese cooking that includes warm foods.

20. Boiling vegetables is a healthier cooking ____________________ than frying them.

21. The southern part of the country is known for its spicy ____________________.
Read the sentence. Choose the phrase that shows the meaning of the underlined word.
22. The coffee was too strong for my taste, so I didn’t drink it.
A. the flavor of food or drink
B. a liking for something
23. People consume a lot of calories when they run.
A. to use something, such as fuel, energy, or time
B. to eat or drink
24. The restaurant uses a special system to keep its drinking water clean.
A. a group of things or parts that work together
B. the body of a person or animal
25. The famous French chef was recognized for his amazing dish.
A. to know again something you have seen or heard before
B. to show that you think that someone has done something good
26. In England, it is a common practice for people to put milk in their tea.
A. the usual way of doing something
B. doing something many times so you become good at it
27. The tongue can have a lot of influence on food preferences.
A. the power to affect, change, or control something
B. a person or thing that affects or changes something

Complete the sentence. Choose A or B.


28. We ate at a _____ restaurant.
A. Brazilian cheap
B. cheap Brazilian
29. She wore a _____ dress to her wedding.
A. silk traditional
B. traditional silk
30. We got a _____ table for the living room.
A. round little
B. little round
31. He helped the _____ woman cross the street.
A. nice, elderly
B. elderly, nice
32. The soup was served in _____ bowls.
A. old white ceramic
B. old ceramic white
33. She bought a pair of _____ boots.
A. expensive leather hiking
B. hiking expensive leather
34. He put an _____ desk in his bedroom.
A. Asian writing ancient
B. ancient Asian writing

Mahshee is a delicious Middle Eastern dish with a unique taste. Mahshee tastes (a) _____. It is made with (b)
_____ rice and many spices. The rice is wrapped in (c) _____ leaves. It is often served with (d) _____ tomatoes.
You can enjoy Mashee as a snack or a (e) _____ meal with family and (f) _____ friends.

Complete the paragraph with the adjectives below. Write the letter next to the correct adjective(s).
_____ 35. fresh, red
_____ 36. full
_____ 37. brown or white
_____ 38. boiled grape
_____ 39. sweet but spicy
_____ 40. close

UNIT 3
Read the article. Then choose the best response.

The Power and Profits of F1 Racing


Does your business need a boost?
1 Imagine 350 million people seeing your company logo every year. Imagine that number growing even higher
every year. Imagine being part of one of the most prestigious and glamorous sports in the world and making
millions of dollars at the same time. Sound attractive? Hundreds of companies have already discovered the
financial benefits of sponsoring Formula 1 racing. When you choose to sponsor a team, you can be assured that
your company will grow financially and globally.
Why are companies interested?
2 Companies have realized that investments in the sport of auto racing can bring them huge profits. Businesses,
including banks, hotels, and telecommunication companies, invest tens of millions of dollars every year to
sponsor race teams. Hundreds of millions of people watch car races every year. For companies, this is an
enormous market. Cars race around the track with company logos stuck to the doors, hood, and trunk, and
people notice. Corporate sponsors can invest $5 million in a race team, and make $30 million or more from car
advertising. The costs are cheap compared to the profits. Sponsoring a team also shows the financial stability of
your company. Race cars can cost tens of millions of dollars, and race teams can spend up to $300 million a
year. Companies who invest in race teams are showing the world that they are powerful and dependable.
Why is investing now a good idea?
3 Much of Formula 1’s current success comes from its expansion to global markets. Although most races are in
Europe, today there are races in the Middle East and Asia. Companies support worldwide expansion because it
gives them new customers in emerging markets. They can push their brand globally. Many companies have
already invested in Formula 1’s most recent host locations, including Manama, Abu Dhabi, and Singapore. As a
result, they have been able to expand their business to the Middle East and Asia. These areas of the world are
full of business opportunities, and Formula 1 racing has brought them more growth and success. Expanding
overseas also shows that your company has a global message, which is important in today’s global economy.

Read the statements. Write T (true) or F (false).


_______1. Investing in F1 racing can be profitable.
_______2. Banks and hotels invest in F1 race teams.
_______3. The costs of investing in F1 racing are higher than the profits.
_______4. Investing in F1 racing can make a company look powerful.
_______5. Most F1 races are in the Middle East and Asia.
Scan the text. Choose the best response.
6. How many people watch car races every year?
A. one million
B. tens of millions
C. hundreds of millions
7. How many dollars can corporate sponsors make from race car advertising?
A. 5 million
B. 30 million
C. 300 million
8. How many dollars can race cars cost?
A. hundreds of millions
B. tens of millions
C. hundreds of thousands
9. How much can race teams spend a year?
A. up to 300 million
B. up to 30 million
C. up to 5 million

Match each sentence with the word that completes it.


10. It can be risky to _____ in a brand-new company. A. sponsor
11. Businesses are interested in the _____ of F1 racing into new countries. B. expansion
12. Children are the biggest _____ for toy companies. C. dependable
13. Bosses expect their employees to be responsible and _____. D. invest
14. The soccer team’s uniforms show the names of companies that _____ the team. E. assured
15. The racing team was _____ that it would receive money from the company. F. market

Match each underlined word with the correct definition.


_____ 16. It can take several months to
recover from an overuse injury.
_____ 17. It is a growing trend for younger A. needing a great deal of effort
athletes to play professional B. to get better after an illness or
sports. accident
_____ 18. The child’s tears were a sign that C. a person or thing that is not
she was in a lot of pain. included
_____ 19. With the exception of one D. something that shows someone or
player, everyone on the team something is present, exists or may
participated. happen
_____ 20. The referee made a motion with E. a movement or way of moving
his hand to stop the game. F. a general change or development
_____ 21. Professional athletes have a
demanding schedule.
Complete each sentence with the words from the word bank:
involved in sure about upset about due to known for nervous about

22. The athlete’s hands were shaking because he was ____________________ playing his first game.

23. F1 racing is ____________________ being an expensive sport.

24. The coach looked ____________________ losing the championship game.

25. Her injury was ____________________ a skiing accident.

26. Some teenagers are ____________________ two or three sports at a time.

27. She is not ____________________ going to the game tonight because she feels tired.

Correct the error in subject-verb agreement. Rewrite each sentence correctly.


28. The football team practice every other day.

_______________________________________________________________

29. The coach does not likes players who quit.

_______________________________________________________________

30. He and his son watches soccer on TV.

_______________________________________________________________

31. It do not look like she is going to win.

_______________________________________________________________

32. Gymnastics are a fun but demanding sport.

_______________________________________________________________

33. They do not wants the athlete to stop playing.

_______________________________________________________________

34. The instructor teach his class on Saturdays and Sundays.

_______________________________________________________________
Read the sentences for an opinion paragraph. Then number the sentences to show the correct order.
_______ 35. Cheap tennis rackets only cost around thirty dollars.
_______ 36. Tennis is a good sport to play.
_______ 37. First, it does not cost much money.
_______ 38. Tennis players strengthen their legs when running.
_______ 39. Second, it is good for your body.
_______ 40. Running also helps exercise the lungs.

UNIT 4
Read the article. Then choose the best response.

The Benefits of Second Life


1 Second Life offers users a quick and easy escape from the real world. Many residents see this as one of the
main benefits of using the site. Spending time on Second Life allows them to escape the stresses and problems
of their daily lives. If a user is having a stressful day at work, she can visit a beautiful island, go skiing, or even
fly to another planet during her lunch break. If she is tired at the end of a long day, she can go to a classical
music concert while dinner is cooking and never leave home. Users can even visit other planets to help them
forget their “first-life” problems for a little while.
2 In addition to escaping the stress of their daily lives, users can also escape who they are in the real world and
live out their fantasies. For example, residents can change their occupations, physical appearance, and even
their nationalities. A doctor from the United States can be a Brazilian musician on Second Life, or a banker can
choose to be an Olympic basketball player. Basically, Second Life lets users live in a world without limitations.
This is very exciting to many people.
3 Although Second Life started as a way for people to escape the real world, it has become more and more like
the real world in many ways. Now on Second Life, some countries have virtual embassies, businesses have
meetings in virtual rooms, and universities have places where students can view the campus and take classes.
Today, Second Life allows users to experience both fantasy and reality in the same place.

1. When can users make a quick visit to Second Life?


A. before going to work
B. during their lunch break
C. after eating their dinner
2. What can users do to relax after a stressful day?
A. hold a business meeting
B. cook their dinner
C. go to a concert
3. What can people do in Second Life that they cannot do in real life?
A. change their nationality
B. visit other planets
C. go skiing
4. The passage mentions all of the things Second Life users can do EXCEPT:
A. get married
B. choose an occupation
C. change their appearance
5. The passage mentions all of these real-life organizations on Second Life EXCEPT:
A. universities
B. embassies
C. banks

Match the paragraph with the notes that summarize it.


_____ 6. Paragraph 2 A. Second Life = more like real world
_____ 7. Paragraph 3 B. Second Life = live out fantasies
_____ 8. Paragraph 1 C. Second Life = escape the real world quickly
Choose the best response.
9. Next to which paragraph should a student write, “escape stress --> visit island”?
A. paragraph 1
B. paragraph 2
C. paragraph 3
10. Next to which paragraph should a student write, “US doctor --> Brazilian musician on Second Life”?
A. paragraph 1
B. paragraph 2
C. paragraph 3

Complete each sentence with the correct word from the word bank:
realistic social transactions explore interact benefits

11. She took a computer class to ____________________ her new interest in computers.

12. The pictures looked so ____________________ no one knew they were drawn by hand.

13. He didn’t go to the movie with friends because he doesn’t enjoy ____________________ events.

14. The Internet lets people easily ____________________ with others all over the world.

15. Free airplane travel is one of the ____________________ of joining an airline's frequent-flyer program.

16. Many banks let their customers make ____________________ online.

Match each sentence with the word that completes it.


17. The new job changed Roy’s _____, and now he spends his time doing different
A. regret
things.
B. discover
18. I send my sister an e-card when she has a special _____, such as her birthday.
C. survive
19. My mother did not _____ her decision to go back to school after twenty years.
D. occasion
20. The _____ tested how often people used false names while chatting online.
E. lifestyle
21. Traveling to foreign countries helps people _____ new foods and cultures.
F. experiment
22. Some people cannot _____ if they lose their cell phones.
Read the sentence. Choose the word that shows the meaning of the underlined word.
23. Because she moved away, they had to communicate by phone.
A. talk
B. review
C. recognize
24. The website lets people build a fake house online.
A. virtual
B. original
C. demanding
25. Safety is important when purchasing an item over the Internet.
A. advertising
B. buying
C. discussing
26. Getting married is an important time in a person’s life.
A. moment
B. portion
C. opportunity
27. One disadvantage of compact discs is that they get scratched easily.
A. trend
B. feature
C. limitation
28. My father has an unusual ability to know exactly what I am thinking.
A. dependable
B. rare
C. special

Correct the error in parallel structure in the sentence.


29. He enjoys listening to music, going to concerts, and he likes playing the guitar.

______________________________________________________________

30. She is interested in starting a website and sells her paintings online.

_______________________________________________________________

31. On modern cell phones, people can chat, text, and there are games.

_______________________________________________________________

32. They watched TV instead of going out to a movie or they eat at a restaurant.

_______________________________________________________________

33. The program lets people find information easily and it is quick.

_______________________________________________________________
34. Instead of driving and she sits in traffic, she decided to bike to work.

_______________________________________________________________

35. You can buy the product in stores or by going on the Internet.

_______________________________________________________________

1 Second Life offers users a quick and easy escape from the real world. Many residents see this as one of the
main benefits of using the site. Spending time on Second Life allows them to escape the stresses and problems
of their daily lives. If a user is having a stressful day at work, she can visit a beautiful island, go skiing, or even
fly to another planet during her lunch break. If she is tired at the end of a long day, she can go to a classical
music concert while dinner is cooking and never leave home. Users can even visit other planets to help them
forget their “first-life” problems for a little while.

Read the sentence. According to the text, is it a summary or a personal response? Choose A or B.
36. Second Life residents like getting away from their “first-life” problems.
A. summary
B. personal response
37. Life today is too busy and full of many problems.
A. summary
B. personal response
38. The online environment helps many people feel less stressed.
A. summary
B. personal response
39. People can explore new places and worlds through websites.
A. summary
B. personal response
40. It would be fun to explore other planets after I get home from work.
A. summary
B. personal response

UNIT 5
Read the article. Then choose the best response.
Giving Feels Good
1 Neuroeconomists Bill Harbaugh and his team at the University of Oregon study the biology of altruism. They
look specifically at neuroeconomics, or the connection between the brain and economic decisions. In one of
their experiments, the researchers tried seeing if people’s donations to charity were affected by neurons.
Nineteen women were given $100 to play a charity game on the computer. They could choose to donate or not
to a charity, each decision leading to other situations where they could gain or lose money.
2 At the end of the game, the subjects were able to keep all the money that was left in their accounts. As the
subjects played the game, the scientists scanned their brains. They looked at the brain’s “pleasure center,”
which controls how good people feel. When most subjects donated money to a charity, their pleasure centers lit
up on the brain scan. Some even lit up when the subjects were taxed on their donation. Both results suggest that
the brain’s pleasure center is rewarded for altruistic acts. In addition, the more people donated, the more their
pleasure center lit up. For some, the pleasure center lit up more when the computer gave the charity extra
money than when they received extra money to keep for themselves. The scientists point out that this was “the
first neural evidence for … pure altruism,” meaning that altruism may indeed have a biological connection.
Read the statements. Write T (true) or F (false).
_______ 1. Neuroeconomics is the study of the brain’s role in economic decisions.
_______ 2. Nineteen men and women played the charity game.
_______ 3. The subjects had to give back the money they won.
_______ 4. The charity game taxed people’s donations.
_______ 5. Being altruistic can affect people’s brain activity.
Read the sentence. Should it go in the research, results, or conclusion section of a graphic organizer?
Choose A, B, or C.
6. Altruism has a biological connection.
A. research
B. results
C. conclusion
7. Subjects played a game that let them donate money.
A. research
B. results
C. conclusion
8. Subjects’ pleasure centers lit up on a brain scan.
A. research
B. results
C. conclusion
9. Taxes on donations lit up subjects’ pleasure centers.
A. research
B. results
C. conclusion
10. The computer gave the charity extra money.
A. research
B. results
C. conclusion
Match each sentence with the word(s) that complete(s) it.
11. Parents have a _____ to take good care of their children.
12. The bystander effect can _____ situations where strangers see someone in need A. theory
of help. B. factor
13. The _____ explained to the police officer what she saw. C. responsibility
14. The cost of tuition is one _____ many students consider when choosing which D. prove
university to attend. E. witness
15. It can take researchers many years to _____ a hypothesis. F. apply to
16. The researchers did experiments to test their _____ about how the brain works.

Match the underlined word(s) with the correct synonym.


_____ 17. The initial study did not work, so changes were made the second time.
A. guessed
_____ 18. The study brought about surprising results.
B. understanding
_____ 19. The family expects nothing in return when giving their time to
C. needed
altruistic causes.
D. helping
_____ 20. My mother has always been compassionate toward her children.
E. caused
_____ 21. The success of the scientific experiments relied on careful planning.
F. first
_____ 22. The scientists hypothesized that the bystanders would not help.

Complete each sentence with the correct phrasal verb from the word bank:
ended up pointed out set up called out helped out figured out

23. In the magazine article, the author ____________________ that this was a new theory.

24. The people involved in the car accident ____________________ for help.

25. The doctor and patient ____________________ an appointment to discuss the test results.

26. Eventually, the student ____________________ the solution to the problem.

27. When the man fell down on the sidewalk, a number of people ____________________.

28. The scientist was happy that he ____________________ with good results.
Correct the error in the use of gerunds or infinitives in the sentence.
29. Spending time with loved ones make people feel happy.
_______________________________________________________________

30. He appeared being worried about getting to class late.

_______________________________________________________________

31. Live with a roommate can sometimes be difficult.

_______________________________________________________________

32. He considered to move so he could be closer to his family.

_______________________________________________________________

33. She wants become a social psychologist after she graduates from college.

_______________________________________________________________

34. Being a good friend mean helping out whenever you can.

_______________________________________________________________

35. Make mistakes is more common when people are nervous.

_______________________________________________________________

Having a big family makes children learn to be more helpful. (a) _____. For example, children know cleaning
the house will go faster if they help their parents. (b) _____. Then the parents will be able to spend time doing
more fun things with the kids. (c) _____. Older children often help their younger brothers and sisters when their
parents are busy. (d) _____. Older brothers and sisters can also help younger siblings with their homework.
There are many benefits to having a large family. (e) _____.

Complete the paragraph with the sentences below. Write the letter next to the correct sentence.
_______ 36. The house gets clean faster because everyone cleans a small part.
_______ 37. For example, an older sister can help a younger sister get ready for school.
_______ 38. First, children end up helping their parents more.
_______ 39. When there are many children in a family, every one learns an important lesson about helping
one another.
_______ 40. Second, children learn to help each other more.

UNIT 6
Read the article. Then choose the best response.

Regulating the Ads We See


1 It is difficult to get away from exposure to advertisements. The problem is not just the number of
advertisements people see. Around the world, people are also looking for ways to regulate, or control, the kind
of advertising we see on a daily basis. Some countries believe it is the government’s job to regulate advertising.
In other countries, advertising companies agree to regulate themselves.
2 In recent years, organizations have studied the consequences of advertising on people, and especially children.
A study by the Kaiser Family Foundation, for example, supported the belief that fast-food advertising can lead
to childhood obesity (being very overweight). Since that study, many European countries have tried to regulate
advertising that can negatively influence children.
3 There has been a long debate in Europe about advertising regulation, and many European governments have
taken steps to do more. In Greece, the government does not allow any television advertisements for children’s
toys between 7 am and 10 pm. The Swedish government also regulates child advertising. In 1991, it banned any
advertising that was directed at children under age twelve.
4 Other countries around the world regulate advertisements, but not through the government. Instead,
advertising companies in nations such as South Africa and Canada agree to regulate themselves. In these
countries, advertisers work with private broadcasting companies to set standards that they promise to support
and maintain.
5 In Canada, for example, advertisements cannot lie or take advantage of TV viewers. Advertisers are not
allowed to show pictures that might scare children. Advertisers also cannot exaggerate the prices of items. They
have to make sure information about prices is easy for people to read. Canada’s self-regulating advertising
standards are simple but beneficial. They help to make sure advertising stays as honest as possible.
6 Advertising regulation can take two basic forms. It can be controlled by governments or by advertisers
themselves. Either way, the goals are the same. Ads should be truthful, and they should protect children from
any harmful messages.

Match the main idea with the supporting detail from the text.
A. Either governments or advertising
_______ 1. Advertisers in Canada cannot
companies can control the types of
try to trick people.
advertisements shown on TV.
_______ 2. In some countries,
B. No ads for toys can be shown during the
advertising companies
day in Greece.
regulate themselves.
C. Advertising and broadcasting companies in
_______ 3. Advertising can be regulated
South Africa set advertising standards
in different ways.
together.
_______ 4. Some governments regulate
D. There is a connection between watching
advertising.
ads for fast food and being overweight.
_______ 5. Advertising can negatively
E. TV viewers must be able to understand
affect children.
easily how much a product costs.
Read the sentence. Is it a fact or an opinion? Choose A or B.
86. It is difficult to get away from exposure to advertisements.
A. fact
B. opinion
87. Many European countries have tried to regulate advertising.
A. fact
B. opinion
88. The Swedish government banned advertising directed at children under twelve.
A. fact
B. opinion
89. Advertisers in Canada are not allowed to exaggerate the prices of items.
A. fact
B. opinion
90. Canada’s self-regulating advertising standards are simple but beneficial.
A. fact
B. opinion

Match the underlined word(s) with the correct synonym.


_______ 11. Ads often imply that a product is perfect for you. A. belonging
_______ 12. A recent ad in the paper promised people a free CD player. B. ignore
_______ 13. When you buy a product, it becomes your possession. C. effect
_______ 14. Many people dismiss advertising as trivial, but it is actually quite useful. D. unimportant
_______ 15. Many children are not able to tune out toy commercials. E. new
_______ 16. One consequence of watching too much TV is becoming lazy. F. suggest

Complete the sentence with the correct word from the word bank:
entertain annual donation memorable surrounding exposure

17. Public radio stations ask their listeners for a yearly ____________________ to help pay their costs.
18. The commercial has a very ____________________ song that I can’t get out of my head.
19. Children should not have easy ____________________ to violent movies on television.
20. People often laugh at ads that ____________________ them with funny or strange situations.
21. The ads ____________________ the football stadium can be seen from far away.
22. Many stores increase their advertising during ____________________ holidays like New Year’s Day.
Choose the word that is a different part of speech from the other words.
23.
A. friendly
B. recently
C. certainly
24.
A. unpredictable
B. offensive
C. annually
25.
A. donation
B. painful
C. happiness
26.
A. possession
B. professional
C. memorable
27.
A. relationship
B. dissatisfaction
C. annoying
28.
A. basically
B. unpredictably
C. broadcasting

Read the compound sentence. Choose the correct reason for using the underlined coordinating
conjunction.
96. The movie was terrible, so we decided to leave early.
A. related ideas
B. contrasting ideas
C. a result
D. a choice
97. I like reading the stories in the magazine, but it has too many ads.
A. related ideas
B. contrasting ideas
C. a result
D. a choice
98. Advertisers can make TV ads, and they can advertise on the Internet.
A. related ideas
B. contrasting ideas
C. a result
D. a choice
Complete the sentences with the correct word from the word bank:
or and but so

32. I enjoy watching television, ____________________ I hate seeing commercials.

33. You can shop for appliances online, ____________________ you can go to a store.

34. Companies advertise during sporting events on television, ____________________ they put up ads at

sports stadiums.

35. He was tired, ____________________ he turned off the TV and went to bed.

Dear Editor,
My eight-year-old daughter loves to watch her favorite shows on the weekend. I don’t mind her watching a little
TV to relax, and I often sit with her so we can enjoy the shows together. However, I am getting tired of having
to wait through all the ads to see what happens next. Recently, I watched a show with my daughter, and there
were more than ten ads! (a) _____.
(b) _____. For example, they do not know that ads make them want something they don’t need. Just yesterday,
my daughter asked to go to the store to get her a game that was advertised during her favorite show. She got
upset when I tried to explain that she didn’t really need the game.
(c) _____. The ads are usually much louder than the shows children are watching. (d) _____. Imagine the
damage a lot of ads could do to children’s hearing!
My daughter is a smart girl, but she is not able to understand the real power behind advertising. And I worry
about loud ads damaging her hearing. (e) _____. That is why I think there should be no more ads played during
children shows.
Sincerely,
A Concerned Parent

Complete the letter to the editor with the sentences below. Write the letter next to the correct sentence.
_______ 36. Every time an ad comes on, I have to turn down the volume so it doesn’t hurt my own ears.
_______ 37. I do not think television stations should play ads during kids’ shows.
_______ 38. My other concern is that today’s ads may be dangerous for children’s ears.
_______ 39. Advertising can confuse and harm children.
_______ 40. First of all, children aren’t old enough to understand how ads work.

UNIT 7
Read the article. Then choose the best response.

Buildering
1 Many people have an appetite for climbing. For some, however, the ultimate goal is to climb buildings, not
mountains. Buildering is the act of climbing on the outside of buildings, such as skyscrapers. It is named after
bouldering, which is a style of rock climbing. It is a dangerous activity that has caught the attention of both
climbers and the police.
The Sport
2 Buildering is usually done alone. A single climber will choose a building to climb, usually a tall building in a
major city like London, New York, or Taipei. Climbers prepare for their climb by deciding the best route to take
up the structure. They must also figure out what kind of equipment they will need. It often includes items such
as ropes and bodysuits. Sometimes, however, they climb “free,” with no ropes at all.
The Dangers
3 Not surprisingly, buildering is a very dangerous sport. Climbers put their lives at risk every time they go up
the side of a building. They are also at risk of being arrested by the police. Climbing on the outsides of
buildings is illegal. They are considered private property. The police often wait for the climber to come back
down. Then they arrest the climber for breaking the law. For this reason, many climbers choose to climb at
night so that they do not get caught.
Famous Climbers
4 Buildering may sound like a modern sport, but it has actually been around for more than a hundred years. In
the 1890s, Geoffrey Winthrop Young, a student at Cambridge University in England, used to participate in
“roof climbing.” He even wrote a climbing guide for some of the buildings at the university. After Young, other
students followed, though most performed their risky acts at night.
5 Today, the sport is made most famous by French climber, Alain Robert. Known as the “French Spider-Man,”
Robert has climbed some of the world’s tallest skyscrapers. Some of them include the Eiffel Tower in Paris and
the Petronas Towers in Malaysia. (He was arrested twice at the 60th floor of the Petronas Towers.) Robert is
somewhat of a celebrity. He enjoys the crowds of people who watch him conquer the tallest buildings of the
world—one skyscraper at a time.

Read the statements. Write T (true) or F (false). Then correct each false statement to make it true.
_______1. Buildering is the act of climbing the inside of buildings.

____________________________________________________________

_______2. Large groups of climbers usually go buildering together.

____________________________________________________________

_______3. Buildering is typically done at night.

____________________________________________________________
_______4. Geoffrey Winthrop Young climbed buildings at Cambridge University.

____________________________________________________________

_______5. Alain Robert has climbed some of the tallest buildings in the world.

____________________________________________________________

Match each underlined referent with the noun to which it refers.


_______ 6. He even wrote a climbing guide for some of the buildings.
_______ 7. They are considered private property. A. buildings
_______ 8. They must also figure out what kind of equipment they will B. Geoffrey Winthrop
need. Young
_______ 9. He enjoys the crowds of people who watch him conquer the C. buildering
tallest buildings of the world. D. climbers
_______ 10. It is a dangerous activity that has caught the attention of both E. Alain Robert
climbers and the police.

Complete the sentence with the correct word from the word bank:
trait aspect mental perceive challenge precaution

11. She has a unique ____________________ ability to make herself focus only on the present.

12. Staying calm in dangerous situations is a ____________________ for many people.

13. Mountain climbers must take every ____________________ to keep themselves safe.

14. Getting hurt easily is one negative ____________________ of participating in extreme sports.

15. Many extreme athletes share a personality ____________________ that lets them enjoy risk.

16. He does not ____________________ rock climbing to be as dangerous as I do.


Complete the sentence. Choose A, B, or C.
17. For risk takers who like physical activity, mountain climbing is the _____ challenge.
A. trivial
B. unfortunate
C. ultimate
18. The skier was _____ to compete in next year’s Olympic games.
A. determined
B. embarrassed
C. implied
19. Melinda _____ stepped off the bridge and took her first bungee jump.
A. extremely
B. bravely
C. cheerfully
20. The climbing teacher’s _____ was to make sure that everyone reached the mountain top safely.
A. risk
B. role
C. route
21. There was a _____ amount of snow on the ground, so it was hard to walk.
A. significant
B. complex
C. realistic
22. The athlete has a _____ quality that is unlike anybody else’s.
A. famous
B. friendly
C. distinctive

Read the sentence. Choose the correct definition for the underlined word.
23. The setting sun made the side of the mountain a vivid orange.
A. having very bright light or color
B. producing a strong, clear picture in your mind
24. The strong winds drove us to the other side of the river.
A. control a car or other moving vehicle
B. force somebody to move somewhere else
25. The Olympic athlete never stopped fighting to reach her goals.
A. trying hard to do something
B. arguing about something
26. She earned the respect of her competitors after winning the race.
A. get something that you deserve
B. get money by working
27. According to his friends, John fits the image of someone who loves taking risks.
A. a picture or photograph of a person, animal, or thing
B. the opinion people have about a person
28. The team embraced the coach’s suggestion to practice an extra hour each day.
A. put your arms around someone
B. accept a new idea or opinion
Complete the paragraph with the correct form of the verbs in parentheses. Use the simple past, past
perfect, or simple present.
Rock climbing (29) ____________________ (be) a fun, but dangerous, sport.

When I (30) ____________________ (go) rock climbing last year, I felt a little nervous. My friends and

I (31) ____________________ (get) to the site early in

the morning. I (32) ____________________ (eat) a good breakfast, but my stomach was still full of

nerves. Finally, I (33) ____________________ (start) to climb.

With my friends’ help, I (34) ____________________ (make) it to the top. That day (35)

____________________ (be) one of the most exciting days of my life.

Choose the best response.


36. Choose the sentence that gives background information.
A. About three years ago, I participated in my first marathon.
B. Because of the race, I learned an important lesson.
C. The more prepared I am, the better I will feel.
37. Choose the sentence that describes why the story is important to the author.
A. My mother always told me not to go out late at night.
B. One night, I had a scary experience walking home.
C. I never ignored my mother’s advice again after that scary night.
38. Choose the sentence that describes what the author learned.
A. All my friends ran over to make sure I was not hurt.
B. I will never forget my scary experience with the car that day.
C. That day showed me why I needed to be safe on the street at all times.
39. Choose the event that happened first.
A. We divided into two teams and each team chose one end of the street.
B. When the game started, I ran towards the ball.
C. I made the first goal of the game, and I was ready for more.
40. Choose the event that happened last.
A. While we were playing, a fast car came out of nowhere.
B. I jumped as fast as I could to get out of the way, and the car just missed me.
C. The driver didn’t see me, and the car came closer and closer to me.

UNIT 8
Read the article. Then choose the best response.

Making Cities Green


1 In attempts to make cities more energy efficient, local governments have been putting up solar panels on
rooftops and adding wind turbines to the waterfront—both to generate energy from “free” sources. These are
not new ideas, but they’re becoming more and more common, and even expected, in any plans to make cities
greener, or less dependent on energy from coal or oil.
The Urban Farmer
2 But there are some new ideas that may seem a bit strange. How about gathering your homegrown vegetables
from your roof? Instead of growing fruits and vegetables in rolling fields, miles away from cities, grow them in
the city—on rooftops, in backyards, on vacant lots. A school in New York City has constructed a small farm on
part of its playground, and the produce is sold during the summer at local farm stands. Across the world, small
urban farms are “sprouting” up to help make the cities green and to feed urban dwellers.
3 There’s even talk of bringing farming inside. One company is testing indoor farming. The test farm consists of
rows of vertical panels filled with potted plants. The whole farm is one-eighth of an acre and is expected to
grow 15 times more lettuce than a traditional farm, but use only 5 percent of the amount of water. There are
many experiments like this one going on in cities.
Build It Up
4 Taking this idea a few steps further, Dickson Despommier, a professor of environmental sciences and
microbiology, believes that one day skyscrapers will be the farms of the future, providing enough food for
entire cities. As he describes the advantages of indoor farming in a recent newspaper article, “You can control
nothing outdoors, and you can control everything indoors.” Crops grown in a controlled environment will not be
threatened by floods, droughts, or storms. In addition, the costs will be greatly lowered because no herbicides or
pesticides would be used, and the food would not need to be transported great distances.

Read the statements. Write T (true) or F (false).


_______1. Few cities expect to use solar or wind power to save energy.
_______2. The number of small urban farms is growing.
_______3. Indoor farming uses more water than outdoor farming.
_______4. Today’s skyscrapers provide enough food for cities.
_______5. Indoor farms are harder to control than outdoor farms.
Read the statements from the text. Choose the inference that can be made from each statement.
6. In attempts to make cities more energy efficient, local governments have been putting solar panels on
rooftops.
A. Local governments have been using solar panels for a long time.
B. Solar panels on rooftops are recent additions to cities.
7. Wind turbines generate energy from “free” sources.
A. Wind turbines do not cost any money.
B. Wind is a free natural resource.
8. A school in New York City has constructed a small farm on part of its playground.
A. The school’s farm is located outdoors.
B. Most schools in New York have a small farm.
9. The costs will be greatly lowered because no herbicides or pesticides would be used.
A. Herbicides and pesticides are expensive.
B. Traditional farms do not use herbicides or pesticides.
10. Food would not need to be transported great distances.
A. Today’s farms are located far from the people who eat the food.
B. Everyone in cities would grow food at their own homes.

Complete the sentence with the correct word from the word bank:
environmentally renewable resource chiefly alternative waste

11. A growing number of businesses are trying to become greener by finding ____________________ ways

to do their work.

12. There are many ways for businesses to reduce the amount of ____________________ they produce.

13. More factories are using energy from ____________________ sources that cannot be used up, such as

wind or solar power.

14. Stores that used to advertise ____________________ by mail are now sending ads via email to use less

paper.

15. Because water is a valuable ____________________, many hotels wash bed sheets less frequently to

avoid using too much of it.

16. As time goes on, businesses will continue to look for ways to be more ____________________ friendly.
Match the sentence with the word that completes it.
17. Small towns offer more space, so people there feel less crowded than people in
A. restore
_____ areas.
B. polluted
18. The city will _____ the park to its original beauty.
C. generate
19. Solar panels _____ more energy on sunny days.
D. traditional
20. Rivers that are _____ have water unsafe for drinking.
E. innovative
21. Growing vegetables on rooftops is one _____ way to save space.
F. urban
22. A _____ farm is outdoors and located away from the city.

Read the sentence. Is the underlined word an adjective or a verb? Choose A or B.


59. She uses recycled paper in her artwork.
A. adjective
B. verb
60. The threatened animals left the forest.
A. adjective
B. verb
61. The city lowered the amount of electricity it used.
A. adjective
B. verb
62. The old factory released a lot of smoke.
A. adjective
B. verb
63. Reducing waste was an expected benefit of the recycling program.
A. adjective
B. verb
64. Indoor farmers often use potted plants to grow vegetables.
A. adjective
B. verb

Rewrite the sentence using the passive voice.


29. More and more people grow rooftop gardens.

_______________________________________________________________

30. The farmer planted twenty rows of vegetables.

_______________________________________________________________

31. Renewable energy will power Masgar.

_______________________________________________________________
32. Local factories polluted the river.

_______________________________________________________________

33. A large wall protected the city.

_______________________________________________________________

34. The high school sells local produce.

_______________________________________________________________

35. The dirty water harmed the fish.

_______________________________________________________________

The 3 Rs
There’s simply too much waste on our planet, and almost all of it is generated by human beings. We need to
decrease the amount of trash we produce. (a) _____.
(b) _____. People throw away millions of plastic water bottles each year, for example. But many of us could
buy just one metal or hard plastic bottle and use it again and again. It is an easy way to reduce the number of
plastic bottles that end up in the trash.
(c) _____. Instead of throwing away old plastic shopping bags, we can find alternative uses for them. For
example, we can reuse them as garbage bags in small wastebaskets or while in our cars. Not only are they
effective, but they are also free.
(d) _____. Many cities have recycling programs for glass, plastic, and paper items. This old “trash” can be
turned into new products with little effort. People can also recycle electronic devices, such as cell phones and
computers. They can be fixed and resold, and they benefit other people and the planet.
(e) _____. Reducing, reusing and recycling waste are simple ideas that can have enormous effects. If everyone
worked together to follow these rules, the world would be a much better place to live.

Complete the problem-solution essay with the sentences below. Write the letter next to the correct
sentence.
_______ 36. People can also decrease waste by reusing items they already have.
_______ 37. One of the easiest ways to help our planet is by reducing the amount of trash we produce.
_______ 38. In all, when people stick to the three Rs, they produce less waste and help keep our planet a cleaner
and healthier place to live.
_______ 39. We can cut down the amount of waste we make if we stick to three basic rules: reduce, reuse, and
recycle.
_______ 40. In addition, recycling can greatly cut down on waste.

UNIT 9
Read the article. Then choose the best response.

How a Goat Led to Graduation


1 Among the proud students receiving diplomas at the 2008 graduation ceremony at Connecticut College was a
young woman from Uganda named Beatrice Biira. And what makes her accomplishment so special is that she
owes it all to a goat.
2 Beatrice was born in 1985 and grew up in the village of Kisinga in the mountains of Uganda. It is an
extremely poor village, and Beatrice, the second oldest of six children, wanted very much to attend school, but
her family didn’t have the money to pay for it. In fact, the family was so poor that there was often not enough to
eat.
3 All of this changed in 1993, when Beatrice was 9 years old, and her mother told her that, through the
generosity of an organization named Heifer, they had received a goat. A goat? At the time, Beatrice could not
see the value of something like a goat, especially when her mother told her that she would be responsible for
caring for the goat.
4 They named the goat Mugisa, which in Lokonzo, Beatrice’s language, means “luck.” And soon Beatrice
realized how her luck would change because of this goat. Mugisa was pregnant when she came to Beatrice’s
family and soon gave birth to two more goats. The milk from the goats helped Beatrice and her siblings to get
healthier, and they were soon able to sell the additional milk. The family earned enough money to send Beatrice
to school.
5 Though Beatrice was much older than the other children in school, she didn’t mind. She breezed through the
early grades as an excellent student. One day in 1995 a study tour, sponsored by Heifer, came to visit Beatrice’s
village. Two women who accompanied the tour, Page McBrier and Lori Lohstoeter, were impressed by
Beatrice’s passion for learning. They were inspired by her story, and decided to write a children’s book about
her. They called it Beatrice’s Goat.
6 Beatrice continued to be an excellent student and won a scholarship to a high school in Kampala, the capital
of Uganda. While Beatrice was a student there in 2001, Beatrice’s Goat was published and became a very
popular New York Times bestseller. While on this trip, she met a woman, Rosalee Sinn, who would become a
great help to her. Ms. Sinn and others helped her obtain a full scholarship to attend the Northfield Mount
Harmon School in Massachusetts, a private preparatory school that had a program to help international students
make the transition to college.

1. Where did Beatrice grow up?


A. in Kisinga
B. in Connecticut
C. in Kampala
2. How did Beatrice feel about the goat at first?
A. She was excited about learning to care for it.
B. She worried it would make her family sick.
C. She did not think it had any value.
3. What does Mugisa mean?
A. milk
B. luck
C. goat
4. What happened to the goat when the family first received it?
A. It got sick.
B. It ran away.
C. It had babies.
5. Who wrote Beatrice’s Goat?
A. Beatrice Biira and her mother
B. Page McBrier and Lori Lohstoeter
C. Rosale Sinn and Beatrice Biira

Match the year with the correct information from the reading.
_____ 6. 2001 A. Beatrice was born.
_____ 7. 2008 B. Beatrice graduated from Connecticut College.
_____ 8. 1995 C. A Heifer study tour visited Beatrice’s village.
_____ 9. 1993 D. Heifer donated a goat to Beatrice’s family.
_____ 10. 1985 E. The book, Beatrice’s Goat, was published.

Match each sentence with the word that completes it.


11. Volunteers hurried to _____ food and water to the people affected by the
hurricane. A. adjustment
12. The students worked all night because they had made a _____ to turn in their B. owe
project the next day. C. attend
13. Moving from a small college to a large university is a big _____. D. commitment
14. I _____ my success to my older sister, who helped me finish school. E. inspire
15. All of my family plan to _____ my graduation ceremony. F. distribute
16. The student read many books hoping they would _____ her to write better.
Complete the sentence with the correct word(s) from the word bank:
selfish positive enhance decrease tend to measure

17. Donating money can have a big ____________________ effect on the lives of those who get the money.

18. For example, just a few dollars a month can provide fresh water to a poor village, which can

____________________ sickness.

19. A small amount of money can feed a poor family for weeks and ____________________ their quality of

life.

20. You can ____________________ how many dollars you have, but you cannot put a number on the

benefits the money provides.

21. Donating money is not a ____________________ act—it is done for others.

22. However, when people donate money, they ____________________ feel happier, too.

Complete the sentence with the correct collocation.


23. His parents taught him the importance of _____ others. A. extraordinary generosity
24. They showed _____ when they donated $1 million to the charity. B. generosity toward
25. The family was able to rebuild their home _____ of many volunteers. C. through the generosity

Complete the sentence with the correct collocation.


26. No matter what others say, I believe in _____. A. with kindness
27. You should treat the people you love _____. B. the kindness of strangers
28. Giving change to a homeless person is _____. C. an act of kindness

Match the dependent clause with the correct independent clause.


_____ 29. Because she earned money from two A. they asked a charity
jobs, organization for help.
_____ 30. Since the villagers needed a new B. she applied for a scholarship.
school, C. she was able to pay for
_____ 31. When the two groups worked together, college.
_____ 32. Because she needed money for college, D. they accomplished a lot more.
Write a complex sentence using the sentences and the word in parentheses. Use a comma where
necessary.
33. There was nobody around to help. I had to do it myself. (since)

_______________________________________________________________

_______________________________________________________________

34. The village is small. Everyone knows each other. (because)

_______________________________________________________________

_______________________________________________________________

35. Students study economics. They learn about money. (when)

_______________________________________________________________

_______________________________________________________________

Random Acts of Kindness


We are usually kind to the people we love. Many people are reluctant to help strangers, however. We should all
try to be more kind to people in need. People should try to do one random act of kindness for a stranger a day.
(a) _____.
(b) _____. It can be something as simple as holding the door open for a stranger. It can be as thoughtful as
telling someone they dropped money on the floor. When I go grocery shopping, I try to find ways to help other
shoppers. (c) _____. When I leave the store, I offer to put back another person’s cart. These are simple things to
do, but they help others. People are usually thankful, because they never expected the help.
(d) _____. Tal Ben-Shahar, a psychology teacher at Harvard University, found that giving help makes people
feel good about themselves. He asked his students to do five small acts of kindness a day, such as giving change
to a homeless person or being nice to waiters at a restaurant. When they did these acts, he found that his
students were happier as a result.
(e) _____. It helps the giver and the receiver both. I have already promised myself to do one act of kindness
each day, and I hope others will, too. It doesn’t take much effort. Every time you go out, just look for little
things you can do for other people, and you will very happy with the results.

Complete the cause/effect essay with the sentences below. Write the letter next to the correct sentence.
_______ 36. Then they can help both other people and themselves.
_______ 37. When someone can’t reach an item, I offer to get it for them.
_______ 38. A random act of kindness can also benefit the person giving help.
_______ 39. A random act of kindness is a small task that helps others.
_______ 40. In all, everyone benefits from a random act of kindness.

UNIT 10
Read the article. Then choose the best response.

Social Networking
1 It is hardly news that people are using the Internet for communicating with others more and more. The growth
of Internet use comes in various forms, from email to IMs to social networking sites, like Facebook, MySpace,
or Bebo. Some fear that people will no longer have the need to talk to one another face-to-face. Some recent
studies suggest that people communicate, or stay in touch, even more than they used to. Whichever side you fall
on, what is clear and not debatable, is that human communication has changed in the 21st century. But results of
recent scientific studies may have us reconsider just how much our new communication forms have changed
our world.
2 In June 2008 a British psychiatrist, Himanshu Tyagi, warned that communicating on social networking sites
could have a negative effect on the generation of people born after 1990. This is a group of people who have
never known a world without the Internet. He warns that they use the Internet to communicate with others so
much they may have trouble forming real relationships with people. It is of concern that people can start and
end relationships over the Internet. As he described the Internet, “It’s a world where everything moves fast and
changes all the time, where relationships are quickly disposed of at the click of a mouse.”
3 Despite all of these warnings, studies can be found that demonstrate the positive aspects of social networking
sites. The results of one study show that using social networking sites has boosted the self-esteem of young
adults. Another suggests that college students using these sites feel more “connected” with people when they
use them. In addition to scientific studies, there are many who feel that the benefits of social networking sites
far outweigh the possible negative aspects. Many people, especially those who spend a lot of time at home, for
various reasons, find the Internet a lifeline. They are able to communicate with many people using these sites.
4 Perhaps the key to this new communication style is moderation and common sense. If someone spends all of
his time on the Internet, and never actually sees friends face-to-face, that seems like an unhealthy way to live.
But if someone spends time on the Internet, some of which is spent arranging meetings for “real” time together,
then it seems almost complementary. Moderation is the key to keeping communication alive and well in the
21st century.

1. What do recent studies suggest about communication today?


A. People communicate more than they used to.
B. People don’t talk to others face-to-face anymore.
C. People use social networking sites more than they email.
2. What is the main idea of the whole article?
A. Facebook, MySpace, and Bebo are popular social networking sites.
B. Social networking sites have changed human communication.
C. Social networking sites make people feel more "connected" to others.
3. What is the main idea of Himanshu Tyagi’s warning in paragraph 2?
A. People born after 1990 use the Internet to communicate with others.
B. The Internet moves fast and it changes all the time.
C. Internet social networking can hurt younger generations.
4. What is the main idea of paragraph 3?
A. Social networking sites are a lifeline for many people.
B. Social networking sites can boost self-esteem.
C. Social networking sites can have benefits.
5. What is the author’s opinion about social networking?
A. People need to use social networking sites in moderation.
B. Spending time on social networking sites is an unhealthy way to live.
C. Many people use social networking sites to communicate with others.

Choose the best response.


6. Who is the audience for the article?
A. Internet users who visit or know about social networking sites
B. psychologists who study the effects of social networking
C. investors who might support new social networking companies
7. What is the author’s purpose for writing the article?
A. to tell a story
B. to entertain the reader
C. to express an opinion

Match the sentence from the text to the correct tone.


_______ 8. Whichever side you fall on, what is clear and not debatable, is that
human communication has changed in the 21st century.
_______ 9. Some recent studies suggest that people communicate, or stay in touch, A. lighthearted
even more than they used to. B. serious
_______ 10. It is hardly news that people are using the Internet for communicating C. informative
with others more and more.
Complete the paragraph with the correct word from the word bank:
demand curiosity restricted feature conventional creative

Children have a natural (11) ____________________ about the world around them, always wanting to find out

the answers to their questions. Many of today’s children look for answers on the Internet, but this creates a

new (12) ____________________ on parents to know whether their children looking in the right places.

Luckily, there are now computer programs that make sure children are (13) ____________________ to safe

websites only. The programs have a (14) ____________________ that lets parents control which websites

children can visit. These programs usually allow access to (15) ____________________ websites for children,

such as those from well-known game companies or those for popular television shows. New and (16)

____________________ websites from unknown companies appear daily, however, so there is a chance that a

bad website still might get past the program. Parents should always be nearby to ensure that their children never

go to the wrong place on the Internet.

Match the underlined word to the correct synonym.


_______ 17. My friend reacted positively when I told her the good news.
_______ 18. It takes several days to deliver a letter, but email delivery is often
A. immediate
instantaneous.
B. cautioned
_______ 19. My computer warned me not to open the email because it might have a
C. uncertain
virus.
D. probably
_______ 20. The Internet is potentially the most significant invention of our time.
E. responded
_______ 21. She wanted to look different, so she shortened her hair.
F. cut
_______ 22. Whether or not social networking affects people negatively is debatable.
Match the underlined prefix with its correct meaning.
_______ 23. bicycle A. before
_______ 24. outdoors B. between
_______ 25. international C. two
_______ 26. unnecessary D. beyond
_______ 27. review E. again
_______ 28. preliminary F. not

Read the statement. Write S (complete sentence) or F (sentence fragment).


_______29. Because there are so many people using social networking sites.
_______30. Although I didn’t like text messaging at first, now I love it.
_______31. When I send a text message to my teenage son, I expect a quick response.
_______32. I use abbreviations like IOU because they are easier and faster.
_______33. When dictionaries began including words like cos and wot.
_______34. Since the Internet has allowed people to come into contact with more and more people.
_______35. After the scientists studied the effects of texting on literacy.

Don’t Make Me Text


It seems like everyone today loves to text. In fact, many teenagers prefer it to a conventional phone call. You
can hear and see people texting all around you—in stores, at work, and even in their cars. (a) _____. I disagree
with them. I much prefer calling my family and friends, because it safer, cheaper, and more personal.
(b) _____. When I need to talk to someone while I’m driving, I always talk over the phone. That way, I can see
what is happening around me. When people text, they have to look down at their phones for a long time, which
takes their eyes off the road. This has caused many accidents. With a traditional phone call, you can avoid these
kinds of accidents.
(c) _____. If people don’t text, they don’t really need a cell phone. They can just use a home phone and only
have one phone bill. People who use their cell phones to text message often have to pay more money for the
service. If they go over their limit, they can be charged a lot of money for the extra texts.
(d) _____. I love getting phone calls from my family because I like hearing their voices. I always recognize my
mother or best friend just by hearing them say “hi” when I answer the phone. That personal touch gets lost
when people text. Everyone’s “hi” looks the same on a cell phone screen. Friends might respond with LOL if I
text something funny, but it is nothing compared to hearing their real laugh.
(e) _____. When people just use a phone, they can avoid car accidents, save money, and have more personal
communication with their family and friends. I do not think texting will ever go away, but maybe soon more
people will long for the good old days of a simple phone call.
Complete the opinion essay with the sentences below. Write the letter next to the correct sentence.
_______36. Although texting is popular, these reasons have shown calling is just as good, and often
better.
_______37. In addition, phone calls are cheaper.
_______38. First of all, phone calls are much less dangerous than texting.
_______39. People who text say that it is an easier way to communicate.
_______40. Another reason I prefer calling to texting is because it is more personal.

Updated

Reading Comprehension Entrance Test


Level: 1-2-3
There are 24 reading comprehension articles with 220 questions. The answer key is given in
the box below the question of each reading task.

1. Tom’s Day

On Sunday, Tom gets up at 10 o'clock. Then he reads his newspaper in the kitchen. He has breakfast
at 11.30 and then he telephones his mother in Scotland.

In the afternoon, at 1.00, Tom plays tennis with his sister and after that, they eat dinner in a
restaurant. At 6.00, Tom swims for one hour and then he goes by bike to his brother´s house. They
talk and listen to music.

Tom watches television in the evening and drinks a glass of warm milk. He goes to bed at 11.30.

1. Where does Tom have breakfast?


a. In the kitchen
b. In his bedroom
c. In the lounge
d. Living room
2. Who does he telephone in the morning?
a. His sister
b. His brother
c. His mother
d. Father
3. Where does his mother live?
a. In England
b. In his brother's house
c. In Scotland
d. Walse
4. What time does he play tennis with his sister?
a. 1pm
b. 2pm
c. 11.30am
d. 3pm
5. How long does Tom swim for?
a. One hour
b. Two hours
c. Six hours
d. Four hours
6. How does Tom go to his brother's house?
a. By bike
b. By car
c. On foot
d. By Train
7. What does Tom drink in the evening?
a. Wine
b. Beer
c. Milk
d. Water
8. What time does Tom go to bed?
a. 9 pm
b. 11pm
c. 11.30pm
d. Midnight

Write your answer here!


1 2 3 4 5 6 7 8

2. My working day

My working day starts very early. From Monday to Friday I get up at half past three and I have a
shower and a cup of coffee. I usually leave the house at ten past four because the car always arrives
a few minutes early. I get to the studio at about five o'clock and start work. My programme Good
Morning Britain starts at seven o'clock and finishes at nine o'clock. Then I leave the studio at a
quarter past ten. After that, I go shopping and visit some friends. I get home at three o'clock. A
woman helps me with the housework and the ironing. I read a newspaper and do some work.

Then my husband gets home at half past five in the evening and I cook dinner. We stay at home in
the evening. We don't go out because I go to bed very early. We usually watch television and then I
go to bed at half past eight, I'm usually asleep by nine o'clock.

I think my job is very interesting but I don't like getting up very early.

1. The person is a woman.


a. True
b. False
c. Doesn’t say
d. Not sure
2. She is a television journalist.
a. True
b. False
3. She drives her car to work.
a. True
b. False
4. She goes home after the programme finishes.
a. True
b. False
c. Doesn’t say
d. Not sure
5. She is in the house alone till her husband arrives home.
a. True
b. False
c. Doesn’t say
d. Not sure
6. She goes to bed early during the week.
a. True
b. False
c. Doesn’t say
d. Not sure
7. She works with her husband.
a. True
b. False
c. Doesn’t say
d. Not sure
8. She likes everything about her job.
a. True
b. False
c. Doesn’t say
d. Not sure

Write your answer here!


1 2 3 4 5 6 7 8

3. Who knows you better, your family or your friends?


In our weekly “test,” single people who are looking for a spouse ask their mother and their best
friend to help. The mother chooses one spouse and the best friend chooses another. The test is to
see who can choose the best spouse.
This week’s single man is Richard Taylor, a 26-year-old musician from Vancouver, Canada. His
mother, Meg, chooses one woman, and his best friend, Danny, chooses another. Then Richard
goes to meet each women. Which woman does he prefer? Who knows him better, his mother or
his best friend? Who chooses the right woman for him?

I usually work in Canada, but sometimes I work abroad, too. When I’m not working, I like going to
the movies and eating in nice restaurants. I don’t like sports very much, and I don’t exercise, but at
least I don’t smoke.

I think I’m open and friendly – I get along well with most people – but I can be kind of shy, too. For
example, I don’t like going to the parties. I prefer to meet friends individually or in small groups.

I like intelligent, funny women who make me laugh, and ideally who love music. Physically, I prefer
women with dark hair who are not taller than me. And I like women who are good listeners.

I’m sure that my friend Danny knows me better than my family because we often talk about
problems we have. I don’t usually talk to my family about that kind of thing. My mom always says
that I look for the wrong kind of woman, but that’s what mothers always say!

Read what Richard says. Mark the sentences T (true) or F (false).


1. He sometimes travels for his job.
a. True
b. False
c. Doesn’t say
d. Not sure
2. He's friendly and outgoing.
a. True
b. False
c. Doesn’t say
d. Not sure
3. He likes music and parties.
a. True
b. False
c. Doesn’t say
d. Not sure
4. He prefers women who are shorter than he is.
a. True
b. False
c. Doesn’t say
d. Not sure
5. He likes women who talk a lot.
a. True
b. False
c. Doesn’t say
d. Not sure
6. He doesn't talk to his family about women.
a. True
b. False
c. Doesn’t say
93 | P a g e
d. Not sure
7. His mother doesn't think he's good at choosing a good wife.
a. True
b. False
c. Doesn’t say
d. Not sure
1 2 3 4 5 6 7

4. Mountain Climbers recued by text message


Two British climbers were rescued yesterday after sending an SOS text message to a friend in
London
Rachel Kelsey, 34, and her partner, Jeremy Colenso, 33, who are both experienced climbers, were
on a climbing holiday last week in Switzerland. But on Saturday night, when they were 3,000
metres up in the mountains, there was a terrible storm. The wind was incredibly strong and the
snow was two metres deep. They couldn’t move and they had to spend the night on the mountain.
Rachel had her mobile phone with her, so she sent a text message to five friends in the UK asking
for asking for help
About four hours later, one of her friends, Avery Cunliffe in London, replied with a text message.
Avery said, “I don’t usually have my mobile in the bedroom. At about 4 a.m. I woke up and I saw
Rachel’s text message. I found the number of the police in Switzerland and phoned them. They
contacted the mountain rescue team
But the weather was so bad that the helicopter couldn’t get to the two climbers. The rescue team
sent a text message to Rachel and Jeremy telling them that they had to spend a second night on
the mountain. The message said: “So sorry Rachel. We tried. Wind too strong. Have to wait till
morning. Take care. Be strong” Rachel said, “I thought we were going to die. It was freezing, -15ºC,
and really windy on the mountain. We spent the night talking and planning a holiday in the sun
Next morning the storm passed and the helicopter arrived to take them off the mountain. Avery
said, “When I heard that Rachel and Jeremy were safe I was dancing around my flat.” Rachel said
that she and Jeremy were now looking forward to a hot bath and a good meal. But first they were
going to buy a bottle of whisky.
All the following statements below are true (T), false (F) or doesn’t say (DS)
1. Rachel and Jeremy were climbing for the First time.
a. True
b. False
c. Doesn’t say
d. Not sure
2. They couldn't go down the mountain because of bad weather.
a. True
b. False
c. Doesn’t say
d. Not sure
3. Avery was sleeping when Rachel's message came.
a. True
b. False
c. Doesn’t say
d. Not sure

94 | P a g e
4. He called the mountain rescue team in Switzerland.
a. True
b. False
c. Doesn’t say
d. Not sure
5. Rachel's other four friends didn't reply.
a. True
b. False
c. Doesn’t say
d. Not sure
6. The helicopter couldn't rescue them immediately.
a. True
b. False
c. Doesn’t say
d. Not sure
7. Rachel and Jeremy spent three nights on the mountain.
a. True
b. False
c. Doesn’t say
d. Not sure
8. They wanted to buy Avery a bottle of whisky.
a. True
b. False
c. Doesn’t say
d. Not sure

1 2 3 4 5 6 7 8

5. English Town in China


The building company Hong Kong Housing is building a new suburb in Shanghai, a city with a
population of more than 15 million people. But this suburb is not like other parts of Shanghai. It is a
little piece of England. The name of the new area is "English Town". The manager of Hong Kong
Housing, Shi Guangsheng, says many people in this area of China want to buy an English house
and live in an English town.

"This town is like Bristol or Chester. There is an English square and there are pigeons to feed like
in Trafalgar Square, London. There are four English-style pubs and you can buy English beer to
drink. There is a canal and you can row a boat there like in Cambridge."

Are there any cricket fields? "No, there aren't any places to play cricket because nobody in
Shanghai can play this game. But there is a football stadium like those in England. And the thing
we are very proud of is our shopping street. There are several shops where you can buy traditional
English food like fish and chips or Christmas Pudding."

The rich Shanghai businessman can do some gardening too. For with every house in "English
Town", there is an English lawn and there are long gardens with ponds and tall hedges.
95 | P a g e
But there is one problem. The houses in this part of town cost a minimum of $400,000!

Choose the best answer for the following statements below.


1. "English Town" is in a Chinese city.
a. True
b. False
c. Doesn’t say
d. Not sure
2. These houses are only for English people to buy.
a. True
b. False
c. Doesn’t say
d. Not sure
3. There are pigeons in all the gardens.
a. True
b. False
c. Doesn’t say
d. Not sure
4. The English pubs look like pubs in Cambridge.
a. True
b. False
c. Doesn’t say
d. Not sure
5. There aren't any cricket fields in the new development.
a. True
b. False
c. Doesn’t say
d. Not sure
6. Shi Guangsheng feels proud about the English-style shops.
a. True
b. False
c. Doesn’t say
d. Not sure
7. Most houses have a lawn.
a. True
b. False
c. Doesn’t say
d. Not sure
8. The houses are cheap.
a. True
b. False
c. Doesn’t say
d. Not sure

96 | P a g e
1 2 3 4 5 6 7 8

6. Black Hollywood
Patricia Denning stars in a sad movie about ambition and heartbreak. When Cecilia leaves college
in Texas, she has everything: a wonderful boyfriend, Carl, an interesting job and an expensive
house in a pretty town. Her life is perfect. Even her black Labrador dog, Perky, is a star: he wins
the local dog competition and Hollywood wants to put him in a movie.

Cecilia goes to Hollywood with him and leaves her boyfriend alone back home. When the studio
fires Perky, Cecilia decides to stay in Hollywood and tries to get other movie roles for her beautiful
dog.

Carl gives Cecilia an ultimatum: come home and stop behaving crazily, or stay there forever.
Cecilia decides to stay in Hollywood and wants to become an actress herself. She sends Perky
back to Texas and starts an acting course. After six months, Cecilia realizes she can't act and she
misses home, but it's too late. Carl has found another girlfriend. Cecilia tries to win him back, and
also discovers a lot about herself on her long journey.

This is a very sad movie, which makes you think about what we want from life. Patricia Denning is
very convincing as Cecilia, but the star of the movie is gorgeous Perky, the black Labrador who
steals our hearts. Black Hollywood is in cinemas from next week.
1. What does Cecilia not have?
a. A house she love
b. A wonderful husband
c. A pet
d. A great job
2. When does her wonderful adventure begin?
a. When her dog wins a competition
b. When she marries her boyfriend, Carl
c. When she goes on vacation to Hollywood
d. When she buys the house in a pretty town
3. What happens to Perky in Hollywood?
a. He isn't very successful
b. He becomes a star
c. He's too nervous to work
d. He makes two movies and retires
4. What is Carl's reaction to Cecilia living in Hollywood?
a. He doesn't know she went
b. He's angry
c. He doesn't care
d. He's supportive
5. Why does Cecilia stay in Hollywood?
a. She wants to become an actress
b. She is too busy making movies
c. She hates Carl

97 | P a g e
d. She has no money
1 2 3 4 5

7. London Marathon Runners

Emmanuel Mutai and Mary Keitany completed a Kenyan double as they won the men's and
women's races at the 201 London Marathon.

Mutai raced to a London record of two hours four minutes and 39 seconds, while Keitany won in
2:19.19, the fourth fastest women's marathon ever.

Russian defending champion Liliya Shobukhova was the women's runner-up.

Mutai told BBC Sport: "My dreams have come true, the day has come for me. My aim was just to
win the race, I was not focusing on the time.

"I'm happy with the result, since I came to run in London, I was fourth twice and second once. It's
a fantastic achievement for me personally."

Ethopian Aberu Kebede, last year's winner of the men's race, could only manage a ninth placed
finish.

Great Britain's Jo Pavey came through in a time of 2:28.24, the seventh fastest time by a British
female athlete and inside the British Olympic qualifying time.

Team-mate Louise Damen, the second British athlete across the line, was also inside the
qualifying time in 2:30.00.

Now decide if the below statements are true or false:


1. Emmanuel Mutai broke the record for the fastest running time for the London marathon.
a. False
b. True
c. Doesn’t say
d. Not sure
2. The female winner finished the race before the male winner.
a. False
b. True
c. Doesn’t say
d. Not sure
3. The female runner up is extremely disappointed that she didn't win.
a. False
b. True
c. Doesn’t say
d. Not sure
4. The winner of last year's London Marathon came within the top five.

98 | P a g e
a. False
b. True
c. Doesn’t say
d. Not sure
5. Two female British athletes have qualified for the Olympics.
a. False
b. True
c. Doesn’t say
d. Not sure
1 2 3 4 5

8. The Parliament of the United Kingdom

The UK Parliament is the home of the UK government. The main functions of parliament
are to pass laws, to finance through taxation the work of government, to scrutinize government
policy and administration, including proposals for expenditure, and to debate the major issues of
the day.

General Elections in the UK

In a general election, every area in the country votes for one Member of Parliament (MP)
to represent them in the House of Commons. There are 650 geographical areas,
called constituencies. To be eligible to vote in a general election you must be registered to vote
and aged 18 or over.

Each eligible voter has one vote in their local constituency. The candidate with the most
votes wins a seat in parliament and becomes the MP for that area. This voting system is called
'first past the post'. Usually the political party with the most MPs then forms the government –
though if there is no overall winner, a hung parliament may result. Two or more parties with a
combined majority of MPs may form a coalition government.

There has to be a general election at least every five years. The Prime Minister decides
when to call an election. The last general election was held on 7 May 2015. The next general
election will be held on 8 June 2017. If an MP dies or resigns between elections, there is a by-
election in their constituency.

1. How many parliamentary constituencies are there in the UK?


a. 560
b. 365
c. 650
d. 570
2. How often are general elections usually held in the UK?
a. about every seven years
b. about every five years
c. about every six years

99 | P a g e
d. about every ninth year
3. What type of voting system does the UK currently have?
a. proportional representation
b. first past the post
c. second ballot
d. third round
4. How old do you have to be to vote in a UK election?
a. 21
b. 16
c. 18
d. 32
5. Which party usually forms the government after a UK general election?
a. the party with the most MP’s
b. the party with the most candidates
c. the parties with the most votes
d. the party with the least candidates

6. What is a “hung parliament”?


a. a parliament where there is no overall winner in a general election
b. a parliament that is between governments pending an election result
c. a parliament that have been suspend
d. a parliament with no general election
7. What does the acronym MP stand for?
1. Member of Politics
2. Man of the people
3. Member of Parliament
4. Member of the election committee
8. When do by-elections usually take place?
a. about every five years
b. about every three years
c. when an MP dies or resigns between elections
d. about every ten years

1 2 3 4 5 6 7 8

9. STARTING AT UNIVERSITY
The first step towards becoming a doctor was easy for Maria. In a short time she had completed a
pre-medical program at the University of Rome and passed her exam. Then she was ready to begin
studying at the university’s department of clinical medicine. However, the university was not ready for
her. There had never been a woman in the department and the professor did not want one.

No one knows how Maria managed to convince the university to accept her. She spoke with the head
of medicine faculty, Professor Baccelli, who was also an important politician. He wanted to reform the
Italian school system and the university, but he did not want women to study medicine. So he was no
help. There were reports that she also spoke with Pope Leo XIII and that he helped her. However,
this story may not be true. It may be one of the many legends that later surrounded Maria.

In any case, in 1892, Maria began to attend classes in clinical medicine. She was not received well by
fellow students. They laughed at her and made unpleasant comments to her about women doctors.

100 | P a g e
The presence of a woman in their classes, especially an intelligent woman like Maria, clearly
disturbed them. They did not like the fact that she received better grades than many of her male
classmates. And they also did not like the way she took her studies so seriously. In fact, many
medical students at that time cared only about improving their social position. They were not
interested in learning about medicine or in working as doctors.

Maria, however, felt very differently about her studies. She was interested, first of all in the subjects
themselves. Then she wanted to lead a useful life too. She believed she could do this best as a
doctor. With her beliefs to strengthen her, she paid no attention to other students. If someone was
rude to her at the university, she just stared back at him. In time, her fellow classmates came to
respect her strong nerves as well as her ability.

Only once did Maria have any doubts about her career. That was when she began to attend anatomy
classes. These classes came to a terrible shock to her. Women in those days were not used to
looking or talking about people’s body parts. Both women and men, in fact, wore clothes that almost
completely covered their bodies. Women, especially, were never supposed to talk about or even think
about bodily functions.

So Maria was extremely embarrassed the first time she had to examine a naked body and she also
hatred the idea of working with dead bodies. When she went home after the first anatomy lesson, she
felt so awful that she got sick to her stomach. She later wrote that she was ready at the point to quit
studying medicine.

But she did not quit. The next day she went back. Somehow she managed to control her feeling and
continue.

1. This passage is about


a. how Italian study medicine.
b. Maria’s feeling about the university.
c. Maria’s first experience at the university.
d. How Maria got accepted to the university.
2. Some people thought that Pope Leo XIII
a. helped Maria with her study.
b. wrote newspaper article about Maria.
c. wanted to reform the university system.
d. helped Maria got accepted into medical school.
3. Many of Maria’s fellow classmates
a. liked having a woman in their classes.
b. did not like having a woman in their classes.
c. were interested in a woman like Maria.
d. wanted Maria to become a doctor.
4. As a student, Maria
a. performed less well than her fellow classmates.
b. got the same grades as her fellow classmates.
c. performed better than many of her fellow classmates.
d. got lower grades than most of her fellow classmates.
5. You can infer from this passage that doctor in Italy
a. had a high social status.
b. were laughed by the public.

101 | P a g e
c. had a lower social status.
d. were often very religious.
6. Maria
a. did not take her study very seriously.
b. was too upset to study seriously.
c. took her study very seriously.
d. was only interested in her social status.
7. At first Maria found that anatomy class was very
a. interesting
b. upsetting
c. easy
d. amusing
8. From this passage it seems that Maria
a. often doubted her decision to become a doctor.
b. never doubted her decision to become a doctor.
c. was very uncertain about her decision to become a doctor.
d. rarely doubted her decision to become a doctor.
1 2 3 4 5 6 7 8

10. The Population Explosion


The population of the earth is growing very fast. In 1950, it was 2.5 billion. By 1992, it has jumped to
5.5 billion. By the year 2050, it will probably reach 10 billion. The worldwide trends are clearly towards
rapid population growth. However, it is not happening in all parts of the world.
The population of develop countries has almost stopped growing. But in less developed
countries, it continues to grow at a very fast rate. Every year about 97 million people are added to the
world population. About 90 % of these are in less-developed countries.
The reason for this difference in population growth lies in the birth rate. (The birth rate is the
average number of children per woman of child-bearing age.) Population increases or decrease
according to the birth rate. When the birth rate is over 2.0, the population grows. When it is less than
2.0, the population decreases.
In industrialized countries, a very low birth rate has caused population growth to slow down or
stop altogether. Italy has the slowest birth in the world-only 1.3. In most European countries and in
Japan, the birth rate is in fewer than 2.0. The birth rate in the United States is just over 2.0.
While birth rate has been declining in these countries, life expectancy has been increasing.
(Life expectancy is the average length of a person’s life.) In almost in the industrialized countries, life
expectancy is now well over 70 years old. This means that the percentage of older people in the
population is increasing. In Italy, for example, one quarter of all Italian was over 75 years old by the
year 2015.
In less developing countries, the situation is completely different. The birth rate in many places
is extremely high. It is over 7.0 in many African countries and as high as 8.3 in Rwanda. At the same
time, life expectancy in these countries is very low. For example, the life expectancy of an Ethiopian
is less than 40 years old. The population, on average, is very young, which means a high percentage
of women of child-bearing age. Thus, even birth rate decline, the population will continue to increase
for many years.
102 | P a g e
Rapid population growth also partly explains why less-developed countries remain much
poorer than industrialized countries. There is already an enormous difference in wealth. The average
annual income per person in industrialized countries is about $ 16,500. In less-developed countries, it
is only about $ 750. As the population increases, so do the problems. Crowding on the land means
that water, food, and fire wood become even more difficult to find. Hunger and disease kill millions,
especially children. And, as a result, people flood to the cities, or to other countries to look for a better
life.
Thus, the population explosion is another global problem that needs a global solution.
Choose the best answer and write it in the boxes given below.
1. This passage is about
a. birth rate around the world
b. population growth in less-developed countries.
c. Life expectancy in industrialized countries
d. population growth in different parts of the world
2. Population growth is
a. slowing down around the world
b. happening everywhere at the same rate
c. slowest in less developed-countries
d. fastest in less-developed countries
3. Population generally grows fastest when
a. the birth rate decline
b. average income increase
c. life expectancy increase
d. the birth rate increase
4. The birth rate in industrialized countries is
a. about 1.3
b. generally high
c. rapidly increase
d. around 2.0 or less
5. We can infer from this passage that life expectancy in Italy
a. will decline in a few years
b. is over 70 years
c. is lower than in Japan
d. is under 70 years
6. In some less-developed countries, the birth rate is
a. lower than in industrialized countries
b. slowly increasing
c. generally around 2.0
d. as high as 8.0
7. The population in less-developed countries is very young because of a
a. high life expectancy and low birth rate
b. low life expectancy and high birth rate
c. low average annual income
d. high percentage of women of child bearing age
8. Many less-developed countries stay poor because of
a. their low annual income
b. the crowded cities

103 | P a g e
c. hunger and disease
d. the rapidly increasing population
1 2 3 4 5 6 7 8

11. Cutting Down the Forest


There is nothing new about people cutting down trees. In ancient times, Greece, Italy, and
Great Britain were covered with forests. Over the centuries those forests were gradually cut back,
until now almost nothing is left.
Today, however, trees are being cut down far more rapidly. Each year, about 42 million acres
of forests are cut down. That is more than equal to the area of the whole of Great Britain. While there
are important reasons of cutting down trees, there are also dangerous consequences for life on earth.
A major cause of the present destruction is the world-wide demand for wood. In industrialized
countries, people are using more and more wood for paper, furniture, and house. There is not enough
wood in these countries to satisfy the demand. Wood companies, therefore, have begun taking wood
from the forests of Asia, Africa, and even South American.
Wood is also in great demand as firewood in developing countries. In many areas, people
depend on wood to cook their food. As the population grows, the need for wood grows, too. But when
too many trees are cut at once, forests are destroyed. A future of source of wood is destroyed as well.
When some trees in the forest are left standing, the forest can grow back, but only if it is not cut again
for at least 100 years. In reality, it usually has no chance to grow back. Small farmers who are
desperate for land move in. they cut down the trees and burn them. In this way, many millions of
acres of forests are destroyed every year. Unfortunately, the forest soil is not good for growing food.
Thus, these poor farmers remain as poor and desperate as before. They also lost the resource of the
forest.
However, the poor and desperate people are not the only ones to cut and burn the forests. In
Brazil and Central America, large landowners want to raise lots of cattle for export. They put too many
cattle on to little land. When that land has been ruined, they burn parts of the forests. Then they move
the cattle into the forest land. This may both land and forests are destroyed.
The destruction of the forests affects first the people who used to live there. However, it also
has other effects far away. For example, on the mountainsides, trees help to absorb heavy rains.
When the trees are cut down, the rains pours all at once into the rivers and there are terrible floods
downstream. This has happened to the Ganges, the Mekong, and other major rivers in Asia.
But finally, the loss of forests may have an effect on the climate of our planet. Together with
increasing pollution, it could cause temperatures to rise and the climate to change around the world.
No one knows exactly what effect this would have on our lives. For many people, however, the effects
would probably be destructive.
Circle the best answer of the following stems.
1. This passage is about
a. the tropical forests.
b. how landowners cut down trees in Brazil.
c. why are people cutting down so many trees.
d. the destruction of forests.
2. Greece, Italy, and Great Britain
a. are all covered by forests.
b. used to be covered by forests.
c. have growing population.
d. never had any forests.
3. These days, forests are being cut down in

104 | P a g e
a. Asia, South Africa, and America.
b. Greece, Italy, and Great Britain.
c. Asia, Africa, and South America.
d. the industrialized countries.
4. The demand for wood in industrialized countries
a. is one cause of the problems’ destruction.
b. is helping to save the forests.
c. is easily satisfied by the United States of America.
d. has declined in recent years.
5. We can infer from this passage that poor people generally
a. do not benefit from cutting down the forests.
b. make a lot of money from the forests.
c. do not like living in the forest.
d. want the forests to grow back again.
6. Large landowners in Brazil and Central America
a. use the forest for hunting.
b. worry about cutting down the forests.
c. destroy forests to raise beef cattle.
d. use a lot of wood to build their house.
7. When trees are cutting down on mountainsides
a. the wood is usually in poor quality.
b. there are floods downriver.
c. they grow back quickly.
d. cattle come to eat the grass.
8. The cutting down of the forests
a. could improve the climate.
b. could cause heavy rain.
c. could affect the climate.
d. has no effect on people in cities.
1 2 3 4 5 6 7 8

12. Global thinking in the 21st century


At the end of 20th century, the world is changing in important ways. Until recently nations acted
independently. Each country did its business and tried to solve its problem alone. But now the
economy is worldwide and communications technologies have connected people all over the globe.
Many problems are global, too, and can no longer be solved by individual nations.

Environment destruction is one of these problems. As the world population has grown and
technology has developed, the environment has suffered. Some nations have begun to try to stop the
pollution and the environmental destruction. But the environment is global – the atmosphere, the
ocean, and many forms of life are all connected. Thus, the solutions require global thinking.

The problem of ocean pollution is a good example. All the oceans of the world are connected.
Pollution does not stay where it begins. It spreads out from every river and every harbor and affects
bodies of water everywhere.

For century, people have used the ocean as a dumping place. Many cities take tons of
garbage out to sea and dump it there. The quantity of garbage that ends up in the water is incredible.

105 | P a g e
Five million plastic containers are thrown into the world’s oceans every day! Aside from plastics,
many other dangerous substances are dumped in oceans. These include human waste and chemical
used in agriculture. And every year, oil tankers accidentally spill millions of gallons of oil into the sea.

Some people believe that the oceans are so large that chemicals and waste will disappear.
However, many things, such as chemicals and plastics, stay in the water and create problems. They
eventually float to shore and are eaten by tiny sea creatures. Then the large animals that eat the tiny
creatures are poisoned and died. Harbors and coasts around the world have become unsafe for
humans and animals. The world’s fish populations are rapidly shrinking.

Another global pollution concerns the atmosphere. Until recently, chlorofluorocarbons (CFCs)
were used around the world in manufacturing refrigerators. Scientists discovered that these CFCs
were destroying the ozone layer in the atmosphere. The ozone layer helps protect the earth from the
sun’s ray. Without this layer, most forms of life on earth – including humans – probably would not be
able to live.

CFCs will soon be completely banned in the United States and in most developed countries.
But many other countries still use CFCs in manufacturing. Among these countries are some of the
most populous on earth, such as India and China. These countries need to change their refrigerator
factories to non-CFCs process. But they may not be able to make this change a lone. They will need
help from industrialized countries. This is what global thinking means – working together for solutions.

Choose the best answer for the following statements below.


1. This passage is about
a. air pollution
b. old ways of thinking
c. global thinking
d. refrigerator in China
2. You may infer from this passage that in the past
a. problems were more local
b. nations were more interested in the environment
c. the economy was more global
d. individual nations did not have as many problems
3. Until now, most nations followed the principal that
a. all countries should share their problems
b. all nations were enemies
c. each nation should take care of its own problem
d. what happens in China affects everyone
4. According to this passage, many environmental problems
a. are caused by global thinking
b. are caused by old ways of thinking
c. are caused by the United States and Japanese
d. cannot be solved by local law
5. The solution to ocean pollution requires global thinking because
a. no one cares about the oceans
b. the oceans are so large
c. all the world’s oceans are connected
d. more oil is needed in developed countries
6. Increasing global population and the development of technology

106 | P a g e
a. will solve the problem of ocean pollution
b. have made the pollution problem worse
c. is a result of global thinking
d. have resulted in more business opportunities
7. From this article, we can infer that, in manufacturing refrigerators,
a. CFCs must be used around the world
b. only China uses CFCs
c. other chemical can be used in place of CFCs
d. only CFCs are available in China
8. Nations must learn to think globally because
a. communication technologies will solve our problems
b. large business will spread around the world
c. changes in the world require stronger nations
d. that is the only way to solve global problems.

1 2 3 4 5 6 7 8

13. Visit Angkor Wat


Siem Reap is a small town near the world famous temple of Angkor Wat. The town is charming and
worth exploring, with some fine examples of Khmer and French colonial architecture set among the
more modern developments. Nowadays, visitors are flocking in, using it as a base for visits to the
nearby temples.

A Carved City
From the 9th to the 14th centuries, when Europe was still struggling out of the Dark Ages, the
Cambodian Empire of Angkor covered most of present-day Cambodia, Laos, Vietnam, and Thailand.
The heart of this empire during the 12th century was the ancient capital of Angkor Thom, near
present day Siem Reap, the site of the world’s largest temple complexes, which were rediscovered in
1861.This spectacular city was built over 30 years under the reign of King Suryavarman II (1113-
1150). The area covers about 400 square kilometres and is full of the finest examples of Khmer art
and architecture. Tourists are always amazed at the scale of the place.
In Angkor Wat you will find more than 100 stone monuments and temple buildings, each of which
contains countless statues, sculptures and reliefs that have weathered extremely little over the last
800 years. To see the whole thing can take several days. The most important temples to visit in the
area are Angkor Wat, especially at sunrise or sunset; Angkor Thom, the remains of the capital; Ta
Prohm, a palace overgrown by jungle; and Bayon.

Getting a visa
Visas are required to enter Cambodia. You can obtain one on arrival at Siem Reap International
Airport for $20, and 1 passport photo is required per person. You will also need another passport
photo for the Angkor Temple Entrance Pass. Please ensure you take comfortable walking shoes, light
clothing and plenty of water to drink as it is very hot there. The most commonly accepted currency in
Cambodia is the US dollar.

Comprehension Questions...
1. Most people visit Siem Reap because of the temples in the area.
a. True
b. False
107 | P a g e
c. Does say
d. Not sure
2. In the 9th to the 14th centuries, Europe was more advanced than Cambodia.
a. True
b. False
c. Does say
d. Not sure
3. Angkor Wat was the capital of the Cambodian empire.
a. True
b. False
c. Does say
d. Not sure
4. It took many decades to build the capital.
a. True
b. False
c. Does say
d. Not sure
5. The stonework of Angkor Wat is in poor condition today.
a. True
b. False
c. Does say
d. Not sure
6. It'll take a couple of days to see everything.
a. True
b. False
c. Does say
d. Not sure
7. Dawn and dusk are particularly good times to visit Angkor Wat.
a. True
b. False
c. Does say
d. Not sure
8. You must get a visa before flying to Siem Reap.
a. True
b. False
c. Does say
d. Not sure
9. You will need a couple of passport photos.
a. True
b. False
c. Does say
d. Not sure
10. The US dollar is widely accepted in Cambodia.
108 | P a g e
a. True
b. False
c. Does say
d. Not sure

1 2 3 4 5 6 7 8 9 10

14. Scottish Independence

The majority of people in Scotland are in favour breaking away from the rest of the UK and becoming
independent, according to a poll taken just before the 300th anniversary of the Act of Union, which
united Scotland and England.

A pair of Acts of Parliament, passed in 1706 and 1707 that came into effect on May 1, 1707, created
Great Britain. The parliaments of both countries were dissolved, and replaced by a new Parliament of
Great Britain in Westminster, London.

The poll showed support for independence for Scotland is running at 51%. This is the first time since
1998 that support for separation has passed 50%, and the first time since devolution gave power to
the country in 1999. Six months before elections for the Scottish Parliament, these poll results come
as good news to the Scottish Nationalist Party, who are hoping to make progress against Labour and
further the cause of an independent Scotland.

Many people have become disillusioned with devolution, and believe that the Scottish Parliament has
failed to deliver what they had hoped it would; only a tenth have no opinion. In fact, only 39% of those
polled want to keep things as they are.

109 | P a g e
1. Scotland and England .
a. have always been united.
b. want to break up the union.
c. have been united for a long time.
d. were united by war.
2. Great Britain .
a. was formed by an Act of Parliament in 1706.
b. was formed by two Acts of Parliament in 1707.
c. was formed by an Act of Parliament that came into effect on May 1st 1707.
d. was formed by Acts of Parliament that came into effect on May 1st 1707.
3. People who want independence for Scotland
a. are the vast majority.
b. are in the minority.
c. are the slight majority.
d. have decreased in number since devolution.
4. The majority of people wanted independence for the first time
a. before devolution.
b. in 1999.
c. after devolution.
d. before and after independence.
5. The results of the poll are good news
a. for Labour.
b. for both parties.
c. for the Scottish Nationalist Party.
d. for devolution.
6. Most people's opinions of devolution
a. have gone up.
b. have gone down.
c. are the same.
d. make progress against Labour.
7. The number of people who want to keep things as they are
a. greater than those that don't know.
b. smaller than those that don't know.
c. increasing.
d. the majority.

Write your answer here!


1 2 3 4 5 6 7

110 | P a g e
15. NARESUAN UNIVERSITY TRIES TO SOLVE A MAJOR PROBLEM
Naresuan university has become Thailand’s first “solar city” .Its 22,000 students and professors eat
sweet bananas dried in a solar oven, use powered public phones, and enjoy free rides on Thailand’s
first solar-powered mini-buses.
At the moment, the 1100-hectare campus still has about 13,000 cars or motorbikes moving around
each day. That amounts to some 200,000 gallons of petrol a year, and is the cause of serious air and
noise pollution that is reducing the quality of life at Naresuan.
Determined to promote the use of renewable energy, the university has begun a “green” campaign to
encourage students and staff to quit using regular vehicles around campus and use the free, green-
friendly buses instead.
“Sunlight is an excellent energy source. In this energy-saving world, solar-powered vehicles help save
the planet,’ said the university president. ‘I believe these clean, free vehicles are ideal for this huge
campus and its huge population,’ he added.
With the campus population expected to increase to 30,000 in the next five years, administrators are
determined to cut down on the threat of all the added pollution, noise, energy wastage, accidents and
traffic jams that those extra students will bring with them when they eventually enroll at the eventually
enroll at the university . The key to this is using ‘green’ buses.
Hundreds of students have been injured or killed in motorbike accidents over the last ten years. And
we have suffered a noisy, polluted environment at the university. But we can keep our environment
clean and green by using these buses’ said the president.
‘The key lies in harnessing the sun’s energy to generate the electricity to charge the batteries that power
the mini-buses,’ he said. ‘Normal electric vehicles receive electricity from power plants, which pollute
the environment,’ he explained. ‘But we can produce all the energy we need, thanks to the “solar plant”
we have made here on campus,’ said the president.
Forty –eight of the buses are in action on campus. It takes about one month to assemble one of the
attractive-looking vehicles, each of which can carry up to 20 passengers. The buses are free, travel at
a maximum speed of a very careful 20 kilometers an hour, and run every ten minutes from 6.30am to
midnight, every day Monday to Sunday, on routes around the campus.
‘But,’ the president added, ‘most students refuse to use the new, free buses. They don’t want to wait at
a bus stop. They prefer to create noise and pollution rather than have to wait for the next bus. But our
serous environmental problems must be addressed immediately.’

A. Choose the best answer for the following


1. Narasuen University is taking the lead in Thailand in using the sun as a local energy source.
a. True
b. False
c. Doesn’t say
d. Not sure
2. The university has major environmental problems now.
a. True
b. False
c. Doesn’t say
d. Not sure
3. Narasuen University provides buses with cheap fares for students to move around the campus.
a. True
b. False
c. Doesn’t say
d. Not sure
4. The school’s president thinks that solar-powered buses should be used for road transport in the
major cities in Thailand.
111 | P a g e
a. True
b. False
c. Doesn’t say
d. Not sure
5. The current campus population is expected to increase by 8,000 in the next five years.
a. True
b. False
c. Doesn’t say
d. Not sure
6. The university administrators think it is impossible to solve Narasuen’s environment problems.
a. True
b. False
c. Doesn’t say
d. Not sure
Write your answer here
1 2 3 4 5 6

B. Choose the best answer for the following


1. The university president thinks that the buses will at Narasuen.
A. add to the number of road crashes
B. reduce the number of road crashes
C. make the environment noisier
D. increase pollution
2. Narasuen’s solar plant
A. is based in the city.
B. produces more energy than a power plant.
C. can fill Narasuen’s energy requirements.
D. is more expensive than a power plant.
3. The buses
A. are crowded.
B. number three dozen in total.
C. operate 24-hours a day.
D. are safe, regular and run daily.
4. The buses are used by of the students.
A. all
B. most
C. some
D. none
5. Most of the students…..
A. are happy to wait for a bus.
B. spend up to an hour waiting for a bus.
C. want the environmental problems solved immediately.
D. remain selfish.
6. Another suitable heading for this passage would be:
A. A ’Green’ Solution.
B. Bananas, Phones and Buses.
C. How To Prevent Traffic Jams.

112 | P a g e
D. A Popular Solution.
Write your answer here
1 2 3 4 5 6

16. Instructions: Read through the text below, and choose the suitable word to complete in the
space.

Child Labour

Child workers, some as young as 10, have been found working in a textile 1) __________ in
conditions described as close to slavery to produce clothes that appear destined for one the major
high street 2) __________.

Speaking to a British newspaper, the children described long hours of 3) __________ work and
threats and beatings. The company said it was unaware that clothing intended for its 4)
__________ had been improperly 5) __________ to a 6) __________ that used child labour. It further
announced it had withdrawn the garments involved until it had investigated the alleged 7)
__________ of the 8) __________ code it imposed on manufacturers three years ago.

The discovery of these children working in appalling conditions in the Shahpur Jat area of Delhi has
renewed concerns about the 9) __________ by some large retail chains of their 10)
__________ production to India, recognised by the United Nations as one of the worlds's hotspots for
child labour. According 11) __________ one 12) __________, over 20 per cent of India's economy
is 13) __________ on children, which comes to a total of 55 million youngsters under 14 working.

14) __________ in the West should not only be demanding answers from retailers about how
their 15) __________are produced but also should be looking into their consciences at how they
spend their money and whether cheap prices in the West are worth the suffering caused to so many
children.

Comprehension Questions...

1. a. facility b. factory c. office d. bureau


2. a. Warehouse b. Retailer c. Warehouses d. Retailers
3. a. Inpaid b. Unpaid c. without pay d. without payment
4. a. Warehouses b. Stores c. Outlet d. Branch
5. a. Outsource b. Outsourcing c. Outsources d. outsourced
6. a. Association b. Sweatshop c. closed shop d. retailer
7. a. Breaches b. Errors c. Mistakes d. wrongdoings
8. a. Ethic b. Ethnic c. Ethical d. ethnical
9. a. Outsource b. Outsourcing c. Outsources d. outsourced
10. a. Garment b. Raiment c. Garments d. raiments
11. a. By b. To c. Of d. From
12. a. Estimate b. Estimating c. Estimates d. estimated
13. a. Depends b. Dependent c. Dependant d. dependence
14. a. Consume b. Consumption c. Consumer d. Consumers
15. a. Stuff b. Ware c. Goods d. Garment

Write your answers here!

113 | P a g e
1 2 3 4 5 6 7 8 9 10 11 12 13 14 15

17. The Banker to the Poor


Muhammad Yunus is a Bangladeshi banker and economist. 1) __________ a professor of
economics, he is most famous for his successful application of the concept of microcredit, the 2)
__________ of small loans to entrepreneurs too poor to qualify for traditional bank loans. He is
the 3) __________ of Grameen Bank, and he and the bank were 4) __________ awarded the
Nobel Peace Prize for their efforts to create economic and social development among the poor.

During visits to the poorest households in the village of Jobra near Chittagong University, Yunus
discovered that very small loans could make a 5) __________ difference to a poor person. Jobra
women who made bamboo furniture had to take out usurious loans to buy the bamboo. He made a
loan of USD 27.00 from his 6) __________pocket to 42 women in the village, who made a net
profit of USD 0.02 each on the loan.Yunus believed that if given the chance the poor would repay
the 7) __________ money and hence microcredit could be a 8) __________business 9)
__________. He eventually managed to 10) __________ a loan from the Janata Bank to
lend it to the poor in Jobra in December 1976.
By July 2007, the Grameen Bank had 11) __________ USD 6.38 billion to 7.4 million borrowers.
To ensure repayment, the bank uses a system of solidarity groups. These small informal groups
apply together for loans and its members act as co-guarantors of repayment and support each
other's efforts at economic 12) _________.

The 13) __________ of the Grameen model of microfinancing has inspired similar efforts in
many countries throughout the developing world, and even in industrialised nations, including the
USA. Many, but not all, microcredit projects also retain his emphasis on lending specifically to
women. More than 94% of Grameen loans have 14) __________ to women, who suffer
disproportionately from 15) __________ and who are more likely than men to devote their
earnings to their families.
Choose the best word or phrases to complete the space above
1. a. Former b. Formerly c. Ex- d. The former
2. a. extend b. extent c. extension d. extends
3. a. finder b. founder c. foundation d. findings
4. a. joint b. either c. jointly d. bi-
5. a. disproportionate b. disproportionately c. proportionate d.
proportionately
6. a. self b. own c. only d.
private
7. a. lend b. loan c. borrowing d. borrowed
8. a. viable b. viably c. unviable d. unviably
9. a. models b. modeling c. model d. modeled
10. a. incur b. secure c. insecure d. procure
11. a. ensued b. issued c. borrowed d. lend

114 | P a g e
12. a. advance b. advancing c. advancement d.
advanced
13. a. failure b. mediocrity c. success d.
problem
14. a. gone b. arrived c. sent d. issued
15. a. poor b. poorly c. impoverished d. poverty
Write your answer here!
1 2 3 4 5 6 7 8 9 10 11 12 13 14 15

18. Scottish Independence

The majority of people in Scotland are in favour breaking away from the rest of the UK and
becoming independent, according to a poll taken just before the 300th anniversary of the Act of
Union, which united Scotland and England.

A pair of Acts of Parliament, passed in 1706 and 1707 that came into effect on May 1, 1707,
created Great Britain. The parliaments of both countries were dissolved, and replaced by a new
Parliament of Great Britain in Westminster, London.

The poll showed support for independence for Scotland is running at 51%. This is the first time
since 1998 that support for separation has passed 50%, and the first time since devolution gave
power to the country in 1999. Six months before elections for the Scottish Parliament, these poll
results come as good news to the Scottish Nationalist Party, who are hoping to make progress
against Labour and further the cause of an independent Scotland.

Many people have become disillusioned with devolution, and believe that the Scottish Parliament
has failed to deliver what they had hoped it would; only a tenth have no opinion. In fact, only 39%
of those polled want to keep things as they are.

115 | P a g e
Comprehension Questions...
1. Scotland and England .
a. have always been united.
b. want to break up the union.
c. have been united for a long time.
d. were united by war.
2. Great Britain .
a. was formed by an Act of Parliament in 1706.
b. was formed by two Acts of Parliament in 1707.
c. was formed by an Act of Parliament that came into effect on May 1st 1707.
d. was formed by Acts of Parliament that came into effect on May 1st 1707.
3. People who want indepence for Scotland
a. are the vast majority.
b. are in the minority.
c. are the slight majority.
d. have decreased in number since devolution.
4. The majority of people wanted independence for the first time
a. before devolution.
b. in 1999.
c. after devolution.
d. before and after independence.
5. The results of the poll are good news
a. for Labour.
b. for both parties.
c. for the Scottish Nationalist Party.
d. for devolution.
6. Most people's opinions of devolution
a. have gone up.
b. have gone down.
c. are the same.
d. make progress against Labour.
7. The number of people who want to keep things as they are
a. is greater than those that don't know.
b. is smaller than those that don't know.
c. is increasing.
d. is the majority.

Write your answer here!


1 2 3 4 5 6 7

116 | P a g e
19. DOING BUSINESS AROUND THE WORLD
There are different customs in different parts of the world. For example, how would you say the
date 2-1-1999? Many South Americans give the answer as the second of January, 1999. They put
the day before the month. But North Americans give the answer as February the first. They put the
month before the day. When they’re in another country, business people sometimes miss meetings
because they confuse the various ways of writing dates.
If someone nods their head up and down, what does it mean to you? “No” or “Yes”? Nodding
the head up and down means “yes” in Europe and the United States, but “no” in Greece and Turkey.
The difference only confuses the tourist, but to business people it sometimes means losing a contract.
To prevent misunderstandings, some business people attend classes or read books to learn about the
different manners and customs of the other countries. They don’t want to lose a contract by offending
their client.
Both classes and books show that what are good manners in one country are sometimes bad
manners in another. Even simple gestures can have different meanings. The “thumbs-up” sign means
“excellent” in the United States, but it’s an insult in parts of Africa. Tapping the head with the forefinger
in parts of South America means “I’m thinking hard” but in Holland it means “that’s crazy.” To point a
finger at someone is okay in Canada, but it’s rude in Japan.
Greeting clients correctly makes a good first impression. European men and women generally
shake hands when meeting for the first time, but in Arab countries men never shake hands with women
they’re not related to. In Japan, people bow to each other. People from India place their hands together
as if praying, then bow the head.
In business, entertaining is important. It’s a good idea to invite a client out to eat. However,
unless the host is careful, dining out can be a problem. There are many food taboos. Some people are
vegetarian; others don’t drink alcohol. In the West, people eat with knives and forks; in the East, they
eat with chopsticks.
Sometimes business executives are invited into the homes of their clients. . Even here there
are rules. In England giving a knife is bad luck. The Chinese refuse a gift three times before accepting
it because they do not want to appear greedy. Some people in Southeast Asia don’t give handkerchiefs
because it makes them think of crying at funerals. One of the biggest difficulties is how much to spend.
If the guest spends only a little, the host might think the gift is stingy. If the guest spends too much, the
host might think the gift is a bribe. In Malaysia and many other countries, there are strict rules against
bribery.
If you ever see a worried-looking foreigner in a gift shop, he or she is probably a business
executive wondering what to buy.

A. TRUE/ FALSE STATEMENTS


 Direction: Scan through the passage so that you will find out the factual information.

(A) Write T for “True” statements.


(B) Write F for “False” ones.
(C) Write for N.S.“Not Sure”.
(D) Write N.G. for “Not Given”.

(1) There are the same customs in different parts of the world.
a. True
b. False
c. Not sure
d. Not given
(2) Nodding the head up and down means “no” in Europe and the United States, but “yes” in Greece
and Turkey.
a. True
b. False
c. Not sure
d. Not given
(3) To point a finger at someone is okay in Canada, but it’s rude in Japan.
a. True
b. False
c. Not sure
d. Not given
(4) In Arab countries, men never shake hands with women they’re not related to.
a. True
b. False
c. Not sure
d. Not given
(5) In China, people write date based on year-month-day order.
a. True
b. False
c. Not sure
d. Not given
(6) It’s the custom in most countries for the guest to take a small gift to the host.
a. True
b. False
c. Not sure
d. Not given
(7) In the West, people eat with chopsticks.
a. True
b. False
c. Not sure
d. Not given
(8) If the guest spends too much, the host might think the gift is a bribe; therefore, anti-corruption
unit may punish them.
a. True
b. False
c. Not sure
d. Not given
(9) Some people in Southeast Asia don’t give handkerchiefs because it makes them think of crying at
funerals.
a. True
b. False
c. Not sure
d. Not given
(10) In England giving a knife is good luck.
a. True
b. False
c. Not sure
d. Not given
Write your answer here!
1 2 3 4 5 6 7 8 9 10

B. COMPREHENSION CHECK: MULTIPLE CHOICES


 Direction: Read the passage carefully. Then choose the answer that best matches with the
passage.

(11) A very expensive gift might be taken for a ______________.


(A) business deal (C) funeral
(B) bribe (D) taboo
(12) The Chinese refuse gift before they finally accept it, because they do not want to
appear______________.
(A) solemn (C) greedy
(B) stingy (D) too happy
(13) Business people learn about the customs of the country they’re going to visit because they
don’t want to______________.
(A) be ignorant in the intercultural (C) lose a contract by offending their
communication client
(B) misunderstand the other cultures (D) Both (B) and (C)

(14) Tapping the head with the forefinger means “______________” in Holland.
(A) that’s clever (C) that’s a bribe
(B) that’s crazy (D) I’m thinking hard
(15) If you greet people politely, it makes a good first_______________.
(A) bribe (C) entertainment
(B) difficulty (D) impression

(16) In England, some people say that giving a knife is ______________.


(A) bad luck (C) a bribe
(B) good luck (D) a contract

(17) ______________make people think of funerals in some parts of Southeast Asia.


(A) Knives (C) Eating with chopsticks
(B) Bribes (D) Handkerchiefs

(18) When business executives travel to foreign countries, they are sometimes invited into the
homes of their ______________.
(A) guests (C) clients
(B) business people (D) tourist agents

(19) Many people are______________. They don’t eat meat.


(A) business
(B) hosts
(C) guests
(D) vegetarians
International University, IU
Test Level 3

(20) This article mainly focuses on:


(A) It can be difficult to do business in foreign countries.
(B) It’s a good idea to learn the customs of a place before doing business there.
(C) Doing business in the other countries requires a lot of capital investment.
(D) There are many strange and interesting customs in the countries of the world.
(11) (12) (13) (14) (15) (16) (17) (18) (19) (20)

20. The hardest language

People often ask which is the most difficult language to learn, and it is not easy to answer
because there are many factors to take into consideration. Firstly, in a first language the
differences are unimportant as people learn their mother tongue naturally, so the question of
how hard a language is to learn is only relevant when learning a second language.

A native speaker of Spanish, for example, will find Portuguese much easier to learn than a
native speaker of Chinese, for example, because Portuguese is very similar to Spanish, while
Chinese is very different, so first language can affect learning a second language. The greater
the differences between the second language and our first, the harder it will be for most people
to learn. Many people answer that Chinese is the hardest language to learn, possibly influenced
by the thought of learning the Chinese writing system, and the pronunciation of Chinese does
appear to be very difficult for many foreign learners. However, for Japanese speakers, who
already use Chinese characters in their own language, learning writing will be less difficult than
for speakers of languages using the Roman alphabet.

Some people seem to learn languages readily, while others find it very difficult. Teachers and
the circumstances in which the language is learned also play an important role, as well as each
learner's motivation for learning. If people learn a language because they need to use it
professionally, they often learn it faster than people studying a language that has no direct use
in their day to day life.

Apparently, British diplomats and other embassy staff have found that the second hardest
language is Japanese, which will probably come as no surprise to many, but the language that
they have found to be the most problematic is Hungarian, which has 35 cases (forms of a nouns
according to whether it is subject, object, genitive, etc). This does not mean that Hungarian is
the hardest language to learn for everyone, but it causes British diplomatic personnel, who are
generally used to learning languages, the most difficulty. However, Tabassaran, a Caucasian
language has 48 cases, so it might cause more difficulty if British diplomats had to learn it.

Different cultures and individuals from those cultures will find different languages more difficult.
In the case of Hungarian for British learners, it is not a question of the writing system, which
uses a similar alphabet, but the grammatical complexity, though native speakers of related
languages may find it easier, while struggling with languages that the British find relatively easy.

No language is easy to learn well, though languages which are related to our first language are
easier. Learning a completely different writing system is a huge challenge, but that does not
necessarily make a language more difficult than another. In the end, it is impossible to say that
there is one language that is the most difficult language in the world.
Page 2 of 170
International University, IU
Test Level 3

1. The question of how hard a language is to learn is relevant to both first and second
language acquisition.
a. True
b. False
c. Not sure
d. Not given
2. Portuguese is definitely easier than Chinese.
a. True
b. False
c. Not sure
d. Not given
3. A Japanese speaker may well find the Chinese writing system easier than a speaker of a
European language.
a. True
b. False
c. Not sure
d. Not given
4. The Hungarian alphabet causes problems for British speakers.
a. True
b. False
c. Not sure
d. Not given
5. Hungarian is the hardest language in the world.
a. True
b. False
c. Not sure
d. Not given
6. Hungarian has as many cases as Tabassaran.
a. True
b. False
c. Not sure
d. Not given
7. Many British diplomats learn Tabassaran.
a. True
b. False
c. Not sure
d. Not given
8. The writer thinks that learning new writing systems is easy.
a. True
b. False
c. Not sure
d. Not given
Page 3 of 170
International University, IU
Test Level 3

Write your answer here!

1 2 3 4 5 6 7 8

21. Piranhas
Scientists in the UK have announced that the piranha fish's reputation 1) __________ a
fearsome 2) __________ may well not be deserved. The fish, which is found in the Amazon in
Brazil, have been portrayed as deadly 3) __________ that work in shoals to overwhelm their
prey and strip it of its flesh in seconds.

However, 4) __________ from St Andrews University say that piranhas are omnivores that
mainly eat fish, plants and insects. They form big groups not to hunt but to defend 5)
__________ against other predators, according to the team.

"Previously it was thought piranhas shoaled as it 6) __________ them to form a cooperative


hunting group," said Professor Anne Magurran. "However, we have found that it is primarily a
defensive 7) __________."

Piranhas can be attacked by animals 8) __________ dolphins, caimans and large fish, so
forming a shoal is a good way of 9) __________ being killed. Piranhas of 10) __________ age
stay in the middle of the group for 11) __________ and the 12) __________ of the shoal 13)
__________ according to the level of 14) __________. When the water level is high, the fish
form small groups as there is space to escape, but when the water level drops, they form large
groups as protection against 15) __________.

Q1 a. as b. as if c. as though d. since
Q2 a. kill b. killer c. murderer d. murder
Q3 a. carnivore b. carnivorous c. carnivors d. carnivores
Q4 a. expert b. expertise c. experts d. expectant
Q5 a. them b. themselves c. it d. itself
Q6 a. able b. ables c. enable d. enabled
Q7 a. behave b. behavior c. misbehavior d. misbehaviours
Q8 a. such b. as c. like d. as such
Q9 a. prevent b. preventing c. avoid d. avoiding
Q10 a. reproduce b. reproductive c. reproduction d. reproductively
Q11 a. protection b. protecting c. protect d. protective
Q12 a. rate b. size c. amount d. dearth
Q13 a. different b. differ c. differs d. defer
Q14 a. risk b. fate c. destiny d. chance
Q15 a. predator b. predators c. predatory d. predate

Write your answer here!

1 2 3 4 5 6 7 8 9 10 11 12 13 14 15

22. Changes in Town

I have returned to my hometown of Wilson Creek after an absence of 10 years.


Page 4 of 170
International University, IU
Test Level 3

So many things have changed around here. When I left Wilson Creek, there was a small pond
on the right as you left town. They have filled in this pond and they have built a large shopping
mall there. A new post office has also been built just across from my old school.

There is a baseball stadium on the outskirts of Wilson Creek which has been changed
completely. They have now added a new stand where probably a few thousand people could
sit. It looks really great.

The biggest changes have taken place in the downtown area. They have pedestrianised the
centre and you can't drive there anymore. A European-style fountain has been built and some
benches have also been added along with a grassy area and a new street cafe.

My street looks just the same as it always has but a public library has been built in the next
street along. There used to be a great park there but they have cut down all the trees which is a
pity. The library now has a large green area in front of it but it's not the same as when the park
was there.

Another improvement is the number of new restaurants that have opened in Wilson Creek. A
Chinese and an Italian restaurant have opened in the town centre and a Mexican restaurant has
opened near my home. Which is where I am going tonight!

1. Jake's school doesn't exist anymore.


a. True
b. False
c. Not sure
d. Not given
2. They have improved the baseball stadium.
a. True
b. False
c. Not sure
d. Not given
3. Jake likes the new-look baseball stadium.
a. True
b. False
c. Not sure
d. Not given
4. It's only possible to reach the downtown area on foot.
a. True
b. False
c. Not sure
d. Not given
5. The buildings in the centre of the town look European.
a. True
b. False
c. Not sure

Page 5 of 170
International University, IU
Test Level 3

d. Not given
6. Jake's street hasn't changed much.
a. True
b. False
c. Not sure
d. Not given
7. He is sad about the park being ruined.
a. True
b. False
c. Not sure
d. Not given
8. Jake is going to eat Italian food tonight.
a. True
b. False
c. Not sure
d. Not given
Write your answer here!

1 2 3 4 5 6 7 8

23. A Traditional Wedding


Everybody loves a good wedding and I'm no exception. I've been to a load of them in my native
Britain and I must say that I usually have a great time. I've also been to a few abroad, including
the Caribbean and Spain, and most recently (last week in fact) to one in the mountains of
Sardinia. No two weddings are ever the same and I really enjoyed this one for one or two of the
differences from those in the UK.

First, the two families spent at least three weeks before the big day preparing all the food, from
wonderful home-made delicacies to simple traditional breads and pastas. In my experience, in
the UK that onerous task is left to the caterers! In the week leading up to the wedding there is a
dinner or some form of celebration every day - training for the stomach I guess. I know that we
have the traditional Bachelor party and Bachelorette party, but this is more family orientated and
certainly a little less rowdy. This particular ceremony was in a beautiful country church and
afterwards the couple was driven to the reception in a wonderfully decorated classic Fiat 500,
which was really similar to what happens in the UK, even down to the string of tin cans trailing
behind the car!

The reception itself was also very similar until I realized that the seven tables in the hall each
sat sixty guests (that's four hundred and twenty, for those of you who didn't study Math), an
average number for Sardinia but would be considered a very large wedding where I'm from.

The wine flowed, as did the chatter - the famous Italian exuberance showing itself to the full.
There were the five or six courses of wonderful food, screaming kids running wild, the
ceremonial cutting of the cake by the bride and groom - but no speeches! Not one. In the UK it's
traditional for the father of the bride to propose a toast, followed by the groom and finishing up

Page 6 of 170
International University, IU
Test Level 3

with that of the best man. His is meant to be he highlight of the lunch \ dinner, generally having
a good laugh at the groom's expense, but here the groom was spared that particular discomfort.

Instead there was a delightful custom which I'd never seen before, in which six or seven of the
male guests pass round the hall banging trays, drums, pots, pans or basically anything that
makes a horrendous noise, selling pieces of the groom's tie which has been cut into tiny bits.
The money raised is then given to the happy(!) couple to help them set up their new life
together. Really nice. Finally the evening saw a lot of traditional dancing, a little disco dancing
and some karaoke. Pretty much the part I like best, and again I wasn't disappointed. Can't wait
for the next one.

1. The writer generally likes weddings.


a. True
b. False
c. Not given
d. Not sure
2. He didn't particularly enjoy the Sardinian one.
a. True
b. False
c. Not given
d. Not sure
3. The families helped the caterers to prepare the food.
a. True
b. False
c. Not given
d. Not sure
4. There is usually a dinner the night before the wedding.
a. True
b. False
c. Not given
d. Not sure
5. The groom meets the bride outside the church.
a. True
b. False
c. Not given
d. Not sure
6. UK wedding receptions are often a lot smaller than Sardinian ones.
a. True
b. False
c. Not given
d. Not sure
7. The writer found it hard to understand the speeches.
a. True
b. False
c. Not given
d. Not sure
8. The staff of the reception hall cut the cake for the guests.
a. True
b. False
c. Not given
d. Not sure
Page 7 of 170
International University, IU
Test Level 3

9. There is an auction to sell the groom's tie.


a. True
b. False
c. Not given
d. Not sure
10. The money from this helps to pay for the reception.
a. True
b. False
c. Not given
d. Not sure
Write your answer here!
1 2 3 4 5 6 7 8 9 10

24. Snake Bites Boy


Three-year-old Teddy Lasry was napping yesterday in his cowboy outfit yesterday at his family's
Fifth Ave. apartment when he shot up in bed screaming. A 3-foot-long black-and-white snake
was coiled around his left arm and had just bitten his pinky.

"The baby-sitter freaked out," said Teddy's father, David Lasry, who, along with his wife, Evelyn,
was at work when the reptile showed up about 4 p.m.

The horrified nanny called 911 and the building's doorman. The doorman and two cable TV
workers helped pry the snake off the boy's arm and stow it in a garbage bag, Lasry said. Police
rushed Teddy to Mount Sinai Medical Center, where his parents said he spent two hours
attached to a heart monitor as a precaution in case the snake was poisonous. It wasn't. Experts
at the snakebite treatment center at Jacobi Medical Center in the Bronx, where cops took the
critter, determined it was a non-venomous California king snake. But how did it end up in
Teddy's bed?

A little sleuthing determined that the serpent had escaped two weeks ago from its cage in the
apartment of a doctor whose family lives four floors below the Lasrys. The apologetic owner
said his son's pet snake likely traveled up the radiator pipes and into his neighbor's apartment.
"It's a very docile, very harmless snake," he said. "It's handled by our family all the time."

Lasry, 42, a fine arts publisher, said he believed the pet was simply hungry after two weeks of
cruising. Teddy's mother, Evelyn Lasry, 37, said her son seems to have gotten over his fright by
thinking of himself as a hero cowboy as he rode in the back of the police cruiser to the hospital.
"I told Teddy he's a pretty snake, a nice pet snake who got out of his cage," Evelyn Lasry said.
"But he asked, 'Why did he bite my finger, Mamma?' And I said, 'Because he saw that you are a
big boy, Teddy, in your cowboy outfit and he got scared.'"
1. What did the babysitter do?
a. She ran out of the apartment.
b. She took the snake off Teddy's arm.
c. She called for help.
d. She called the television company.
2. What do we learn about the snake?

Page 8 of 170
International University, IU
Test Level 3

a. It was poisonous.
b. It had escaped from a zoo.
c. It was about a meter long.
d. It had escaped earlier in the afternoon.
3. Which of these statements is true?
a. Teddy was awake when the snake arrived.
b. Teddy's father was working and his mother was at home.
c. Teddy needed a heart machine to stay alive for two hours.
d. The snake is used to being touched.
4. What does Teddy think now of the snake attack?
a. He was attacked because the snake was scared of him.
b. He was attacked because he was asleep.
c. He was attacked because the snake was hungry.
d. He was attacked because his parents weren't at home.

1 2 3 4

Updated

WRITING PLACEMENT TEST


LEVEL 3
PAT I: Questions 1-108
Why Are Social Networking Sites So Popular?
1- Social networking sites become more and more popular every day, and they are popular all around the
world. In Japan, the top site is Mixi. In Europe, it is Bebo. The most popular site in Latin America is
Orkut. In the United States, the top site is Facebook. In fact, Facebook is one of the most popular social
networking sites in the world. A Harvard University student started Facebook in 2004, and it spread to
more than 400 million users in just a few years.
2- Why is the social networking trend spreading so rapidly? One reason that these websites are popular is
because people are social. We like to communicate with other people. We make friends with people in
school, at work, and online. Most people like to stay closely connected to their friends and family. We use
cell phones, email, instant messaging, and websites to learn what our friends are doing. The Internet is a
good way to socialize and communicate, and social networking sites allow people to do this in many
ways.
3- Social networking sites are interactive and personal. People can share photographs of themselves and
of others. They can tell people what they are doing at any moment and keep in touch. They can post a link
to a site with their favorite song or band. They can join groups with others who share their interests. Many
Page 9 of 170
International University, IU
Test Level 3

people post videos of themselves on sites like YouTube. Other users can comment on these photos and
videos. This interaction makes these websites become more popular.
Choose the best response.
34. Which of the following is the main idea of paragraph 2?
A. People stay connected to their friends and family in many ways.
B. Social networking sites are popular because people are social.
C. The Internet is a good way to socialize and communicate.
D. The Internet is not a good way to socialize and communicate.
35. Which of the following is the main idea of paragraph 3?
A. On social networking sites, people can join groups with others who share their interests.
B. Many people post videos of themselves on sites like YouTube, and others can comment on
these videos.
C. Social networking sites are very popular because they are interactive and personal.
D. On social networking sites, people can’t join groups with others who share their interests.
36. Which of the sentences is NOT a main idea in this reading passage?
A. A Harvard University student started Facebook in 2004, and it spread to more than 400
million users in just a few years.
B. Every day more and more people join social networking sites all around the world.
C. Because social networking sites are interactive and personal, their popularity has
increased.
D. More people join social networking sites all around the world.

Should I Kiss, Bow, or Shake Hands?


1- Most people want to be polite and behave well around others. Being polite means knowing how to
greet and talk to people. It means using good manners when eating. It means knowing how to give and
receive gifts appropriately. Polite behavior in one country, however, may be impolite in another part of
the world. Travelers need to understand the cultural differences in politeness so that they don’t cause
embarrassment.
2- For instance, when people meet, they often shake hands. How long should a handshake be? Should you
hold the other person’s hand gently or firmly? In the United States, people prefer to shake hands firmly
for a few seconds. In some Middle Eastern countries, people hold the person’s hand gently for a longer
time. Handshaking varies around the world.
3- What about eye contact? In some countries you show respect when you look someone directly in the
eye. In other parts of the world, to look at someone directly is rude. To be respectful, a person looks down
at the ground.
4- There are also cultural differences in the way people use personal space. When two people are talking,
should they stand close together or far apart? Exactly how close should they stand? In North America, for
instance, people usually stand about an arm’s length apart during a conversation. However, in some
countries in the Middle East and Latin America, people stand closer. It can be awkward if one person
likes to stand close and the other person likes to stand farther apart.
5- Another cultural difference is time. If someone invites you to dinner at their house at 6 p.m., what time
should you get there? Should you arrive early, late, or exactly on time? In Germany, it is important to
arrive on time. In Argentina, polite dinner guests usually come 30 to 60 minutes after the time of the
invitation. When traveling, remember that each country has a different definition of time.
Choose the best response.
37. Which of the following is NOT an example of a supporting detail in paragraph 1?
A. Most people want to be polite and behave well around others.
B. Being polite means knowing how to greet and talk to people.

Page 10 of 170
International University, IU
Test Level 3

C. It means knowing how to give and receive gifts appropriately.


D. Knowing how to greet and talk to people.
38. Which of the following details about the Middle East is NOT found in the text?
A. People in the Middle East stand closer than Americans during a conversation.
B. When shaking hands, people in the Middle East hold a person’s hand gently for a longer
time.
C. In the Middle East, it is important to bring a gift to someone’s house.
D. People stand closer in Americans during a conversation.
39. Which of the following is the main idea of paragraph 2?
A. When people meet, they often shake hands.
B. How long should a handshake be?
C. Handshaking varies around the world.
D. While people are meet, they are shaking hands.
40. Which of the following is NOT a supporting detail in paragraph 2?
A. In some Middle Eastern countries, people hold the person’s hand gently for a longer time.
B. Should you hold the other person’s hand gently or firmly?
C. In the United States, people prefer to shake hands firmly for a few seconds.
D. In the Middle East, people prefer to shake hands firmly for a few seconds.

The Richest and the Best


1- Manchester United Football Club is the richest sports team in the world. The English club is worth
more than $1.8 billion. The Spanish soccer team Real Madrid is worth $1.3 billion. These clubs have
enough money to pay the high salaries of the best players in the world. For example, in 2008, Manchester
United paid Cristiano Ronaldo more than $18 million. They paid Wayne Rooney more than $16 million.
Other teams also spend a lot of money on their players. These wealthy clubs include Barcelona in Spain,
AC Milan in Italy, and Chelsea in England. As a result, these teams have been very successful. For
example, Manchester United won championships in 2006, 2008, and 2009. Money is a clear advantage
for these professional teams.
2- The wealthiest teams make money in several ways. First, they can sell tickets at high prices because
fans want to see the top players in action. Second, television stations pay the teams to broadcast their
games. Third, large companies give financial support to the best teams. These corporate sponsors pay
money so that their company name can be on the clubs’ shirts, shoes, and stadiums.

3- The recent success of the English team Chelsea shows an example of the effect of money on a team.
For many years, Chelsea was a club with little money and few wins. Everything changed in 2003, when
Russian billionaire Roman Abramovich bought the team for $233 million. In his first year, Abramovich
spent more than $350 million on players. By 2006, Chelsea had won two championships. Now Chelsea is
one of the richest clubs in the world.
4- Many sports fans feel that money in sports creates an unfair advantage. Some teams have so much
money that it seems impossible for poorer clubs to beat them. Wealthy teams are usually more successful,
so they sell more tickets and make more money. Teams that do not have as much money usually are not
as successful. It is difficult for them to get money from tickets, television, and corporate sponsors. Sports
fans know that money gives some clubs a great advantage.
Choose the best response.
41. Which of the following details would NOT be listed in your notes about the text?
A. In 2008, Manchester United paid Wayne Rooney more than $16 million.
B. The Spanish soccer team Real Madrid is worth $1.3 billion.
C. For example, in 2008, Manchester United paid Cristiano Ronaldo more than $18 million.
D. Cristiano Ronaldo is the richest player on the Real Madrid team.
42. When taking notes about the text, what main idea should be listed for paragraph 2?
A. The best teams can sell their tickets at high prices.
Page 11 of 170
International University, IU
Test Level 3

B. There are several ways the richest teams can earn money.
C. Companies pay teams to have their name on the teams’ shirts.
D. People can sell their tickets at high prices.
43. What supporting idea would be in your notes about paragraph 3?
A. TV stations paid money to broadcast the games.
B. A Russian billionaire bought the team for $233 million.
C. Wayne Rooney was paid more than $16 million.
D. TV stations did not pay money to broadcast the games.
44. What is an important way to use notes that you have taken?
A. to study for a test
B. to think of ideas
C. to write to friends
D. to write a plan

A Disposable Society
1- In our modern world, when something wears out, we throw it away and buy a replacement. If a shirt is
torn or a coffee machine breaks, you throw it away. The problem is that countries around the world have
growing mountains of trash because people are throwing out more trash than ever before. For example, in
the United States, the amount of trash per person nearly doubled from 1960 to 2000.

2- How did we become a throwaway society? First of all, it is now easier to replace an item than to spend
time and money to repair it. Thanks to modern manufacturing and technology, companies are able to
produce items quickly and inexpensively. Products are plentiful and prices are low, so we would rather
buy something new than repair it. Even if we did want to repair something, many items—from toasters to
TVs—are almost impossible to repair. These products contain many tiny, complicated parts. Some even
contain small computer chips. It’s easier to throw these items away and buy new ones than to fix them.

3- Another contributing factor is our love of disposable products. As busy people, we are always looking
for ways to save time and make our lives easier. Why should we use cloth kitchen towels? It is easier to
use a paper towel once and toss it out. Companies manufacture thousands of different kinds of disposable
items: paper plates, plastic cups, cameras, and razors for shaving, to name a few. Because these products
aren’t designed to last, companies know that consumers will have to replace them, buying them over and
over again. “What’s wrong with that?” you ask. The problem is that disposable products are contributing
to our trash problem.
Choose the best response.
45. Which of the following statements from the reading is a fact?
A. We would rather buy something new than repair it.
B. It is easier to use a paper towel once and toss it out.
C. It’s easier to throw these items away and buy new ones than to fix them.
D. Companies manufacture thousands of different disposable items.
46. Which of the following statements from the text is an opinion?
A. In the United States, the amount of trash per person nearly doubled from 1960 to 2000.
B. It’s easier to throw items away and buy new ones than to fix them.
C. Some toasters and TVs contain small computer chips.
D. Companies manufacture thousands of different disposable items.
47. Which of the following statements is a fact?
A. He’s the best president we’ve ever had in this country.
B. It’s easier to throw items away and buy new ones than to fix them.
C. It’s impressive that he’s one of the youngest presidents of the United States.
D. He was sworn in as president on January 20, 2009.
Page 12 of 170
International University, IU
Test Level 3

48. Which of the following statements is an opinion?


A. It’s cold in here.
B. I know when the library opens.
C. He was sworn in as president on January 20, 2009.
D. She is a great dancer.
49. Which of the following statements is a fact?
A. The last day we had any significant rainfall was on July 1, 2010 when there was an
afternoon shower.
B. Today is the most miserable day of the summer.
C. There are billions of people in the world, and most of them love music.
D. It’s easier to throw items away and buy new ones than to fix them.

The Ingredients of a Good Story


1- American playwright Elmer Rice said that a play or story in its simplest form has three parts. He said
that the writer needs to:
1. Put a man up a tree.
2. Throw stones at him.
3. Get him down.
This may seem surprisingly simple, but his clever idea works well for many kinds of stories. Let’s look at
how you can use this recipe for writing your own short story.

2- In part 1, Put a man up a tree, we have two important ingredients of a story: the main character and the
setting. Your main character is the person at the center of your story. Who is this person? What does your
character look like? What is his or her personality? Your setting is where your story takes place. This is
the physical location of the story. In some stories, the setting is very important, while in other stories, it
isn’t. In the example, the setting is a tree, and this is essential to the story. The setting can also include the
time of the story, or whether it takes place in the past, present, or future. The time span is also important.
For a short story, the time span is usually short.

3- Part 2 of the story, Throw stones at him, is the conflict. The conflict is the problem facing the main
character. In the example, the character’s problem is that he is in a tree, and he wants to come down.
There is a conflict between the character and the person who is throwing stones at him. The conflict in a
story is the basic problem that your character must resolve. What is the problem facing your character?
Why does it matter to the main character?

4- In part 3, Get him down, the main character solves the problem or overcomes the challenge at the end
of the story. He comes down from the tree. This is the resolution of the story. In a good story, the
resolution isn’t always easy, and the main character may resolve the problem in an unexpected way.
Choose the best response.
50. Which of the following is NOT a tip for summarizing?
A. Take notes about the main ideas and important details in a text.
B. Don’t go back to reread the text a second time.
C. Put your ideas together into sentences or paragraphs.
D. Identify the sections of the text.
51. Look at paragraph 2. Which is the best summary?
A. The main character and the setting are two of the most important parts of a story. The main
character is the person at the center of the story. The setting is where the story takes place.
It is important to think carefully about both of these parts of the story and ask yourself a lot
of questions.

Page 13 of 170
International University, IU
Test Level 3

B. Paragraph 2 talks about the introduction of a story. It introduces two of the most important
parts of a story, the main character and the setting, and explains what each of them is.
C. It’s really important to write as much as possible about the main character of the story.
Also the setting is important. Make sure your story has this information in it, and give lots
of details.
D. Paragraph 2 talks about the introduction of a story. It doesn’t introduce two of the most
important parts of a story, the main character and the setting, and explains what each of
them is.
52. Look at paragraph 3. Which of the following is NOT an important idea for a summary of this
paragraph?
A. As a writer, you need to show why the problem is important to the main character.
B. It is important to make the person stop throwing stones at the main character.
C. The conflict in a story is the basic problem that your character must resolve.
D. As a writer, you must show why the problem is important to the main character.
53. Look at paragraph 4. Which is the best summary?
A. The third part of a story is the end, when the character solves the problem.
B. In the third part of a story, the main character resolves the problem that he faced in the
middle of the story, and the story ends.
C. In a good story, the resolution is not difficult and different from what you would expect.
D. In the third part of a story, the character comes down from the tree because the person has
finally stopped throwing stones at him.

Read the paragraph. Then choose the best definition for each word using the context.
United Parcel Service (UPS) is a big delivery company. Its company color is brown. When UPS started in
the 1920s, brown was a good color for a safe, reliable company. From the beginning, UPS used brown
trucks and brown uniforms. In today’s world, brown may seem like a boring color choice for a company.
But UPS decided to make it a positive symbol of its business. Their ads ask, “What can brown do for
you?” When people see the big brown UPS trucks, the company hopes they will think of excellent,
dependable service.
54. reliable
A. dependable
B. mature
C. secure
D. excellent

55. boring
A. stable
B. uninteresting
C. bad
D. disappointed
56. symbol
A. sign
B. gesture
C. idea
D. term

Read the paragraph. Choose the sentence that shows the main idea.
57. Social networking sites first became popular with college students. At one college, students said
that they spent almost two hours every day just on Facebook. Teenagers also use these sites to stay
connected with their friends. These days, even older people are using social networks. The

Page 14 of 170
International University, IU
Test Level 3

Internet keeps changing, but one trend is clear: People enjoy using websites that let them connect
with others.
D. These days, even older people are using social networks.
E. People do not enjoy using websites that let them connect with others.
F. People enjoy using websites that let them connect with others.
G. Social networking sites first became popular with college students.

Read the main idea. Then choose the best supporting sentence for the main idea.
58. In many cultures, it is rude to touch people you do not know very well.
A. In US, people shake hands or kisses for greetings.
B. For example, in Thailand, it is rude to touch a person’s head with the palm of your hand.
C. In many cultures, it is also rude to stand too close to somebody when talking.
D. Touching people you know is usually OK.
59. Each culture has specific rules for gift-giving.
A. In every culture, gift-giving is an important way to show appreciation and love for others.
B. In China, you should open a gift after the person is gone.
C. Rules for gift-giving vary from culture to culture.
D. Gift-giving is an important way to show appreciation and love for other people.

Choose the best response.


60. Which of the sentences below shows a very strong positive statement of opinion?
A. Officials shouldn’t allow girls to play the same sports as boys.
B. Officials must allow girls to play the same sports as boys.
C. Officials should allow girls to play the same sports as boys.
D. Officials ought to allow girls to play the same sports as boys.
61. Which of the sentences below shows a weak negative statement of opinion?
A. Professional athletes must not be allowed to take drugs.
B. Owners must not put a limit on the salaries of professional soccer players.
C. Soccer coaches should not earn more money than soccer players.
D. Soccer coaches should earn more money than soccer players.
62. Which phrase is NOT used to introduce an opinion in an opinion paragraph?
A. I don’t think
B. I believe
C. In my opinion
D. I know

63. Which sentence would be a strong concluding sentence in a paragraph about college sports
players?
A. In my opinion, college sports players should have good grades to play sports.
B. For these reasons, I feel college sports players should focus more on their grades than on
their sports.
C. For these reasons, I feel college sports players should not focus more on their grades than
on their sports.
D. Second, they are representing the college or university when they play.

Complete the sentence. Choose A, B, or C.


64. I don’t have a good Internet __________. Sometimes I can’t get online.
A. connect
B. conects
Page 15 of 170
International University, IU
Test Level 3

C. connection
D. connected
65. Hong __________ movies very much. Every weekend, he goes to the theater.
A. enjoys
B. enjoyment
C. enjoyed
D. enjoying
66. This __________ about traveling to Spain is very useful.
A. information
B. inform
C. informing
D. informed
34. We have a __________ of drinks to choose from. Which would you like?
D. mass
E. culture
F. variety
G. idea
35. They plan to __________ a new library system next year.
D. argue
E. establish
F. encourage
G. start
36. I don’t buy coffee at that shop because of the bad __________. The coffee is always cold by the
time I get it.
D. service
E. environment
F. smell
G. connection
37. The mayor doesn’t support the construction of a new mall because it will hurt the __________.
D. building
E. emotion
F. feeling
G. environment
38. Her parents __________ her to study more to improve her grades.
D. demotivate
E. encourage
F. represent
G. request

39. You should __________ talking to your boss if you’re not satisfied at work.
D. Affect
E. understand
F. consider
G. prefer

Read the sentence. Is the underlined word a noun or a verb? Choose A, B, or C.


40. I am not good at math, so I have to study very hard.
D. noun
E. adjective
Page 16 of 170
International University, IU
Test Level 3

F. adverb
G. verb
41. I like watching previews of new movies when I go to the movie theater.
D. adverb
E. noun
F. verb
G. adjective
42. Jasmine gave her friend a gift for his birthday.
D. adverb
E. noun
F. verb
G. adjective
65. She uses recycled paper in her artwork.
A. adverb
B. adjective
C. verb
D. noun
66. The threatened animals left the forest.
A. adjective
B. adverb
C. verb
D. noun
67. The city lowered the amount of electricity it used.
A. Adjective
B. adverb
C. verb
D. noun
68. The old factory released a lot of smoke.
A. Adjective
B. adverb
C. noun
D. verb
69. Reducing waste was an expected benefit of the recycling program.
A. adjective
B. verb
C. adverb
D. noun
70. Indoor farmers often use potted plants to grow vegetables.
A. adjective
B. verb
C. adverb
B. noun

Read the paragraph. Choose the best topic sentence for the paragraph.
71. More and more things are being done by computer these days. Job applications, banking, and
travel reservations can all be done by computer. Most jobs require a person to know how to use a
computer. Also, people are using computers more and more to socialize and connect with other
people. Email, instant messaging, and social networking sites like Facebook are all examples of
how people are using computers to connect.
D. More and more things are being done by computer these days.
Page 17 of 170
International University, IU
Test Level 3

E. It is very important to learn how to use a computer.


F. You can download and play music and videos on the computer.
G. Most people know how to use a computer today.

Read the paragraph. Then choose the best response.


FREEWRITING TOPIC: What is your favorite color? Why?
Purple is my favorite color. I think it is an interesting color. You don’t see it every day. When I was a
child I had my favorite purple shirt. I wore it almost every day! I also had a dog when I was a child. When
I see purple, it makes me think of mysterious and exciting things.
72. Which sentence does NOT focus on the topic?
A. You don't see it every day.
B. I also had a dog when I was a child.
C. It makes me think of mysterious and exciting things.
D. All answers
73. Which of the following sentences can be added to the paragraph?
A. My favorite flower, the iris, is purple, too.
B. I think green is a very pretty color, too.
C. One mysterious thing I think of is dreams.
D. All answers

Choose the best response.


74. Which is the best topic sentence?
A. My first teacher made a very good impression on me.
B. My first teacher always came to class well prepared.
C. My first teacher greeted the students kindly.
D. My first teacher always came to class early.
75. Which is the best topic sentence?
A. A good doctor tries to understand how patients feel.
B. A good doctor has two important qualities.
C. A good doctor is caring toward patients.
D. A good doctor is generous.
76. Which is the best supporting sentence?
A. Read the job descriptions very carefully.
B. Here are five tips for searching job websites.
C. Finding the right job online is not easy.
D. Finding the right job online is very easy.
77. Which is the best concluding sentence?
A. First of all, you need to listen to them so they know you care.
B. Do not interrupt them when they are speaking and keep steady eye contact.
C. When you follow these steps, it is easier to deal with difficult customers.
D. Dealing with difficult customers is easy.

78. Which of the following makes a better topic sentence for a paragraph about how we choose our
music?
D. In a study, researchers gave a list of 48 unknown songs to 14,000 teenagers.
E. I like some songs very much.
F. Researchers say that hit songs become popular because listeners care about what other people
think.
Page 18 of 170
International University, IU
Test Level 3

G. Some people look at the list of favorite songs posted by someone they know.

Read the paragraph. Then answer the question with the sentence that synthesizes information from
the paragraph and the passage.
H1N1 Flu
Laboratory studies have shown that no children and very few adults younger than 60 already have
immunity to the 2009 H1N1 flu virus. However, about one-third of adults over 60 may have immunity to
the virus.
79. Which of the following is NOT a reasonable conclusion about the flu virus?
A. Both the seasonal flu and the H1N1 flu are dangerous for the same groups of people.
B. Children of any age can get both the H1N1 flu and the seasonal flu quite easily.
C. Adults younger than 60 are more likely to get the H1N1 flu than adults over 60.
D. All answers
80. Which of the following can NOT be concluded about the H1N1 virus?
A. The H1N1 flu is quite different from the normal seasonal flu.
B. Adults over 60 are more likely to get the seasonal flu than the H1N1 flu.
C. The H1N1 flu has existed almost as long as the seasonal flu.
D. All answers
The sentences below form a paragraph. Match the sentences with their order in the paragraph.
_______ 59. I don’t like the idea of an electronic reading device.
E. second
_______ 60. Also, when reading an e-book, I always worry about the battery
sentence
running out.
F. first sentence
_______ 61. Finally, I think a house doesn’t feel like a home without a collection
G. last sentence
of real books.
H. third sentence
_______ 62. In particular, I don’t like the way an e-book feels in my hands.

The sentences below form a paragraph with supporting examples. Match the sentences with their
order in the paragraph.
_____ 63. One kiss is nothing for the French; when two French
people meet, they usually give four kisses, two on each
cheek.
_____ 64. For example, in Mexico, when two people meet, they
E. first sentence
typically give each other one kiss on the left cheek.
F. second sentence
_____ 65. Unlike in France and Mexico, Americans don’t kiss each
G. third sentence
other on the cheek when they meet; they prefer to shake
H. fourth sentence
hands or hug if they know each other well.
_____ 66. In many cultures, people kiss each other on the cheek
when they meet as a form of greeting.

Read the sentences for a narrative paragraph. Then number the sentences to show the correct
order.
_______ 67. The princess didn’t believe him, but she kissed him anyway. As soon as she did, he turned
into a handsome prince.
_______ 68. One day, she found a frog in her bedroom. The frog asked her to kiss him. He said he would
become a handsome prince.
_______ 69. Finally, the prince and the princess got married, and they lived happily ever after.
_______ 70. Many, many years ago, there was a princess who wanted to meet a handsome prince.

Read the steps for writing a defining paragraph. Then number the steps in the correct order.
Page 19 of 170
International University, IU
Test Level 3

_______ 71. Make sure the definition is clear.


_______ 72. Explain how the term or concept is different from similar terms.
_______ 73. Write a topic sentence that states the term or concept and defines it.
_______ 74. Write about the term or concept using explanations and examples.
_______ 75. Explain what the concept is not.

Read the sentences for an opinion paragraph. Then number the sentences to show the correct
order.
_______ 76. Cheap tennis rackets only cost around thirty dollars.
_______ 77. Tennis is a good sport to play.
_______ 78. First, it does not cost much money.
_______ 79. Tennis players strengthen their legs when running.
_______ 78-. Second, it is good for your body.
_______ 81. Running also helps exercise the lungs.

Complete the sentence with the correct conjunction.


but so and or

82. He is sick, ____________________he is staying home today.

83. Bhim has one brother ____________________ one sister.

84. Do you like dogs ____________________ cats more?

85. I called her, ____________________ she did not answer the phone.

Read the compound sentence. Choose the correct reason for using the underlined coordinating
conjunction.
86. The movie was terrible, so we decided to leave early.
E. related ideas
F. contrasting ideas
G. a result
H. a choice
87. I like reading the stories in the magazine, but it has too many ads.
A. related ideas
B. contrasting ideas
C. a result
D. a choice
88. Advertisers can make TV ads, and they can advertise on the Internet.
A. related ideas
B. contrasting ideas
C. a result
D. a choice

Complete the letter to the editor with the sentences below. Write the letter next to the correct
sentence.
Dear Editor,
My eight-year-old daughter loves to watch her favorite shows on the weekend. I don’t mind her watching
a little TV to relax, and I often sit with her so we can enjoy the shows together. However, I am getting
tired of having to wait through all the ads to see what happens next. Recently, I watched a show with my
Page 20 of 170
International University, IU
Test Level 3

daughter, and there were more than ten ads! (a) _____.
(b) _____. For example, they do not know that ads make them want something they don’t need. Just
yesterday, my daughter asked to go to the store to get her a game that was advertised during her favorite
show. She got upset when I tried to explain that she didn’t really need the game.
(c) _____. The ads are usually much louder than the shows children are watching. (d) _____. Imagine the
damage a lot of ads could do to children’s hearing!
My daughter is a smart girl, but she is not able to understand the real power behind advertising. And I
worry about loud ads damaging her hearing. (e) _____. That is why I think there should be no more ads
played during children shows.
Sincerely,
A Concerned Parent
_______ 89. Every time an ad comes on, I have to turn down the volume so it doesn’t hurt my own
ears.
_______ 90. I do not think television stations should play ads during kids’ shows.
_______ 91. My other concern is that today’s ads may be dangerous for children’s ears.
_______ 92. Advertising can confuse and harm children.
_______ 93. First of all, children aren’t old enough to understand how ads work.

Complete the problem-solution essay with the sentences below. Write the letter next to the correct
sentence.
The 3 Rs
There’s simply too much waste on our planet, and almost all of it is generated by human beings. We need
to decrease the amount of trash we produce. (a) _____.
(b) _____. People throw away millions of plastic water bottles each year, for example. But many of us
could buy just one metal or hard plastic bottle and use it again and again. It is an easy way to reduce the
number of plastic bottles that end up in the trash.
(c) _____. Instead of throwing away old plastic shopping bags, we can find alternative uses for them. For
example, we can reuse them as garbage bags in small wastebaskets or while in our cars. Not only are they
effective, but they are also free.
(d) _____. Many cities have recycling programs for glass, plastic, and paper items. This old “trash” can
be turned into new products with little effort. People can also recycle electronic devices, such as cell
phones and computers. They can be fixed and resold, and they benefit other people and the planet.
(e) _____. Reducing, reusing and recycling waste are simple ideas that can have enormous effects. If
everyone worked together to follow these rules, the world would be a much better place to live.
______ 94. People can also decrease waste by reusing items they already have.
______ 95. One of the easiest ways to help our planet is by reducing the amount of trash we produce.
______ 96. In all, when people stick to the three Rs, they produce less waste and help keep our planet a
cleaner and healthier place to live.
______ 97. We can cut down the amount of waste we make if we stick to three basic rules: reduce,
reuse, and recycle.
______98. In addition, recycling can greatly cut down on waste.

Complete the cause/effect essay with the sentences below. Write the letter next to the correct
sentence.
Random Acts of Kindness
We are usually kind to the people we love. Many people are reluctant to help strangers, however. We
should all try to be more kind to people in need. People should try to do one random act of kindness for a
stranger a day. (a) _____.
Page 21 of 170
International University, IU
Test Level 3

(b) _____. It can be something as simple as holding the door open for a stranger. It can be as thoughtful
as telling someone they dropped money on the floor. When I go grocery shopping, I try to find ways to
help other shoppers. (c) _____. When I leave the store, I offer to put back another person’s cart. These are
simple things to do, but they help others. People are usually thankful, because they never expected the
help.
(d) _____. Tal Ben-Shahar, a psychology teacher at Harvard University, found that giving help makes
people feel good about themselves. He asked his students to do five small acts of kindness a day, such as
giving change to a homeless person or being nice to waiters at a restaurant. When they did these acts, he
found that his students were happier as a result.
(e) _____. It helps the giver and the receiver both. I have already promised myself to do one act of
kindness each day, and I hope others will, too. It doesn’t take much effort. Every time you go out, just
look for little things you can do for other people, and you will very happy with the results.
_______ 99. Then they can help both other people and themselves.
_______ 100. When someone can’t reach an item, I offer to get it for them.
_______ 101. A random act of kindness can also benefit the person giving help.
_______ 102. A random act of kindness is a small task that helps others.
_______ 103. In all, everyone benefits from a random act of kindness.

Complete the opinion essay with the sentences below. Write the letter next to the correct sentence.
Don’t Make Me Text
It seems like everyone today loves to text. In fact, many teenagers prefer it to a conventional phone call.
You can hear and see people texting all around you—in stores, at work, and even in their cars. (a) _____.
I disagree with them. I much prefer calling my family and friends, because it safer, cheaper, and more
personal.
(b) _____. When I need to talk to someone while I’m driving, I always talk over the phone. That way, I
can see what is happening around me. When people text, they have to look down at their phones for a
long time, which takes their eyes off the road. This has caused many accidents. With a traditional phone
call, you can avoid these kinds of accidents.
(c) _____. If people don’t text, they don’t really need a cell phone. They can just use a home phone and
only have one phone bill. People who use their cell phones to text message often have to pay more money
for the service. If they go over their limit, they can be charged a lot of money for the extra texts.
(d) _____. I love getting phone calls from my family because I like hearing their voices. I always
recognize my mother or best friend just by hearing them say “hi” when I answer the phone. That personal
touch gets lost when people text. Everyone’s “hi” looks the same on a cell phone screen. Friends might
respond with LOL if I text something funny, but it is nothing compared to hearing their real laugh.
(e) _____. When people just use a phone, they can avoid car accidents, save money, and have more
personal communication with their family and friends. I do not think texting will ever go away, but
maybe soon more people will long for the good old days of a simple phone call.
_______104. Although texting is popular, these reasons have shown calling is just as good, and
often better.
_______105. In addition, phone calls are cheaper.
_______106. First of all, phone calls are much less dangerous than texting.
_______107. People who text say that it is an easier way to communicate.
_______108. Another reason I prefer calling to texting is because it is more personal.

Page 22 of 170
International University, IU
Test Level 3

PART II: Questions 1-51


Read each paragraph. Then choose the best topic sentence and write it on the line provided.
(1). ___________________________________________. I usually go skiing every weekend in the
winter even though it is expensive. I love the feeling of flying down a mountain. The views are beautiful
from the top of a mountain and along the trails. Even the danger of falling and getting hurt can’t keep me
away from the slopes on a winter day.
a. Skiing is expensive.
b. I love the feeling of flying down a mountain.
c. Skiing is my favorite sport.
d. Skiing is dangerous.

(2). __________________________________________. North Americans send cards for many


occasions. They send cards to family and friends on birthdays and holidays. They also send thank-you
cards, get well cards, graduation cards, and congratulation cards. It is very common to buy cards in stores
and send them through the mail, but turning on the computer and sending cards over the Internet is also
popular.
a. Birthday cards are the most popular kind of card.
b. Sending cards is more important.
c. It is important to send thank-you cards.
d. Sending cards is very popular in North America.

(3). __________________________________________. I enjoy summer sports like water skiing and


baseball. The weather is usually sunny and hot, so I can go to the beach almost every day. Gardening is
my hobby and I spend many summer days working in my garden. Unfortunately, the days pass too
quickly in summer.
a. Gardening is my hobby.
b. I like to garden in summer.
c. Summer is my favorite season.
d. Summer is to short.

(4). __________________________________________. First of all, we need money to repair old roads


and build new roads. We also need more to pay teachers’ salaries and to pay for services such as trash
collection. Finally, more tax money is needed to give financial help to the poor citizens of the city. It is
clear that the city will have serious problems if taxes are not raised soon.
a. Taxes needed to give financial help to the poor citizens of the city.
b. We should raise city taxes.
c. City taxes are too high.
d. City taxes pay for new roads.

(5). _______________________________________. For example, a person can have breakfast in New


York, board an airplane, and have dinner in Paris. A businesswoman in London can instantly place an
order with a factory in Hong Kong by sending a fax. Furthermore, a schoolboy in Tokyo can turn on a TV
and watch a baseball game being played in Los Angeles.
a. Business people can instantly place orders easily.
b. Airplanes have changed our lives.
c. Advances in technology have made the world seem smaller.
Page 23 of 170
International University, IU
Test Level 3

d. The fax machine was an important invention.

(6). ______________________________________. One thing you must consider is the quality of the
university’s educational program. You also need to think about the school’s size and location. Finally,
you must be sure to consider the university’s tuition to make sure you can afford to go to school there.
a. There are many colleges and universities in the United States.
b. It is expensive to attend a university in the United States.
c. There are several factors to consider when you choose a university to attend.
d. You should consider getting a good education.

(7). _________________________________________. Jim likes to play football. His sister plays


baseball. His brother is on a basketball team. His parents like to ski in the winter.
a. Jim has a large family.
b. Jim likes sports for some reasons.
c. Jim's family enjoys sports.
d. Playing sports is good exercise.

(8). _________________________________________. My school has recess every day. I like to play on


the swings and jungle gym with my friends. We like to laugh and run. Recess is my favorite time of the
day.
a. Recess is my favorite time of the day.
b. Recess is when I can have fun with my friends.
c. Some kids bring their toys from home for recess.
d. Parents sometimes bring their small children during recess for a visit.

(9). ________________________________________. I play basketball every weekend. My friends and I


play it at recess and after school. My mom took me to a basketball game. I got to see Michael Jordan
play. That was the greatest because basketball is my favorite game.
a. Recess is my favorite time of the day.
b. Michael Jordan was a great basketball player.
c. Some people play both football and basketball.
d. Basketball is my favorite game.

(10). ________________________________________. Matt likes dogs and playing soccer. Claire likes
cats and playing tennis. Matt likes to eat pizza. Claire's favorite food is spaghetti.
a. Matt and Claire like food.
b. Matt and Claire like animals.
c. Pizza and spaghetti taste good.
d. Matt and Claire like different things.

(11). _______________________________________. It took our bus four hours to get to Sea World. We
saw whales, seals, and sharks. There were also many fish in tanks.
a. Our class took a trip.
b. One of my favorite things was the whale.
c. I ate lunch outside with my class.
d. We like a trip.

Page 24 of 170
International University, IU
Test Level 3

(12). ______________________________________. Georgia has a new flag. The flag has the state seal.
South Carolina's flag has a tree and the moon. The flag of Wyoming has a buffalo.
a. California's flag has a bear.
b. The flag of Wyoming has a buffalo.
c. Every state has a different flag.
d. Georgia's flag is very interesting.

(13). ___________________________________________________________. First, living in a foreign


country helps you learn another language faster than studying it at school. Second, you can learn directly
about the history, geography, and culture of a country. Third, you become a more tolerant person because
you experience different ways of living. Fourth, living in a foreign country makes you appreciate your
own country better.
a. Living in a foreign country helps you learn.
b. Everyone should live in a foreign country for a while.
c. Living in a foreign country has four benefits.
d. Everyone becomes tolerant.

(14). _______________________________________________________________________. Some


colleges and universities in the United States are private. Private colleges and universities do not get
money from taxes, so they are usually more expensive. Other colleges and universities are public; that is,
the citizens of each state pay some of the costs through their taxes. As a result, public colleges are cheaper
for students to attend. No matter which type of college you attend-public or private, you can get a good
education.
a. Private colleges and universities do not get money from taxes
b. Public colleges and universities get money from taxes.
c. There are many colleges and universities in the United States.
d. There are two main types of colleges and universities in the United States.

(15). ______________________________________________________________. One reason for


choosing a small college is that classes are small. The average class in a small college is twenty students.
Another reason is that it is easy to meet with professors. Professors in small colleges have time to help
students and are usually happy to do so. In addition, small colleges are friendly, so new students make
friends quickly. For these three reasons, small colleges are better than large universities for many
students.
a. Professors in small colleges have time to help students
b. Small colleges are friendlier than large universities.
c. There are several reasons for attending a small college instead of a large university.
d. You can get an excellent education at a small college.

(16). ______________________________________________________________. First of all,


employers want workers to be dependable. That is, they want workers who come to work every day.
Second, employers want workers who are responsible. Can the boss give the worker a project to do and
know that it will be done well? Third, employers look for workers who can work well with others. The
ability to get along with co-workers is important to the success of a business. To summarize, employers
look for dependable, responsible team players.
a. The ability to get along with co-workers is important to the success of a business.
b. It is difficult to find good employees these days.
c. Employers read job applications very carefully.
Page 25 of 170
International University, IU
Test Level 3

d. Employers look for three main qualities in their employees.

(17). _______________________________________________. Green curry is the hottest Thai curry.


People who like very spicy food will enjoy green curry. Red curry is medium hot. It is flavorful, but it
doesn't burn your mouth. Yellow curry is the mildest of all. It is usually the choice of people who eat Thai
food for the first time. In short, you have three delicious choices when you order Thai curry.
a. People who like very spicy food will enjoy green curry.
b. It is usually the choice of people who eat Thai food for the first time.
c. Thai food is delicious.
d. There are three delicious choices for Thai curry.

(18). _________________________________________________. First, good teachers know their


subject very well. That is, a math teacher has advanced education in mathematics, and an English teacher
is an expert in English grammar. Second, good teachers must be good communicators. Being a good
communicator means presenting information in ways that students can understand. Third, good teachers
are enthusiastic. That is, they are so interested in their subject that they make it fun to learn. To
summarize, good teachers have expert knowledge, good communication skills, and contagious
enthusiasm.
a. English teachers are experts in English grammar.
b. Good teachers are friendly and approachable.
c. Good teachers have some qualities.
d. Good teachers build knowledge of their subjects.

(19). __________________________________________________. The first type of shopper doesn't


like to waste time. She knows what she wants to buy and how much she wants to pay. If the store has
what she wants, she buys it and leaves. She is a good kind of customer because she doesn't take too much
of a salesperson's time. A second type of shopper comes into a store with a general idea of what she
wants, listens to the salesperson's suggestions, tries on a few items, and makes a decision. She is also a
good kind of customer. A third kind of shopper has no idea what she wants. She spends two hours trying
on one outfit after another. She takes up a lot of a salesperson's time and sometimes doesn't buy anything.
In conclusion, the first two types of shoppers are a salesperson's dream, but the third type is a
salesperson's nightmare.
a. The shopper doesn't like to waste time.
b. The shopper comes into a store with a general idea of what she wants, listens to the salesperson's
suggestions, tries on a few items, and makes a decision.
c. The shopper has no idea what she wants.
d. There are different types of shoppers.

(20). ___________________________________________________. First of all, teachers must know


their subjects very well. At a minimum, they should take several college courses and pass a written test in
every subject that they teach. Also, they should take refresher classes every few years to keep their
knowledge up-to-date. Third, teachers should take education classes in college to learn how to teach.
Fourth, they should have spent at least one year practice teaching. Practice-teaching is teaching real
children in a real classroom under the supervision of an experienced teacher. Only after a person has met
these requirements should he or she receive a teaching license.
a. Teachers must know their subjects very well.
b. Teachers are friendly and approachable.
c. There are some requirements to be teachers.
Page 26 of 170
International University, IU
Test Level 3

d. Teachers build knowledge of their subjects.

(21). ___________________________________________________. Even small towns in the United


States have at least one pizzeria and one Chinese restaurant. Every midsize town has at least one taqueria,
where you can get a delicious Mexican taco or burrito. French food has always been popular, and hot
dogs and hamburgers, German in origin, are found everywhere. More recently, Middle Eastern shish
kebab, Japanese sushi, and English fish and chips are increasingly available in the United States.
a. French food has always been popular.
b. Middle Eastern shish kebab, Japanese sushi, and English fish and chips are increasingly available in the
United States.
c. Mexican taco or burrito is popular.
d. There are different kinds of foods available in the United States.

(22). ___________________________________________________. Some people skip breakfast


because they think it will help them lose weight. Another reason people give is that they simply don't like
breakfast. For others, the reason is cultural. People in some cultures consume only two meals each day
instead of three, and breakfast isn't traditionally one of them. The most common reason people give is
lack of time. They like to stay in bed until the last minute, and then they have to rush to get to work or to
school on time.
a. The most common reason people give is lack of time.
b. Some people skip breakfast because they think it will help them lose weight.
c. Lunch is the most important meal.
d. Breakfast is not important.

(23). __________________________________________________. English speakers relaxing at home,


for example, may put on kimonos, which is a Japanese word. English speakers who live in a warm
climate may take an afternoon siesta on an outdoor patio without realizing that these are Spanish words.
In their gardens, they may enjoy the fragrance of jasmine flowers, a word that came into English from
Persian. They may even relax on a chaise while snacking on yogurt, words of French and Turkish origin,
respectively. At night, they may shampoo their hair and put on pajamas, words from the Hindi language
of India.
a. Relaxation is more important for English speakers.
b. English speakers relaxing at home, may put on kimonos, which is a Japanese word.
c. English speakers like doing different things for their daily routines.
d. English is originated from different languages.

(24). __________________________________________________. In European universities, students


are not required to attend classes. In fact, professors in Germany generally do not know the names of the
students enrolled in their courses. In the United States, however, students are required to attend all classes
and may be penalized if they do not. Furthermore, in the European system, students usually take just one
comprehensive examination at the end of their entire four or five years of study. In the North American
system, on the other hand, students usually have numerous quizzes, tests, and homework assignments,
and they almost always have to take a final examination in each course at the end of each semester.
a. Students are not required to attend classes.
b. Students are required to attend all classes.
Page 27 of 170
International University, IU
Test Level 3

c. Attending the class is not important.


d. There are some different educational systems between European universities and the United States.

(25). __________________________________________________. Trail climbing is the easiest.


Climbers just walk along trails to the top of a mountain. The trails are not very steep, and the mountains
are small. The second type, rock climbing, takes place on steeper slopes and bigger mountains. Climbers
generally have to use special equipment such as climbing shoes, ropes, and metal nails called pitons. The
third type is ice climbing. Ice climbing takes place only on very high mountains and requires a lot of
special equipment. Equipment used in ice climbing includes ice axes and crampons, which are spikes
attached to a climber's boots for walking on ice and hard snow. Indeed, the sport of mountain climbing
can range from an easy uphill walk to a difficult climb up a frozen waterfall.
a. Mountain climbing requires special skills and equipment.
b. Climbers generally have to use special equipment.
c. The sport of mountain climbing is practiced worldwide.
d. There are three main types of mountain climbing.

(26). _________________________________________________. For example, Kansas City, in the very


center of the United States, is known for its beef, and Kansas City barbecue is everyone's favorite way to
enjoy it. In Boston, people love baked beans. In the Southwest, chili, a stew made of meat, beans,
tomatoes, and hot peppers, is the regional dish. Wisconsin, a state with many dairy farms, is famous for
its cheese. Go to Maryland and Virginia for crab cakes and to the Northeast for clam chowder and maple
syrup. Indeed, many U.S. cities and regions have a special food for everyone to enjoy.
a. There is a variety of food in the United States.
b. Food in the United States varies from sweet desserts to spicy stews.
c. Different regions of the United States have their own traditional foods.
d. Food in the United States is quite delicious.

(27). ___________________________________________________. Before the 2004 tsunami in the


Indian Ocean, water buffalo stampeded' in Thailand, and dogs in Sri Lanka refused to go out for their
regular walks. An unusual number of pets ran away from their homes in the days before the 1989 San
Francisco earthquake. Japanese researchers have analyzed fishermen's stories about the abnormal
behavior of fish in the days or hours before earthquakes in that country. These are just a few examples of
strange animal behavior just before earthquakes occur.
a. Dogs, elephants, water buffalo, and fish can predict earthquakes.
b. Earthquake prediction is an important science.
c. Animals may be able to sense earthquakes before they happen.
d. Japanese researchers have analyzed fishermen's stories.

Paragraph unity
Read the following paragraphs. Some sentences break the rule of unity. How many sentence(s) is/are
(not) relevant?

Page 28 of 170
International University, IU
Test Level 3

(28). California is a state with every type of geography. It has mountains where you can enjoy the
winter sports of skiing, snowboarding, and snowshoeing. It has deserts where temperatures can reach
110°F (43°C) in the summer. It has beaches where you can surf, swim, and fish. It has forests where the
world's tallest trees grow. Finally, it has farmland where a lot of the nation's fruits and vegetables grow.
California also has Hollywood and Disneyland, which are world-famous centers of entertainment.
California is a popular place to live, so many people move there every year. Indeed, California has it all:
mountains, deserts, beaches, and farms.
a. None
b. 1
c. 2
d. 3

(29). A nurse should have at least five characteristics. First, he or she must be a caring person. He or
she must have genuine concern about sick, injured, frightened people. Second, a nurse must be organized.
If a nurse forgets to give a patient his or her medicine on time, the consequences could be serious. Third,
a nurse must be calm. He or she may have to make a life and-death decision in an emergency, and a calm
person makes better decisions than an excitable one. Doctors need to stay calm in emergencies, too. In
addition, a nurse should be physically strong because nursing requires a lot of hard physical work.
Finally, a nurse must be intelligent enough to learn subjects ranging from chemistry to psychology and to
operate the complex machinery used in hospitals today. There is a shortage of nurses today, so they earn
good salaries. In brief, nursing is a profession for people who are caring, organized, calm, strong, and
smart.
a. None
b. 1
c. 2
d. 3

(30). There is a canyon in northern Arizona that is the most beautiful spot on Earth. It is called Havasu
Canyon, and it is part of the Havasupai tribal reservation. It is not easy to get there, for you have to hike
down a long, hot trail. At the end of the trail is Supai Village. The Havasupai are a tribe of about 650
people. Their language has been written down only in the past twenty years. Beyond the village, another
trail leads to the top of a steep cliff overlooking Havasu Canyon. Your first view of the canyon takes your
breath away. Directly in front of you, the trail disappears straight down the 200-foot cliff. On your right,
you see a beautiful waterfall. Water pours straight down into a bright blue-green pool at the bottom of the
canyon. Directly across the canyon, hundreds of small waterfalls gush from the cliff face, and little green
ferns grow everywhere. At the bottom, the water cascades from one turquoise pool into another until it
disappears into the trees on the left. As you view this scene, you can only think that Havasu Canyon is
truly a magical place.
a. None
b. 1
c. 2
d. 3

(31). My first apartment was very small. It was a studio apartment, so it had only one main room and a
bathroom. The main room was divided into three areas. At one end of it was a kitchenette, where I cooked
and ate my meals. My living/sleeping area was at the opposite end. I had just enough space for a bed, a
coffee table, a floor lamp, and a small television. My study area was against the back wall. I lived there

Page 29 of 170
International University, IU
Test Level 3

for two years, but I moved because my landlord raised the rent. My apartment was so small that I could
never invite more than three friends at the same time!
a. None
b. 1
c. 2
d. 3

(32). A good ad has three characteristics. First of all, a good ad is simple. It lets pictures, not words, tell
the story. Of course, all ads need some words, but a good ad has a powerful headline and only a small
amount of text. Second, a good ad is directed to a particular group of consumers. For example, ads for
face creams are for older women, and ads for motorcycles are for unmarried young men. Third, a good ad
appeals to emotions. Women in the thirty-to-fifty age group, for instance, want to look and feel younger,
so face cream ads tell them that women who use XYZ face cream will look like the twenty-year-old
models pictured in the ad. Teenagers want to feel popular, so ads directed at teens often show a happy,
confident-looking group of young people using the product in the ad. Teenagers have a surprising amount
of money to spend, so advertisers research teenage fads and fashions. In conclusion, good ads are simple,
are directed at a specific group, and make an emotional connection.
a. None
b. 1
c. 2
d. 3

(33). Artichokes are vegetables that look like green pinecones but taste delicious. If you have never
eaten one, you may not know how or where to start. Lobster is another food that is difficult to eat. Here's
how to cook and eat an artichoke. First, cut off the stem with a knife. Then cut off the end of each leaf
with scissors. Steam the whole artichoke in a pot of water until it is soft. To eat it, pull off one leaf at a
time, hold the cut end, and dip the leaf into melted butter or mayonnaise. Then put the dipped end into
your mouth. Pull the leaf through your teeth and scrape off the soft part. Throw away the tough part.
Continue doing this until all of the leaves are gone. Next, take a spoon, scrape out the fuzzy part in the
center, and throw it away. A lobster also has parts that you don't eat. The remaining part of the artichoke-
the bottom - is the heart, which is the best part. Cut the heart into pieces, dip each piece into the sauce,
and enjoy the delicious taste.
a. None
b. 1
c. 2
d. 3

(34). Kimchi, or kimchee, is Korea's national dish. It is made of fermented' vegetables. There are many
ways to make Kimchi, but it usually contains Chinese cabbage, salt, garlic, red pepper, green onion, fish
sauce, and ginger. These ingredients are mixed together, put into a container, and allowed to ferment for
three or four days. Nutritionists say that Kimchi is very good for you. In fact, the U.S. magazine Health
says it is one of the five healthiest foods because it has a lot of vitamins and because it helps digestion. It
may even prevent cancer. The other four healthy foods are yogurt, olive oil, lentils, and soy. Kimchi is
very spicy and has a very strong taste. Most Westerners have to get used to the taste, but Koreans adore it.
They eat it with every meal, either alone or mixed with rice or noodles. They also use it in soup,
pancakes, and even as a pizza topping.
a. None
b. 1
c. 2
d. 3

Page 30 of 170
International University, IU
Test Level 3

(35). Colors create biological reactions in our bodies. These reactions, in turn, can change our behavior.
In one study, prisoners were put in a pink room, and they underwent a drastic and measurable decrease in
muscle strength and hostility within 2.7 seconds. In another study, athletes needing short bursts of energy
were exposed to red light. Their muscle strength increased by 13.5 percent, and electrical activity in their
arm muscles increased by 5.8 percent. Athletes needing more endurance for longer performances
responded best when exposed to blue light. Other studies have shown that the color green is calming.
Green was a sacred color to the Egyptians, representing the hope and joy of spring. It is also a sacred
color to Moslems. Many mosques and religious temples throughout the world use green (the color of
renewal and growth) and blue (the color of heaven) to balance heavenly peace with spiritual growth. To
sum up, color influences us in many ways (Daniels 10).
a. None
b. 1
c. 2
d. 3

(36). Colors create biological reactions in our bodies. These reactions, in turn, can change our behavior.
In one study, prisoners were put in a pink room, and they underwent a drastic and measurable decrease in
muscle strength and hostility within 2.7 seconds. In another study, athletes needing short bursts of energy
were exposed to red light. Their muscle strength increased by 13.5 percent, and electrical activity in their
arm muscles increased by 5.8 percent. Athletes needing more endurance for longer performances
responded best when exposed to blue light. Other studies have shown that the color green is calming.
After London's Blackfriars Bridge was painted green, the number of suicides decreased by 34 percent.
These and other studies clearly demonstrate that color affects not only our moods but our behavior as well
(Daniels 10).
a. None
b. 1
c. 2
d. 3

(37). Colors create biological reactions in our bodies. These reactions, in turn, can change our behavior.
In one study, athletes needing short bursts of energy were exposed to red light. Their muscle strength
increased by 13.5 percent, and electrical activity in their arm muscles increased by 5.8 percent. Athletes
needing more endurance for longer performances responded best when exposed to blue light. Blue is not a
good color for dinnerware, however. Food looks less appetizing when it is served on blue plates, perhaps
because very few foods in nature are of that color. Other studies have shown that the color green is
calming. After London's Blackfriars Bridge was painted green, the number of suicides from it decreased
by 34 per cent. It is clear that color affects not just our moods, but our behavior as well (Daniels 10).
a. None
b. 1
c. 2
d. 3

(38). Adventure travel is the hot trend in the tourism industry. Ordinary people are no longer content to
spend their two weeks away from the office resting on a sunny beach in Florida. More and more often,
they are choosing to spend their vacations rafting down wild rivers, hiking through steamy rain forests,
climbing the world's highest mountains, or crossing slippery glaciers. People of all ages are choosing
educational study tours for their vacations.
a. None
b. 1
c. 2
d. 3
Page 31 of 170
International University, IU
Test Level 3

(39). Daredevil sports are also becoming popular. Young people especially are increasingly willing to
risk life and limb while mountain biking, backcountry snowboarding, or high-speed skateboarding.
Soccer is also popular in the United States now, although football is still more popular. One of the riskiest
new sports is skysurfing, in which people jump out of airplanes with boards attached to their feet.
Skysurfing rivals skydiving and bungee jumping for the amount of thrill sand risk.
a. None
b. 1
c. 2
d. 3

CONCLUDING SENTENCES
Read the following paragraphs. Then choose the best concluding sentence for it.

(40). There are two reasons I love big cities. First of all, big cities are alive 24/7. You can go shopping,
see a movie, exercise at a gym, get something to eat, or go roller skating at any time of the day or night.
Second, I love big cities because of their anonymity. You can be completely invisible in big cities. No one
watches your daily comings and goings.' Neighbors don't bother you as long as you don't bother them.
You can stay out all night or stay home all day, and no one cares.
a. To sum up, I love big cities because you can be independent.
b. In short, big cities attract me because there are many things to do.
c. In brief, I like big cities because of their energy and anonymity.
d. To sum up, you can stay out all night or stay home all day, and no one cares.

(41). There are two reasons I hate big cities. First of all, big cities are full of noise 24/7. You can hear
horns honking, traffic roaring, music blaring, and people talking at all hours of the day and night. It is
never quiet in a big city. Second, I hate big cities because of their anonymity. No one knows or cares
about you. Neighbors who have lived next door to each other for many years don't even know each others'
names. You can be very lonely in a big city.
a. In brief, big cities are noisy, lonely places to live.
b. In conclusion, I prefer to live in a small town, where it is quieter and people are friendlier.
c. Also, big cities have a lot of crime.
d. To sum up, I love big cities because you can be independent.

(42). Unless you live on an uninhabited island in the middle of a big ocean, you cannot escape
advertising. People in the modern world are continually exposed to ads and commercials on the radio, on
television, on billboards, in their mailboxes, and on their computers. However, advertising is not a
modern phenomenon. Advertising has been around for a long time, as a review of its history shows.
a. As early as 3000 B.C., merchants carved signs in wood, clay, and stone to put above their shops.
b. In ancient Egypt, merchants hired people called criers to walk through the streets announcing the
arrival of ships and their cargo.
c. In medieval Europe, shop owners hired criers to direct customers to their shops.
d. The invention of the printing press was the start of the advertising industry as we know it today.

(43). On my first day of class in an American university, I discovered that there are many differences
between universities in the United States and in my country. One difference hit me immediately when the
professor walked into the classroom dressed in casual pants and a sports shirt. Then he sat down, and I
received a second shock. He sat down on the desk, not behind the desk. The biggest shock happened
when he passed out a piece of paper listing the requirements of the class. I learned that I was not allowed
to miss any classes! In my country, professors do not know or care if students attend lectures, but in the
Page 32 of 170
International University, IU
Test Level 3

United States, professors actually call the roll at the beginning of each class meeting. In my opinion,
compulsory attendance in college is completely inappropriate for two reasons.
a. College students are adults, not elementary school children.
b. Students often have other obligations such as jobs and family.
c. Classes should be optional.
d. Students are not allowed to miss their classes.

(44). Everyone needs goals. Having goals makes you more successful because they keep your mind on
what is really important to you. However, goals can change at different times in your life. Your goals
when you are ten are very different from your goals at fifteen or twenty. My major goals this semester are
to get a part-time job and to master the use of the English language.
a. My first goal is to get a part-time job in an area related to my field of study.
b. I also plan to improve my ability to speak, write, read, and understand English.
c. Having goals makes you more successful because they keep your mind on what is really important to
you.
d. Goals are not important.

(45). Female airline pilots? Male nurses? When my parents were young, such job descriptions were not
possible. In the past thirty-five years, however, society has become more accepting. Although it is still
somewhat unusual, men now work in traditionally female occupations. In particular, more and more men
are becoming nurses, secretaries, and elementary school teachers.
a. The nursing profession has seen the greatest increase in male participation.
b. Besides nursing, more men are becoming secretaries.
c. Elementary school teaching is a third occupation that men are taking up.
d. Teaching is an important occupations for men.

(46). Goldfish have three characteristics that make them good pets. First of all, goldfish are very quiet.
They don't bark, howl, meow, chirp, squawk, screech, or race around the house at night while you and
your neighbors are trying to sleep. Second, they are economical. You can buy a goldfish at your local pet
store for about 50¢, and a small bowl for it costs less than $3.00. Water is practically free. Also, they eat
only a pinch of dried fish food daily, so their food bill is quite low. Third, goldfish are very well behaved.
They don't have teeth, so they can't chew your furniture or bite your guests. They don't ever go outside, so
they can't dig holes in your garden. In addition, you don't have to spend hours teaching them commands,
such as "Sit!" or "Lie down!"
a. In brief, you can buy a goldfish at your local pet store with less price.
b. All in all, goldfish are nice.
c. Having goldfish are more than other pets.
d. In summary, goldfish are quiet, economical, and well behaved.

(47). You can be a good conversationalist by being a good listener. When you are conversing with
someone, pay close attention to the speaker's words while looking at his or her face. Show your interest
by smiling and nodding. Furthermore, do not interrupt while someone is speaking; it is impolite to do so.
If you have a good story, wait until the speaker is finished. Also, watch your body language; it can affect
your communication whether you are the speaker or the listener. For instance, do not sit slumped in a
chair or make nervous hand and foot movements. Be relaxed and bend your body slightly forward to
show interest in the person and the conversation.
a. Showing your interest is important for making a conversation.
b. If you have a good story, wait until the speaker is finished.
c. A good conversationalist is understandable.
d. A good conversationalist is patient, flexible, and attentive to details.

Page 33 of 170
International University, IU
Test Level 3

(48). Modern communication technology is driving workers in the corporate world crazy. They feel
buried under the large number of messages they receive daily. In addition to telephone calls, office
workers receive dozens of e-mail and voice mail messages daily. In one company, in fact, managers
receive an average of 100 messages a day. Because they do not have enough time to respond to these
messages during office hours, it is common for them to do so in the evenings or on weekends at home.
(Compose)
a. Modern communication technology is more important.
b. Modern communication technology helps the world lots.
c. The world is really busy with modern communication technology.
d. Modern communication technology has both advantages and disadvantages.

(49). The island nation of Singapore, the smallest nation in Asia, is one of the best cities in the world to
live in and to visit. First of all, Singapore is a very safe city. It has a very low crime rate. Citizens and
tourists alike can walk the streets at night without fear. Also, Singapore is very clean. The nation has very
strong antilittering laws, so there is almost no litter anywhere. Third, Singaporeans come from many
different racial, ethnic, and religious backgrounds, but they live together in peace. Finally, Singapore's
economy is one of the healthiest in Asia, so its people enjoy a very high standard of living.
a. Singapore is a very safe city.
b. Singapore is very clean.
c. People enjoy a very high standard of living in Singapore.
d. Singapore is a good country in Asia.

(50). The college cafeteria is an inexpensive place to eat. For example, you can get a cheeseburger,
french fries, and a soda for only $3.00. A slice of pizza is only $1.50, and a cup of coffee is only 50¢.
There is a daily special for about $2.50. It includes an entree, rice or potatoes, and a vegetable. The salad
bar is the best deal of all. You get all you can eat for $2.00.
a. The college cafeteria is an expensive place to eat.
b. The college cafeteria is a truly lifesaver.
c. The college cafeteria is place where you can have everything with reasonable prices.
d. The college cafeteria is a place where you don’t need yourselves to keep busy with your own cooking.

(51). Animals living in modern zoos enjoy several advantages over animals in the wild. The first
advantage is that zoo animals are separated from their natural predators. They are protected, so they live
without risk of being attacked. Another advantage is that someone feeds them regularly, so they do not
have to hunt for food. Also, they do not suffer times when food is hard to find. A third advantage of living
in zoos is that veterinarians give animals regular checkups, and sick animals get prompt medical attention.
a. In conclusion, because all their needs are taken care of, most zoo animals are healthy and contented.
b. In conclusion, living in a zoo has many advantages for animals, but it also has some disadvantages.
c. In conclusion, zoos keep animals safe from predators.
d. In conclusion, zoos attract more visitors.

Page 34 of 170
International University, IU
Test Level 3

PART III:
Directions: Read the passage below. Then answer questions about errors in the passage.

Paragraph 1
My name is 1) Jay Hammond I am a firefighter. 2) I live in 128 Pine Lane, in Jackson,
Mississippi. 3) I have two childs. One is a girl named Clair. 4) The other 5) is boy named Thatcher. 6) His
name after my father. I also have a wife named Jenna. She is 7) beutiful. She has long, dark, soft hair. 8)
We also got a dog named Buck. He is very obedient but sometimes he barks at night and it upsets our
neighbors!
1)
A. Jay. Hammond I am
B. Jay Hammond. I am
C. Jay Hammond I’m
D. Jay Hammond, I am
2)
A. on 128 Pine Lane, on
B. in 128 Pine Lane, on
C. at 128 Pine Lane, in
D. in 128 Pine, Lane in
3)
A. two child
B. two children
C. two childrens
D. Correct as is
4)
A. Other
B. Another
C. The another
D. Correct as is
5)
A. is boy name
B. is boy of name
C. is a boy named
D. is a boy who name
6)
A. He is named after
B. He has name from
C. His name is same
D. He is same named
7)
A. butiful
B. beautyful
C. beautiful
D. beautifull
8)
A. got dog named Buck.
Page 35 of 170
International University, IU
Test Level 3

B. got a dog name Buck.


C. got dog name of Buck.
D. have a dog named Buck.

Paragraph 2
1) This morning, I were making toast for 2) brekfast. Then I went to the basement
3) put some clothes in the washer. I heard the smoke alarm. 4) I think, “Someone is
burning 5) something I put the soap in the washer. When I got 6) up to stairs, everybody
was running outdoors. Two fire trucks and a police car were outside of my house. Smoke
was coming out of my apartment! My kids 7) standing outside. They were 8) weering
their pajamas. My kids! Oh no! My toast!
1)
A. are
B. is
C. was
D. Correct as is
2)
A. brakfast
B. breckfast
C. breakfast
D. breackfast
3)
A. to put
B. for put
C. to puts
D. for putting
4)
A. Thinked
B. I thinked
C. I thought
D. I am thought
5)
A. something! I
B. something!” I
C. something”! I
D. something”? I
6)
A. upstairs
B. to upstairs
C. the upstairs
D. Correct as is
7)
A. was standing
B. are standing
C. were stand
Page 36 of 170
International University, IU
Test Level 3

D. were standing
8)
A. waring
B. wareing
C. wearing
D. Correct as is

Paragraph 3
My name is Thomas Martin. I live 1) in a farm near Cedar Rapids, Iowa.
2) I raises livestock and grow corn on my farm. My 3) kid help me. 4) In the morning, we
5) up very early to feed the animals. 6) Everybody help milk the cows. Then I drive the
kids to school. I work in 7) the fields all the day. When the children come 8) home, we all
work together.
1)
A. in farm
B. on farm
C. on a farm
D. Correct as is
2)
A. raise
B. raising
C. raised
D. Correct as is
3)
A. children
B. child
C. kids
D. All of the above are correct
E. Both A and B are correct
F. Both A and C are correct
4)
A. In morning,
B. In a morning,
C. On the morning,
D. Correct as is
5)
A. all up very early
B. wake up very early
C. are waking up very early
D. Correct as is
6)
A. Everyone helps
B. Everybody helps
C. All of the above are correct
Page 37 of 170
International University, IU
Test Level 3

D. None of the above are correct


7)
A. the fields during day.
B. the fields all day.
C. the fields day.
D. Correct as is
8)
A. home we all
B. home, us all
C. home, we are all
D. Correct as is

Paragraph 4
My name is Jacqueline. I am 1) a clerk at Dave’s Grocery Store. Sometimes I work
at a cash register. When 2) I no working there I work 3) in the service counter. The
grocery store has a 4) bank a flower shop, and a pharmacy. 5) There is an ATM 6)
between the entrance. The store is on Main Street. 7) All most everybody in town shops
8) to Dave’s.
1)
A. clerk at Dave’s
B. a clerk in Dave’s
C. a clerk at the Dave’s
D. Correct as is
2)
A. I not
B. I am not
C. I’m not
D. Both B and c are correct
3)
A. in the services
B. at the service
C. at service
D. Correct as is
4)
A. bank: a flower shop,
B. bank. a flower shop,
C. bank, a flower shop,
D. Correct as is
5)
A. There is a
B. Their is an
C. There are an
D. Correct as is
6)
A. between entrance.
Page 38 of 170
International University, IU
Test Level 3

B. near the entrance.


C. near entrance.
D. Correct as is
7)
A. All most every body
B. All most everyone
C. Almost everybody
D. Correct as is
8)
A. at Dave’s.
B. by Dave’s.
C. at the Dave’s.
D. Correct as is.

Paragraph 5
My name is Myuki. 1) Today is Saturday March 4. 2) I going shopping with my
daughter Misuzu. She is twelve years old. 3) We are going to buy clothes. Misuzu wants
4) new boots black and a new winter coat. 5) I need new 6) pair the gloves. We are also
going 7) for buy a birthday present for Misuzu’s friend. 8) Tommorrow is her birthday.
She is having a party.
1)
A. the Saturday of March 4.
B. Saturday March the 4.
C. Saturday, March 4.
D. Saturday March Four.
2)
A. I am going shopping
B. I am going shop
C. I shopping
D. I am shop
3)
A. are going buying
B. are go to buy
C. going buy
D. Correct as is
4)
A. new black boots
B. new boots blacks
C. new blacks boots
D. news blacks boots
5)
A. I needs a new
B. I need a new
Page 39 of 170
International University, IU
Test Level 3

C. Need a new
D. Need new
6)
A. pair of gloves.
B. pair of glove.
C. gloves a pair.
D. gloves the pair.
7)
A. to buy a birthday
B. to buy the birthday
C. for buy the birthday
D. Correct as is
8)
A. Tomorow
B. Tommorow
C. Tomorrow
D. Tomorogh

Paragraph 6
The university library 1) open every day. Students go to the library early in the
morning and 2) late at night some students 3) work in the library all night! The lights are
always on. There is a coffee shop 4) near to library. You can’t take coffee 5) go into the
library, but you can 6) take books into the coffee shop. The library workers 7) always
very busy. The students are always 8) studying. The library and the coffee shop are busy
places!
1)
A. open up
B. is open
C. is open up
D. is opens
2)
A. late at night. Some
B. late in night. Some
C. late in night. some
D. late night some
3)
A. work in library
B. work at library
C. work during library
D. Correct as is
4)
A. by the library
B. by to the library
Page 40 of 170
International University, IU
Test Level 3

C. near the library


D. Both A and C are correct
5)
A. for go into the library,
B. go into the library,
C. into the library,
D. into library,
6)
A. taking the books
B. taking books
C. take book
D. Correct as is
7)
A. are always very busy
B. are very always busy
C. were always very busy
D. always very are busy
8)
A. studing
B. studdying
C. stundying
D. Correct as is

Paragraph 7
Sam is an artist. He loves 1) paint and draw. He loves color. His 2) favrit color is
bright green. Sam 3) built his house. 4) He’s house has two bedrooms, a kitchen, a
bathroom, and one big room. 5) A big room is Sam’s art room. The art room has many
6) window larges. Sam 7) like paint in natural light. Of course, 8) Sams house is painted
bright green!
1)
A. the paint and draw.
B. do paint and draw.
C. to do the paint and draw.
D. to paint and draw.
2)
A. favorit
B. faverit
C. favorite
D. faverite
3)
A. builded
B. had builded
C. was builded
D. Correct as is
Page 41 of 170
International University, IU
Test Level 3

4)
A. He house has
B. His house has
C. Him house has
D. He is house has
5)
A. An big room
B. The big room
C. This big room
D. Correct as is
6)
A. large windows
B. windows large
C. large window
D. Correct as is
7)
A. likes to paint
B. like to paints
C. likes the painting
D. Correct as is
8)
A. Sam’s house
B. the Sam’s of house
C. the Sam house
D. the house that Sams

Paragraph 8
Dana and Mahmoud are married. Dana likes 1) sing and dance. Mahmoud likes to
read and write. Dana 2) enjoys talking 3) in the telephone. Mahmoud likes to play
4) chess Dana is studying drama 5) in the university. Mahmoud is studying
6) mathematic. Dana cooks big meals. Mahmoud eats big meals. Dana and Mahmoud like
7) difference things. 8) They love each the other.
1)
A. the sing and dance.
B. to sing and dance
C. singing and the dance.
D. Correct as is
2)
A. enjoys the talking
B. is enjoying talks
C. enjoys to talk
D. Correct as is
3)
Page 42 of 170
International University, IU
Test Level 3

A. on the telephone.
B. with the telephone.
C. at the telephone.
D. for the telephone.
E. Correct as is
4)
A. chess, so Dana
B. chess, but Dana
C. Chess, Dana
D. Chess. Dana
5)
A. at the university.
B. of the university.
C. on the university.
D. about the university.
E. around the university.
6)
A. mathematics
B. mathmatique
C. matemathics
D. mathmatics
7)
A. the different things.
B. difference things.
C. different things.
D. things that differ.
8)
A. Each loves the another.
B. They each loves other.
C. They loves each other.
D. They love each other.

Paragraph 9
The post office is 1) in downtown. There are 2) a lots of people at the post office. 3)
They are wait in line. Some people 4) wants to buy stamps. 5) Others people want to mail
letters. Some people want to send 6) packeges. Some people want to pick up their mail.
7) There are’nt many parking spaces near the post office. There are a lot of cars in front
of the post 8) office, the drivers are waiting for parking places and honking their horns.
1)
A. downtown.
B. at downtown.
C. in the downtown.
D. at the downtown.
2)
A. lot people
Page 43 of 170
International University, IU
Test Level 3

B. lots people
C. a lot of people
D. alot of people
3)
A. They be waiting
B. They are waiting
C. They is waiting
D. They was waiting
E. They were waiting
4)
A. want buy
B. want to buy
C. want to be buying
D. wants to buying
5)
A. Other people
B. Other peoples
C. Others of the people
D. Another people
6)
A. package
B. packages
C. paccages
D. pakages
7)
A. Their arent
B. They’re ar’ent
C. There aren’t
D. Correct as is
8)
A. office, so the
B. office, but the
C. office. The
D. Correct as is

Paragraph 10
Tomorrow I 1) is going to sing a solo. I’m so 2) nervouse! There are so many
things that could go wrong. I’m worried that my voice will shake. 3) Maybe I going to
open my mouth and no sound will 4) comes out. What if I can’t sing the high 5) notes
What if I 6) trip and fell down? What if I fall off the stage? What if I sound 7) terible? I
hope I can 8) do. I think I can.
1)
A. are going to
B. will going to
Page 44 of 170
International University, IU
Test Level 3

C. am going to
D. Both A and C are correct.
2)
A. nervis
B. nervose
C. nerveus
D. nervous
3)
A. Maybe I am going to open
B. Maybe I gonna open
C. Maybe I will open
D. Maybe I can open
4)
A. come out.
B. came out.
C. be coming out.
D. go out.
5)
A. notes. What
B. notes, what
C. notes? What
D. notes, but what
6)
A. trip and fall
B. trip and fell
C. trip and fall down
D. Both A and C are correct.
7)
A. terable
B. terrable
C. terrible
D. tearible
E. Correct as is
8)
A. do it.
B. did it.
C. done.
D. done it.

Paragraph 11
Susanna came home 1) from a work. She 2) putted the key in the lock of the
3) apartament door. She opened the door. She clearly heard a 4) voise inside her
apartment. Was it the TV? Was it the radio? Was it her neighbor? 5) She not know if she
should 6) go in or run away! She couldn’t move. She 7) couldnt think. She heard the soft
Page 45 of 170
International University, IU
Test Level 3

sound of footsteps. She couldn’t breathe. The door slowly opened. “Mom! What are you
doing 8) here” Susanna said, when she caught her breath. “Hi Honey! Dad and I are
cooking dinner for you!”
1)
A. from work
B. from the work
C. from the working
D. Correct as is
2)
A. put key
B. put the key
C. putted a key
D. putted the key
3)
A. apartment
B. apartement
C. apartamente
D. apartemente
4)
A. boise
B. boce
C. voice
D. voce
5)
A. She did not knew
B. She didn’t knew
C. She was not knowing
D. She didn’t know
6)
A. go out
B. go up
C. go through
D. Correct as is
7)
A. couldn’t think
B. not could think
C. could not think
D. Both A and C are correct.
8)
A. here?”
B. here”?
C. here”.
D. here”!

Paragraph 12
Page 46 of 170
International University, IU
Test Level 3

Where is that music coming 1) from Emma looks around. She sees women’s
clothing to her right. 2) she sees men’s clothing to her left. She sees 3) childrens clothing
4) in front her. It sounds like someone is 5) plaing a piano. Emma walks down the aisle of
the department store. The music gets louder and louder. 6) She enter the jewelry
department. The music gets even louder. 7) There is it! A woman is playing the piano in
the jewelry department! 8) It sound so nice!
1)
A. from? Emma
B. from! Emma
C. from. Emma
D. from, Emma
2)
A. She See
B. She see
C. She Sees
D. She sees
3)
A. childs clothing
B. childs clothings
C. children’s clothing
D. children’s clothings
4)
A. at front of her
B. in the front of her
C. on front of her
D. in front of her
5)
A. plaeing
B. playing
C. plaeying
D. playeing
6)
A. is entering
B. are entering
C. enters
D. Correct as is
7)
A. There it is!
B. Their it is!
C. They’re it is!
D. Correct as is
8)
A. Sounds so nice!
B. It is sounding so nice!
C. It sounds so nice!
D. Correct as is

Page 47 of 170
International University, IU
Test Level 3

Paragraph 13
Some days are good and some days are bad. Today is 1) day bad. I woke up 2)
very too late. My alarm clock 3) not ring. I ran to take 4) a shower. I didn’t have 5) a
towel I ran to find a towel. I got dressed. I couldn’t find my socks. I 6) finally found my
shoes and socks. I ran to the corner to catch the bus. 7) He was leaving. I slowly walked
home. Then I saw that my socks were two different colors. I am going back 8) for bed.
1)
A. bad day
B. a bad day
C. a day bad
D. the day bad
2)
A. too much late.
B. much too late.
C. too late.
D. Both B and C are correct.
3)
A. not rang
B. didnt ring
C. didn’t ring
D. didn’t rang
4)
A. shawer
B. schower
C. schawer
D. Correct as is
5)
A. towel. I
B. a towel. I
C. towel, I
D. a towel, I
6)
A. found final my shoes
B. final found my shoes
C. found finally my shoes
D. Correct as is
7)
A. It
B. She
C. This
D. They
8)
A. to bed
B. at bed
C. in the bed
D. to the bed

Page 48 of 170
International University, IU
Test Level 3

Paragraph 14
Yesterday evening we visited some friends. 1) Their daughter and our daughter
are friends. Their daughter 2) have a cat. The cat just had 8 kittens. One kitten followed
our daughter all evening. When our daughter sat 3) down the couch, the kitten sat at her
feet. When our daughter 4) to outside, the kitten followed her. When it was time to go,
the kitten followed our daughter 5) go our car. When we opened the car 6) door the kitten
jumped into the car. Our daughter’s friend gave 7) the kitten our daughter. We
8) brout it home with us. But I’m allergic to cats!
1)
A. They daughter
B. Them daughter
C. They’re daughter
D. Correct as is
2)
A. has cat
B. has a cat
C. have cat
D. Correct as is.
3)
A. on couch
B. in couch
C. on the couch
D. in the couch
4)
A. outside
B. go outside
C. go to outside
D. went outside
5)
A. to our car
B. go to our car
C. going our car
D. went to our car
6)
A. door, the
B. door. the
C. door, The
D. door. The
7)
A. our daughter the kitten
B. the kitten to our daughter
C. our daughter to the kitten
D. Both A and B are correct.
8)
Page 49 of 170
International University, IU
Test Level 3

A. brought
B. brougth
C. brouht
D. brought

Paragraph 15
I’m Elena Guedon. 1) I am from Paris France, but now I live 2) in Chicago Illinois.
3) I am divorce. I have one son named Pierre. 4) Pierre is doctor at a large hospital in
Chicago. 5) He is planning for get married. His girlfriend Cassie is a surgeon at the
hospital. She is older than he is. I am older than my 6) ex-hubsand. 7) So I am worry
about Pierre’s future. But Pierre is sure 8) that him and Cassie will be happy.
1)
A. am from Paris France, but
B. am from Paris, France, but
C. come from Paris, France, but
D. Both B and C are correct.
2)
A. on Chicago Illinois.
B. in Chicago Illinois.
C. in Chicago, Illinois.
D. from Chicago, Illinois.
3)
A. I got divorce.
B. I am a divorce.
C. I am divorced.
D. I am a divorced.
4)
A. Pierre is doctor at large hospital
B. Pierre is doctor at a hospital large
C. Pierre is a doctor at a large hospital
D. Pierre is doctor at a hospital large
5)
A. He plans to get
B. He plans for get
C. He is planning to get
D. Both A and C are correct
6)
A. x-husband
B. ex-husband
C. ex-hunsbad
D. Correct as is
7)
A. So I worry
B. So I am worried
C. So am I worried
Page 50 of 170
International University, IU
Test Level 3

D. Both A and B are correct


8)
A. him and Cassie will happy.
B. Cassie and him will happy.
C. he and Cassie will be happy.
D. he and Cassie they will be happy.

Page 51 of 170

You might also like